Sunteți pe pagina 1din 119

CARDIOLOGY

C) it often metastasises
NT - 1.1 it can mimic mitral stenosis during
These can be heard in mitral stenosis, D)
physical examination
except for: Show comment and answer
INT - 1.6
apical holosystolic murmur radiating Which one is true about the
A)
towards the axilla mechanism of action of digoxin?
low-frequency apical diastolic
B)
murmur A) it inhibits the Na-K-ATPase
C) opening snap it lowers intracellular
B)
D) loud first heart sound Na+ concentration
Show comment and answer C) it increases intracellular ATP levels
INT - 1.2 D) it enhances cAMP-production
Part of the therapy of decompensated
it decreases Ca-release from the
heart failure, except for: E)
sarcoplasmic reticulum
Show comment and answer
A) mineralocorticoid-antagonists INT - 1.7
B) diuretics All of the following statements about
C) digoxin nitroglycerine are true, except for:
D) parenteral volume expansion
E) ACE-inhibitors it increases intracellular cGMP
A)
levels
Show comment and answer
INT - 1.3 B) its primarily metabolised in the liver
Causes of acute left ventricle failure, it can induce significant reflex
C)
except for: tachycardia
it significantly prolongs AV-
D)
A) asthma bronchiale conduction
B) acute myocardial infarction E) it can lead to postural hypotension
C) hypertensive crisis Show comment and answer
D) severe aortic stenosis INT - 1.8
The typical side effect of nitrates is:
Show comment and answer
INT - 1.4
Characteristic of hypertrophic A) hypertension
obstructive cardiomyopathy, except B) headache
for: C) bradycardia
D) sexual dysfunction
might be combined with mitral E) anemia
A)
insufficiency
Show comment and answer
digoxin is important in the early INT - 1.9
B)
stage The typical features of Prinzmetal
C) often familial angina:
D) diastolic dysfunction is common
E) syncope is a common symptom ST segment depression during
A)
angina
Show comment and answer
INT - 1.5 B) negative T waves during angina
Features of atrial myxoma, except for: C) pathologic Q waves during angina
D) elevated necroenzymes
the most common form of primary
A) E) ST segment elevation during angina
cardiac tumors
Show comment and answer
it can be diagnosed with
B)
echocardiography
INT - 1.10 E) ischemic cardiomyopathy
Types of unstable angina pectoris, Show comment and answer
except for: INT - 1.15
The most important risk factor of
A) angina at rest atherosclerosis:
B) crescendo angina
C) effort angina elevated serum LDL-cholesterol
A)
D) new-onset angina level
Show comment and answer elevated serum HDL-cholesterol
B)
INT - 1.11 level
The most common pathology of C) elevated serum triglyceride level
myocardial infarction: D) elevated serum cholesterol level
Show comment and answer
A) coronary embolism INT - 1.16
B) rupture of an atherosclerotic plaque Risk factors of coronary artery disease,
C) dissection of coronary walls except for:
growing of an atherosclerotic
D) A) positive family history
plaque
E) coronary inflammation B) diabetes mellitus
Show comment and answer C) smoking
INT - 1.12 elevated serum HDL-cholesterol
D)
Normal mean electrical axis in the level
front plane, except for: E) metabolic syndrome X
Show comment and answer
A) +60° INT - 1.17
B) +90° The ideal target value of serum LDL-
C) –45° cholesterol in a diabetic patient after
myocardial infarction:
D) 0°
E) +45°
A) < 1,8 mmol/l
Show comment and answer
B) > 2,6 mmol/l
INT - 1.13
If acute myocardial infarction is C) < 3,5 mmol/l
suspected, the following diagnostic D) > 3,5 mmol/l
procedures should be carried out, Show comment and answer
except for: INT - 1.18
A 65 patient with a history of smoking
A) EKG presents with sudden pain in his left
blood tests (to measure CK-MB and leg. The limb feels cold, the toes are
B) pale and peripheral pulse can not be
troponin)
C) physical examination palpated.
D) cardiac stress test
A) deep vein thrombosis
E) echocardiography
B) Buerger’s disease
Show comment and answer
C) embolism in the peripheral arteries
INT - 1.14
Ischemic heart disease can present D) Raynaud’s disease
with, except for: Show comment and answer
INT - 1.19
A) acute myocardial infarction Paramedics arrive to a 55 year old
B) stable angina patient with chest pain that started 3
hours ago. On the patient’s ECG they
C) deep vein thrombosis
notice ST segment elevation in leads I,
D) sudden cardiac death aVL, V5 and V6. What should they do?
Accessory pathway-mediated reentry
transfer the patient to the regional tachycardy (AVRT) can be terminated
A)
Emergency Department with, except for:
transfer the patient to the regional
B) A) propafenone
Coronary Care Unit
transfer the patient to the regional B) adenosine
Cardiology Department for a C) lidocaine
C)
troponin test and if it’s positive, D) radiofrequency ablation
transfer to a PCI center.
E) verapamil
transfer the patient directly to the
D) Show comment and answer
closest PCI center.
INT - 1.24
Show comment and answer A regular wide QRS complex
INT - 1.20
tachycardia can not be:
Upon the physical examination of a 45
year old man without any symptoms a
A) ventricular tachycardia
soft systolic murmur and ejection click
can be heard in 2L2. These have been supraventricular tachycardia with
B)
known since he was a child. What is bundle branch block
the most likely diagnosis? atrial fibrilltion with bundle branch
C)
block
A) patent ductus arteriosus antidromic atrioventricular reentry
D)
B) coarctation of the aorta tachycardia (WPW-syndrome)
C) ventricular septal defect atrial flutter with bundle branch
E)
block
D) Ebstein’s anomaly
Show comment and answer
E) pulmonary valve stenosis INT - 1.25
INT - 1.21 A patient was admitted to the
Correct statements about aortic Emergency Department because of a
stenosis, except for: palpitation that started three hours
earlier. On his ECG an atrial fibrillation
A) it causes pulsus parvus et tardus. with rapid (150 bpm) ventricular
B) it can cause syncope. response was seen. His blood pressure
C) it can cause anginalike chest pain. was 130/90 Hgmm. In the patient’s
Austin-Flint murmur can be heard history there wasn’t anything that
D) indicated structural heart disease.
upon auscultation
What is the best first step in this
it causes concentric left ventricular
E) situation?
hypertrophy.
Show comment and answer
A) pharmacological cardioversion
INT - 1.22
Correct statements about aortic B) immediate electrical cardioversion
insufficiency, except for: C) coronarography
immediate anticoagulation to
D)
it causes pulsus celer et altus prevent thromboembolism
A)
(Corrigan’s pulse). E) cardiac stress test
B) it can be acute and chronic, too. Show comment and answer
it usually doesn’t lead to left INT - 1.26
C)
ventricle dilation. Which one is the most common
D) it can be congenital. permanent arrhythmia?
it predisposes to infective
E) A) ventricular extrasystoles
endocarditis.
Show comment and answer B) atrial fibrillation
INT - 1.23 C) ventricular tachycardia
D) supraventricular tachycardia
E) junctional escape rhythm who has chest pain at exertion the first
Show comment and answer step should be:
INT - 1.27
A patient who has been taking A) stress echocardiography
amiodarone for a long time was B) excercise test
prescribed a fluoroquinolone antibiotic C) stress perfusion scintigraphy
because of a respiratory infection. She
D) Holter ECG monitoring
had recurring, short-term syncopes
that had never occurred before. Which E) stress MRI
one is the most likely cause of the Show comment and answer
syncopes? INT - 1.31
The best first-choice drug for
hypotension induced by the bradycardia during myocardial
A) infarction is:
medications
torsade de pointes ventricular
B) tachycardia induced by the A) isoproterenol
medications B) theophyllin
sinus bradycardia induced by the C) atropine
C)
medications D) dobutamine
her symptoms are not induced by Show comment and answer
D) her medications, it is just a INT - 1.32
coincidence The most common side effect of ACE
AV block induced by the inhibitors is:
E)
medications
Show comment and answer A) diarrhea
INT - 1.28 B) cough
These could be the first ECG findings in C) vomiting
the acute phase of myocardial
D) erythema
infarction, except for:
E) anasarca
A) pathologic Q wave Show comment and answer
INT - 1.33
B) inverted T waves
Which disease(s) cause(s) systolic
C) ventricular fibrillation hypertension?
D) ST segment elevation
E) ST segment depression 1) aortic insufficiency
Show comment and answer 2) thyreotoxicosis
INT - 1.29 3) beriberi
Pathologic Q wave in leads II, III and 4) atherosclerosis
aVF with isoelectric ST segment and
positive T waves indicate: A) Answers 1, 2 and 3 are correct
B) Answers 1 and 3 are correct
A) acut ischemia C) Answers 2 and 4 are correct
acute phase of a progressing D) Only answer 4 is correct
B)
myocardial infarction E) All four answers are correct
C) previous myocardial infarction Show comment and answer
aneurysm after myocardial INT - 1.34
D)
infarction Aortic aneurysm can be caused by:
E) subendocardial ischemia
1) arteriosclerosis
Show comment and answer
INT - 1.30 2) Marfan’s syndrome
In the diagnostic workup of a 60 year 3) vascular syphilis
old patient with a history of smoking 4) giant-cell arteritis
A) Answers 1, 2 and 3 are correct Which one(s) is/are correct?
B) Answers 1 and 3 are correct
1) Aspirin has no effect on the
C) Answers 2 and 4 are correct prostacyclin production of the
D) Only answer 4 is correct endothelial cells.
E) All four answers are correct 2) According to multicenter trials
Show comment and answer aspirin reduces the chance of a second
INT - 1.35 myocardial infarction.
The possible cause(s) of perciarditis: 3) It takes 24 hours for heparin to
build up its anticoagulant effect.
1) uremia 4) Heparin and alteplase are
2) transmural myocardial infarction treatment options in pulmonary
3) tuberculosis embolism.
4) metastatic cancer
A) Answers 1, 2 and 3 are correct
A) Answers 1, 2 and 3 are correct B) Answers 1 and 3 are correct
B) Answers 1 and 3 are correct C) Answers 2 and 4 are correct
C) Answers 2 and 4 are correct D) Only answer 4 is correct
D) Only answer 4 is correct E) All four answers are correct
E) All four answers are correct Show comment and answer
Show comment and answer INT - 1.39
INT - 1.36 Which diuretic-side effect
Secondary cardiomyopathy can be combination(s) is/are correct?
caused by:
1) furosemide - hyperuricemia
1) hyperthyroidism 2) chlortalidone – ototoxicity
2) beriberi 3) spironolactone – gynecomasty
3) amyloidosis 4) etacrynic acid - hyperuricemia
4) glycogenosis
A) Answers 1, 2 and 3 are correct
A) Answers 1, 2 and 3 are correct B) Answers 1 and 3 are correct
B) Answers 1 and 3 are correct C) Answers 2 and 4 are correct
C) Answers 2 and 4 are correct D) Only answer 4 is correct
D) Only answer 4 is correct E) All four answers are correct
E) All four answers are correct Show comment and answer
Show comment and answer INT - 1.40
INT - 1.37 Risk factors of ischemic heart disease:
ACE inhibitors:
1) smoking
1) decrease blood pressure 2) hypercholesterolemia
2) decrease aldosterone levels 3) hypertension
3) increase bradykinin levels 4) family history
4) stop the deterioration of the left
ventricle ejection fraction A) Answers 1, 2 and 3 are correct
B) Answers 1 and 3 are correct
A) Answers 1, 2 and 3 are correct
C) Answers 2 and 4 are correct
B) Answers 1 and 3 are correct
D) Only answer 4 is correct
C) Answers 2 and 4 are correct
E) All four answers are correct
D) Only answer 4 is correct INT - 1.41
E) All four answers are correct Chest pain can be a symptom of:
Show comment and answer
INT - 1.38 1) aortic stenosis
2) mitral prolapse syndrome B) Answers 1 and 3 are correct
3) peptic ulcer C) Answers 2 and 4 are correct
4) Morgagni’s syndrome
D) Only answer 4 is correct
E) All four answers are correct
A) Answers 1, 2 and 3 are correct
Show comment and answer
B) Answers 1 and 3 are correct
INT - 1.45
C) Answers 2 and 4 are correct Diagnostic tests with the ability to
D) Only answer 4 is correct detect asymptomatic angina pectoris
E) All four answers are correct (silent ischemia):
Show comment and answer
INT - 1.42 1) dobutamine stress
Treatments that reduce morbidity and echocardiogram
mortality after myocardial infarction 2) Holter ECG monitoring
(secondary prevention): 3) exercise test
4) ABPM
1) beta-blockers
2) anti-platelet drugs A) Answers 1, 2 and 3 are correct
3) HMG-CoA reductase inhibitors B) Answers 1 and 3 are correct
4) ACE inhibitors C) Answers 2 and 4 are correct
D) Only answer 4 is correct
A) Answers 1, 2 and 3 are correct
E) All four answers are correct
B) Answers 1 and 3 are correct
Show comment and answer
C) Answers 2 and 4 are correct INT - 1.46
D) Only answer 4 is correct Early and late complications of acute
E) All four answers are correct myocardial infarction:
Show comment and answer
INT - 1.43 1) ventricular fibrillation
Diagnostic options to determine 2) left ventricular aneurysm
myocardial viability: formation
3) cardiogenic shock
1) low-dose dobutamine stress test 4) pericardial effusion
2) positron-emission tomography
3) stress perfusion scintigraphy with A) Answers 1, 2 and 3 are correct
Tl-201 reinjection B) Answers 1 and 3 are correct
4) Doppler ultrasound C) Answers 2 and 4 are correct
D) Only answer 4 is correct
A) Answers 1, 2 and 3 are correct
E) All four answers are correct
B) Answers 1 and 3 are correct
Show comment and answer
C) Answers 2 and 4 are correct INT - 1.47
D) Only answer 4 is correct Treatment option(s) of heart failure:
E) All four answers are correct
1) pharmacotherapy
Show comment and answer
INT - 1.44 2) heart transplant
Contraindications of cardiac exercise 3) mechanical circulatory support
tests: devices
4) cardiac resynchronization therapy
1) acute myocardial infarction
2) chronic heart failure A) Answers 1, 2 and 3 are correct
3) unstable angina B) Answers 1 and 3 are correct
4) beta-blocker therapy C) Answers 2 and 4 are correct
D) Only answer 4 is correct
A) Answers 1, 2 and 3 are correct
E) All four answers are correct
Show comment and answer 3) it is caused by coronary spasm
INT - 1.48 4) it should be treated with calcium-
Risk factors of atherosclerosis: channel blockers

1) stress A) Answers 1, 2 and 3 are correct


2) AV-nodal reentry-tachycardia
B) Answers 1 and 3 are correct
3) smoking
4) hypotension C) Answers 2 and 4 are correct
D) Only answer 4 is correct
A) Answers 1, 2 and 3 are correct E) All four answers are correct
B) Answers 1 and 3 are correct Show comment and answer
C) Answers 2 and 4 are correct INT - 1.52
Possible complication(s) of deep vein
D) Only answer 4 is correct
thrombosis:
E) All four answers are correct
Show comment and answer 1) pulmonary infarction
INT - 1.49 2) crural ulcer
Drugs that lower serum cholesterol 3) pulmonary embolism
levels: 4) Raynaud’s syndrome

1) rosuvastatin A) Answers 1, 2 and 3 are correct


2) ezetimibe
B) Answers 1 and 3 are correct
3) atorvastatin
4) ivabradine C) Answers 2 and 4 are correct
D) Only answer 4 is correct
A) Answers 1, 2 and 3 are correct E) All four answers are correct
B) Answers 1 and 3 are correct Show comment and answer
C) Answers 2 and 4 are correct INT - 1.53
Possible complication(s) of
D) Only answer 4 is correct
atherosclerosis:
E) All four answers are correct
Show comment and answer 1) dry gangraena of the feet
INT - 1.50 2) aortic aneurysm
Drugs that lower serum triglyceride 3) myocardial infarction
levels: 4) stroke

1) special diet A) Answers 1, 2 and 3 are correct


2) niacin
B) Answers 1 and 3 are correct
3) fibrates
4) alcohol C) Answers 2 and 4 are correct
D) Only answer 4 is correct
A) Answers 1, 2 and 3 are correct E) All four answers are correct
B) Answers 1 and 3 are correct Show comment and answer
C) Answers 2 and 4 are correct INT - 1.54
Might mimic the ECG findings of
D) Only answer 4 is correct
myocardial infarction:
E) All four answers are correct
Show comment and answer 1) pericarditis
INT - 1.51 2) pancreatitis
Characteristic features of Prinzmetal 3) myocarditis
angina: 4) pulmonary embolism

1) it usually occurs at dawn during A) Answers 1, 2 and 3 are correct


rest
B) Answers 1 and 3 are correct
2) ST segment elevation can be
seen during angina C) Answers 2 and 4 are correct
D) Only answer 4 is correct E) All four answers are correct
E) All four answers are correct Show comment and answer
Show comment and answer INT - 1.58
INT - 1.55 A 70 year old patient had an anterior
Enzyme(s) that is/are elevated in wall myocardial infarction three weeks
myocardial infarction: ago. He suddenly develops a fever and
complains of chest pain. On his ECG
1) creatine kinase (CK-MB) there are no new Q waves and his CK-
2) lactate dehydrogenase MB level is normal. What is/are the
3) troponin most likely diagnose(s)?
4) alkaline phosphatase
1) myocardial reinfarction
A) Answers 1, 2 and 3 are correct 2) pulmonary embolism
B) Answers 1 and 3 are correct 3) lobar pneumonia
4) Dressler’s syndrome
C) Answers 2 and 4 are correct
D) Only answer 4 is correct A) Answers 1, 2 and 3 are correct
E) All four answers are correct B) Answers 1 and 3 are correct
Show comment and answer C) Answers 2 and 4 are correct
INT - 1.56
D) Only answer 4 is correct
A 55 year old patient with a history of
hypertension has been rushed to the E) All four answers are correct
emergency room because of severe Show comment and answer
chest pain and ST segment elevation. INT - 1.59
Possible diagnosis/diagnoses: Correct statements about mitral
insufficiency:
1) peptic ulcer
2) acute myocardial infarction 1) it leads to the dilation of all heart
3) mitral valve prolapse chambers
4) aortic dissection 2) it might be the complication of
infective endocarditis
A) Answers 1, 2 and 3 are correct 3) left atrial pressure can be
elevated even when the ejection
B) Answers 1 and 3 are correct
fraction is preserved
C) Answers 2 and 4 are correct
4) its severe form requires surgical
D) Only answer 4 is correct treatment.
E) All four answers are correct
Show comment and answer A) Answers 1, 2 and 3 are correct
INT - 1.57 B) Answers 1 and 3 are correct
Signs of the progressive occlusion of
C) Answers 2 and 4 are correct
the arteries in the lower limbs:
D) Only answer 4 is correct
1) painful cramping (claudication) E) All four answers are correct
occurs during or after taking shorter or Show comment and answer
longer walks INT - 1.60
2) the toes feel cold Correct statements about mitral
3) in vertical position the legs look stenosis:
red, but they turn pale when elevated
4) sudden onset 1) it doesn’t increase the risk of
atrial fibrillation
A) Answers 1, 2 and 3 are correct 2) it predisposes to left atrial
thrombus formation
B) Answers 1 and 3 are correct
3) during auscultation a muffled
C) Answers 2 and 4 are correct
first heart sound and a mesosystolic
D) Only answer 4 is correct click can be heard
4) it might be a late complication of INT - 1.63
rheumatic fever Characteristic of mitral valve prolapse:

A) Answers 1, 2 and 3 are correct 1) it can cause ventricular


extrasystoles
B) Answers 1 and 3 are correct
2) it can present with chest pain
C) Answers 2 and 4 are correct 3) it is common in Marfan’s sydrome
D) Only answer 4 is correct 4) an opening snap can be heard
E) All four answers are correct during auscultation of the heart
INT - 1.61
Echocardiographic findings of aortic A) Answers 1, 2 and 3 are correct
stenosis: B) Answers 1 and 3 are correct
C) Answers 2 and 4 are correct
1) the end-diastolic diameter of the
D) Only answer 4 is correct
left ventricle is not significantly
enlarged E) All four answers are correct
2) concentric left ventricle Show comment and answer
hypertrophy can often be seen INT - 1.64
3) the left atrial diameter can be Hypertrophic cardiomyopathy:
abnormally large
4) a pathologic transvalvular 1) is caused by genetic mutations.
gradient can be measured at the level 2) can cause dynamic left ventricle
of the aortic valve with Doppler outflow obstruction.
ultrasound 3) can cause sudden cardiac death.
4) should not be treated with beta-
A) Answers 1, 2 and 3 are correct blockers.
B) Answers 1 and 3 are correct
A) Answers 1, 2 and 3 are correct
C) Answers 2 and 4 are correct
B) Answers 1 and 3 are correct
D) Only answer 4 is correct
C) Answers 2 and 4 are correct
E) All four answers are correct
D) Only answer 4 is correct
Show comment and answer
INT - 1.62
E) All four answers are correct
Echocardiographic findings of isolated Show comment and answer
mitral stenosis: INT - 1.65
The treatment of ventricular
1) large left atrial diameter extrasystoles:
2) the unidirectional movement of
the anterior and posterior leaflets in M- 1) All cases must be treated with
mode antiarrhythmic agents.
3) an abnormal transvalvular 2) Frequent, symptomatic
gradient can be measured in diastole extrasystoles require antiarrhythmic
at the level of the mitral valve with pharmacological therapy.
continuous wave Doppler ultrasound 3) Class Ic agents have been proven
4) the calcification of the whole to be the best choice.
mitral valve can be visualised with 2D- 4) In most cases antiarrhythmic
echocardiography treatment is not required.

A) Answers 1, 2 and 3 are correct A) Answers 1, 2 and 3 are correct


B) Answers 1 and 3 are correct B) Answers 1 and 3 are correct
C) Answers 2 and 4 are correct C) Answers 2 and 4 are correct
D) Only answer 4 is correct D) Only answer 4 is correct
E) All four answers are correct E) All four answers are correct
Show comment and answer Show comment and answer
INT - 1.66
In the differential diagnosis of wide Show comment and answer
QRS complex tachycardia helps: INT - 1.69
It’s true about the proarrhythmic effect
1) Knowing the organic status of the of antiarrhythmic drugs:
heart
2) Frequency of tachycardia 1) Organic heart disease does not
3) Physical or ECG signs of have influence on the proarrhythmic
atrioventricular dissociation risk
4) Hemodynamic instability of the 2) Dangerous proarrhythmia is most
tachycardia commonly caused by beta blockers
3) Chinidin has the least
A) Answers 1, 2 and 3 are correct proarrhythmic potential
4) Antiarrhythmic drugs in some
B) Answers 1 and 3 are correct
cases cause different type, often worse
C) Answers 2 and 4 are correct
arrhythmia than the arrhythmia which
D) Only answer 4 is correct made the use of these types of drugs
E) All four answers are correct reasonable. This effect is called
Show comment and answer proarrhythmia.
INT - 1.67
Principles of the treatment of wide QRS A) Answers 1, 2 and 3 are correct
complex tachycardia: B) Answers 1 and 3 are correct
C) Answers 2 and 4 are correct
1) If we are not sure in the
mechanism, we should treat it as D) Only answer 4 is correct
ventricular tachycardia E) All four answers are correct
2) Carotid sinus massage should be Show comment and answer
tried first before the medical INT - 1.70
treatment. It is true about long QT syndrome:
3) Immediate synchronized
cardioversion is required in case of 1) Long QT syndrome is most
hemodynamic instability. commonly the consequence of drug
4) Always start the treatment with adverse reaction.
group 1/C agent. 2) Long QT syndrome predispose to
the development of potentially lethal
A) Answers 1, 2 and 3 are correct arrhythmia.
3) Beta blockers are appropriate for
B) Answers 1 and 3 are correct
the treatment of long QT syndrome if
C) Answers 2 and 4 are correct necessary with pacemaker
D) Only answer 4 is correct implantation.
E) All four answers are correct 4) Congenital long QT syndrome can
Show comment and answer be associated with deafness.
INT - 1.68
Etiologic factors of atrial fibrillation: A) Answers 1, 2 and 3 are correct
B) Answers 1 and 3 are correct
1) Hyperthyroidism
C) Answers 2 and 4 are correct
2) Mitral valve disease
3) Cardiomyopathy D) Only answer 4 is correct
4) Ischemic heart disease E) All four answers are correct
Show comment and answer
A) Answers 1, 2 and 3 are correct INT - 1.71
B) Answers 1 and 3 are correct It is true about ventricular
tachycardias:
C) Answers 2 and 4 are correct
D) Only answer 4 is correct 1) The prognosis of ventricular
E) All four answers are correct tachycardia is defined by possible
organic heart disease and by the B) Answers 1 and 3 are correct
function of left ventricle. C) Answers 2 and 4 are correct
2) Ventricular tachycardia is most
D) Only answer 4 is correct
common in ischemic heart disease,
after myocardial infarction. E) All four answers are correct
3) The origin point of ventricle Show comment and answer
tachycardia can be deduced based on INT - 1.74
the type of bundle branch block which The following can be used in the
is seen on 12-lead ECG and on the treatment of pulmonary edema which
frontal plane axis. is associated with myocardial
4) Digitalis is the most important infarction:
drug in the medical treatment of
1) Intravenous furosemide
ventricular tachycardia.
2) Oral verapamil
3) Inhale of oxygen
A) Answers 1, 2 and 3 are correct
4) Nitroglycerin transdermal patch
B) Answers 1 and 3 are correct
C) Answers 2 and 4 are correct A) Answers 1, 2 and 3 are correct
D) Only answer 4 is correct B) Answers 1 and 3 are correct
E) All four answers are correct C) Answers 2 and 4 are correct
Show comment and answer D) Only answer 4 is correct
INT - 1.72
E) All four answers are correct
Myocardial infarction could have the
following symptoms: Show comment and answer
INT - 1.75
1) Back pain The use of the following decreases the
2) Chest pain patient’s mortality with heart failure:
3) Sweating
1) Nifedipine
4) Epigastric pain
2) ACE-inhibitor
3) Diuretic
A) Answers 1, 2 and 3 are correct
4) Beta blockers
B) Answers 1 and 3 are correct
C) Answers 2 and 4 are correct A) Answers 1, 2 and 3 are correct
D) Only answer 4 is correct B) Answers 1 and 3 are correct
E) All four answers are correct C) Answers 2 and 4 are correct
Show comment and answer D) Only answer 4 is correct
INT - 1.73
E) All four answers are correct
To do in case of typical infarction chest
pain is present for longer than one Show comment and answer
INT - 1.76
hour and ST-elevation of more than 1
The following could be the cause of a
mm is detected between two ECG-
left sided heart failure:
leads:
1) Untreated hypertension
1) To take rest myocardial perfusion
2) Viral myocarditis
scintigraphy
3) Aortic stenosis
2) Send the patient to hospital
4) Deep vein thrombosis
where percutaneous coronary
intervention can be performed
A) Answers 1, 2 and 3 are correct
3) To determine the next action
diagnosticate serum-necroenzym level B) Answers 1 and 3 are correct
4) Strict monitoring to detect C) Answers 2 and 4 are correct
arrhythmia D) Only answer 4 is correct
E) All four answers are correct
A) Answers 1, 2 and 3 are correct
Show comment and answer
INT - 1.77 A) Hypertensive crisis
Symptoms of heart failure: B) Third-degree atrioventricular block
C) Edema of the pulmonary- and
1) Paralytic ileus systemic circulation
2) Edema of legs D) Ventricular sustained tachycardia
3) Joint pain with reduced left ventricular
4) Dyspnoea function
Association question
A) Answers 1, 2 and 3 are correct Match the clinical aspects with the
B) Answers 1 and 3 are correct related tests!
C) Answers 2 and 4 are correct
INT - Mitral valve prolapse -
D) Only answer 4 is correct
1.83
E) All four answers are correct INT - Detect of rejection after -
Show comment and answer 1.84 heart transplantation
INT - 1.78 INT - Post myocardial infarction, -
Symptoms of heart failure: 1.85 detection of viability
INT - Sick sinus syndrome -
1) Nocturia
1.86
2) Paresis of lower extremity
INT - Suspected intracardiac -
3) Tachycardia
1.87 thrombus (for example
4) Trophic disorders of the skin of
underlying chronic
upper extremities
embolisation)
A) Answers 1, 2 and 3 are correct
A) Transesophageal echocardiography
B) Answers 1 and 3 are correct B) Heart muscle biopsy
C) Answers 2 and 4 are correct C) Cardiac MRI
D) Only answer 4 is correct D) 2D, color Doppler-
E) All four answers are correct echocardiography
E) 24-hour Holter monitoring (EKG)
Show comment and answer
Association question
Association question Match the side-effects with each
Pair the symptoms and the therapies! medicine!

INT - 1.79 amiodarone - INT - 1.88 Bronchoconstriction -

INT - 1.80 furosemide - INT - 1.89 Cold extremities -

INT - 1.81 urapidil (Ebrantil) - INT - 1.90 Pulmonary fibrosis -

INT - 1.82 pacemaker-therapy - INT - 1.91 Hyperthyroidism -


INT - 1.92 Angioneurotic edema -

A) Hypertensive crisis INT - 1.93 Dry cough -

B) Third-degree atrioventricular block


C) Edema of the pulmonary- and A) amiodarone
systemic circulation B) ACE-inhibitor
D) Ventricular sustained tachycardia C) β - blocker
with reduced left ventricular
function
Association question
Pair the symptoms and the therapies!

INT - 1.79 amiodarone -


INT - 1.80 furosemide -
INT - 1.81 urapidil (Ebrantil) -
INT - 1.82 pacemaker-therapy -
Association question The aortic stenosis cause increased left
Match the medicines which are used in ventricle load, because of that the
the therapy of ischemic heart disease consequence is left ventricular
with their characteristic properties: hypertrophy.

INT - Reduction of the frequency -


Both of them are correct, there is
1.94 and the left ventricular A)
causal relationship between them
contractility Both of them are correct, but there
INT - Mainly reducing the preload -
B) is no causal relationship between
1.95 them
INT - Inhibiting the platelets -
The first part is correct, the second
1.96 aggregation C)
one is wrong
INT - They have lipid reducing -
The first part is wrong, the second
1.97 and pleiotropic effect D)
one is correct
A) Salicylates E) Both of them are incorrect
B) HMG CoA reductase inhibitors Show comment and answer
C) Beta-blockers INT - 1.103
D) Nitrates Some of the diuretics which are used
Association question in the treatment of heart failure may
Match the effect with the right lead to hypokalemia, therefore
medicine! diuretics should always be given with
potassium supplementation.
INT - Antiarrhythmic effect -
1.98 Both of them are correct, there is
A)
INT - Platelet aggregation - causal relationship between them
1.99 inhibiting effect Both of them are correct, but there
INT - „funny” Na-K-channel - B) is no causal relationship between
1.100 inhibitor them
INT - Anticoagulant effect - The first part is correct, the second
C)
1.101 one is wrong
The first part is wrong, the second
A) Platelet ADP-receptor inhibitor D)
one is correct
B) Ivabradine E) Both of them are incorrect
C) Rivaroxaban
Show comment and answer
D) Dronedarone INT - 1.104
Association question
Anticoagulant therapy isn’t required
Match the effect with the right
after myocardial infarction with left
medicine!
ventricular aneurysm because in this
state the risk of intracardiac thrombus
INT - Antiarrhythmic effect -
is low.
1.98
INT - Platelet aggregation -
Both of them are correct, there is
1.99 inhibiting effect A)
causal relationship between them
INT - „funny” Na-K-channel -
1.100 inhibitor Both of them are correct, but there
INT - Anticoagulant effect - B) is no causal relationship between
them
1.101
The first part is correct, the second
C)
A) Platelet ADP-receptor inhibitor one is wrong
B) Ivabradine The first part is wrong, the second
D)
C) Rivaroxaban one is correct
D) Dronedarone E) Both of them are incorrect
INT - 1.102 Show comment and answer
INT - 1.105 E) Both of them are incorrect
For patients who had myocardial Show comment and answer
infarction aspirin and beta-blockers INT - 1.108
treatments are required, because Sudden cardiac death is common
these agents in secondary prevention during heart failure, because the
have positive effect on the mortality predisposition of malignant ventricular
which was proven in multicenter arrhythmia can’t be predisposed in
studies. every case even with
electrophysiological study.
Both of them are correct, there is
A)
causal relationship between them Both of them are correct, there is
A)
Both of them are correct, but there causal relationship between them
B) is no causal relationship between Both of them are correct, but there
them B) is no causal relationship between
The first part is correct, the second them
C)
one is wrong The first part is correct, the second
C)
The first part is wrong, the second one is wrong
D)
one is correct The first part is wrong, the second
D)
E) Both of them are incorrect one is correct
Show comment and answer E) Both of them are incorrect
INT - 1.106
Show comment and answer
During pregnancy taking Syncumar INT - 1.109
(acenocoumarol) is absolutely The abnormally prolonged QT-interval
contraindicated, because Syncumar predisposes to severe ventricular
overdose could cause severe bleeding. arrhythmia because the normal length
of QT interval physiologically depends
Both of them are correct, there is from the ventricular frequency.
A)
causal relationship between them
Both of them are correct, but there Both of them are correct, there is
B) is no causal relationship between A)
causal relationship between them
them Both of them are correct, but there
The first part is correct, the second B) is no causal relationship between
C)
one is wrong them
The first part is wrong, the second The first part is correct, the second
D) C)
one is correct one is wrong
E) Both of them are incorrect The first part is wrong, the second
D)
Show comment and answer one is correct
INT - 1.107 E) Both of them are incorrect
ACE-inhibitors are required in the Show comment and answer
treatment of chronic heart failure INT - 1.110
because multicenter studies have The usage of fast-acting nitrates
proven the positive effect on the decrease the oxygen demand of the
mortality. myocardium, because their vasodilator
effect reduces the ventricular pre-load.
Both of them are correct, there is
A)
causal relationship between them Both of them are correct, there is
A)
Both of them are correct, but there causal relationship between them
B) is no causal relationship between Both of them are correct, but there
them B) is no causal relationship between
The first part is correct, the second them
C)
one is wrong The first part is correct, the second
The first part is wrong, the second C)
D) one is wrong
one is correct
The first part is wrong, the second The first part is wrong, the second
D) D)
one is correct one is correct
E) Both of them are incorrect E) Both of them are incorrect
Show comment and answer Show comment and answer
INT - 1.111 INT - 1.114
The Prinzmetal angina is also called Transesophageal echocardiography
vasospastic angina because all the could be required before the
arteries have increased spasmodism. cardioversion of atrial fibrillation
because diagnosing left atrial
Both of them are correct, there is thrombus with transesophageal
A)
causal relationship between them echocardiography could make the
Both of them are correct, but there cardioversion necessary to be
B) is no causal relationship between postponed.
them
The first part is correct, the second Both of them are correct, there is
C) A)
one is wrong causal relationship between them
The first part is wrong, the second Both of them are correct, but there
D) B) is no causal relationship between
one is correct
them
E) Both of them are incorrect
The first part is correct, the second
Show comment and answer C)
INT - 1.112
one is wrong
When acute myocardial infarction The first part is wrong, the second
D)
occurs it is rightaway indicated by CK one is correct
(creatinine phos) enzyme because the E) Both of them are incorrect
CK-MB fraction is specific for the Show comment and answer
detection of the damage of INT - 1.115
cardiomyocytes. Transesophageal and transthoracic
echocardiography have equal value in
Both of them are correct, there is the diagnosis of infectious endocarditis
A)
causal relationship between them because the mostly significant
Both of them are correct, but there structural differences caused by
B) is no causal relationship between infectious endocarditis can be detected
them easily with both of the procedures.
The first part is correct, the second
C) Both of them are correct, there is
one is wrong A)
The first part is wrong, the second causal relationship between them
D) Both of them are correct, but there
one is correct
E) Both of them are incorrect B) is no causal relationship between
them
Show comment and answer
INT - 1.113 The first part is correct, the second
C)
Silent ischemia should be treated in one is wrong
the same way as angina pectoris The first part is wrong, the second
D)
because the prognosis is similar to one is correct
symptomatic angina pectoris E) Both of them are incorrect
Show comment and answer
Both of them are correct, there is INT - 1.116
A)
causal relationship between them Transesophageal echocardiography has
Both of them are correct, but there an important role in the diagnosis of
B) is no causal relationship between aortic dissection because most part of
them the thoracic aorta can be visualized
The first part is correct, the second with good resolution.
C)
one is wrong
Both of them are correct, there is acquired valvular heart disease
A) because mechanical prosthetic valves
causal relationship between them
Both of them are correct, but there could cause fetal damage by
B) is no causal relationship between autoimmune mechanism.
them
The first part is correct, the second Both of them are correct, there is
C) A)
one is wrong causal relationship between them
The first part is wrong, the second Both of them are correct, but there
D) B) is no causal relationship between
one is correct
them
E) Both of them are incorrect
The first part is correct, the second
Show comment and answer C)
one is wrong
INT - 1.117
The laboratory diagnostic of infectious The first part is wrong, the second
D)
endocarditis doesn’t require to take one is correct
series of hemoculture because the E) Both of them are incorrect
pathogen usually breed from the first Show comment and answer
hemoculture despite of previous INT - 1.120
antibiotic treatment. For patients who underwent
myocardial infarction it is not
Both of them are correct, there is recommend to take load test after
A) discharge from hospital because this
causal relationship between them
Both of them are correct, but there test in case of acute myocardial
B) is no causal relationship between infarction can cause left ventricular
them free wall rupture.
The first part is correct, the second
C) Both of them are correct, there is
one is wrong A)
causal relationship between them
The first part is wrong, the second
D) Both of them are correct, but there
one is correct
B) is no causal relationship between
E) Both of them are incorrect
them
Show comment and answer
The first part is correct, the second
INT - 1.118 C)
one is wrong
For patients with hemodynamically
significant aortic stenosis the presence The first part is wrong, the second
D)
of angina pectoris and syncope are bad one is correct
prognostic signs because in this clinical E) Both of them are incorrect
situation even cardiac surgery doesn’t INT - 1.121
change the long-term clinical course of Beta blockers are not recommended in
the disease. case of hypertrophic cardiomyopathy
because these drugs could increase the
Both of them are correct, there is left ventricle outflow tract obstruction.
A)
causal relationship between them
Both of them are correct, but there Both of them are correct, there is
A)
B) is no causal relationship between causal relationship between them
them Both of them are correct, but there
The first part is correct, the second B) is no causal relationship between
C) them
one is wrong
The first part is wrong, the second The first part is correct, the second
D) C)
one is correct one is wrong
E) Both of them are incorrect The first part is wrong, the second
D)
one is correct
Show comment and answer
INT - 1.119 E) Both of them are incorrect
Biological valve replacement can be Show comment and answer
required in fertile women who have INT - 1.122
Eosiniphilia in the bloodstream is often paroxysmal supraventricular
observed in Löffler type of endocarditis tachycardia, because success rate is
because this type of endocarditis can 90% and complications occur only in
cause restrictive cardiomyopathy. 1-2%.

Both of them are correct, there is Both of them are correct, there is
A) A)
causal relationship between them causal relationship between them
Both of them are correct, but there Both of them are correct, but there
B) is no causal relationship between B) is no causal relationship between
them them
The first part is correct, the second The first part is correct, the second
C) C)
one is wrong one is wrong
The first part is wrong, the second The first part is wrong, the second
D) D)
one is correct one is correct
E) Both of them are incorrect E) Both of them are incorrect
Show comment and answer Show comment and answer
INT - 1.123 INT - 1.126
Most types of diuretics require Implantable cardioverter defibrillator
potassium supplementation because can be used as the treatment of
most of them increase the potassium arrhythmia because these devices can
excretion with urine. cure the underlying disease which
caused the arrhythmia.
Both of them are correct, there is
A)
causal relationship between them Both of them are correct, there is
A)
Both of them are correct, but there causal relationship between them
B) is no causal relationship between Both of them are correct, but there
them B) is no causal relationship between
The first part is correct, the second them
C)
one is wrong The first part is correct, the second
C)
The first part is wrong, the second one is wrong
D)
one is correct The first part is wrong, the second
D)
E) Both of them are incorrect one is correct
Show comment and answer E) Both of them are incorrect
INT - 1.124 Show comment and answer
Long-term use of ACE-inhibitors INT - 1.127
reduce the left ventricle hypertrophy Sporadic ventricular extrasystole for
because their use decreases the serum healthy patients without significant
bradykinin level. complaints also require antiarrhythmic
treatment because antiarrhythmic
Both of them are correct, there is medication clearly improve the
A)
causal relationship between them survival.
Both of them are correct, but there
B) is no causal relationship between Both of them are correct, there is
A)
them causal relationship between them
The first part is correct, the second Both of them are correct, but there
C) B) is no causal relationship between
one is wrong
The first part is wrong, the second them
D) The first part is correct, the second
one is correct
C)
E) Both of them are incorrect one is wrong
Show comment and answer The first part is wrong, the second
D)
INT - 1.125 one is correct
Radiofrequency ablation is the first E) Both of them are incorrect
choice treatment of the symptomatic Show comment and answer
INT - 1.128 E) Both of them are incorrect
The anticoagulant treatment of atrial

fibrillation is based on the CHADS2-
VASc2 Score because the DISORDERS OF GALLBLADDER AND
thromboembolic risk increasing effect BILEDUCTS
of atrial fibrillation depends on the
patient’s other clinical features. INT - 8.1
Which one is the most important factor
Both of them are correct, there is in gall clearence?
A)
causal relationship between them
Both of them are correct, but there A) gastrin
B) is no causal relationship between B) nitrogen monoxide
them C) secretin
The first part is correct, the second D) cholecystokinin
C)
one is wrong
E) VIP
The first part is wrong, the second
D) Show comment and answer
one is correct INT - 8.2
E) Both of them are incorrect Which one is the most frequent
Show comment and answer gallstone?
INT - 1.129
Paroxysmal supraventricular A) Cholesterol stone
tachycardia is a mostly lethal B) Bilirubin stone
arrhythmia therefore these patients
C) Mixed stone
require combined antiarrhythmic
treatment. D) Oxalate stone
Show comment and answer
Both of them are correct, there is INT - 8.3
A) Which one is the most frequent
causal relationship between them
Both of them are correct, but there complication of an ERCP?
B) is no causal relationship between
them A) Perforation of the duodenum
The first part is correct, the second B) Infection of the bile ducts
C)
one is wrong C) Acute pancreatitis
The first part is wrong, the second Hemorrhage of the major duodenal
D) D)
one is correct papilla
E) Both of them are incorrect E) Paralytic ileus
Show comment and answer Show comment and answer
INT - 1.130 INT - 8.4
In case of acute myocardial infarction, Which symptom is not characteristic to
the use of beta blockers is preferred if disorders of biliary excretion?
there is no contraindication because
beta blockers reduce the oxygen A) abdominal distension
demand of the myocardium. B) ”fullness” feeling
C) severe steatorrhoea
Both of them are correct, there is
A) D) abdominal discomfort
causal relationship between them
E) pain under the right ribs
Both of them are correct, but there
B) is no causal relationship between Show comment and answer
them INT - 8.5
How often is the outlet of the ductus
The first part is correct, the second
C) choledochus and the ductus
one is wrong
pancreaticus joint (Ampulla of Vater)?
The first part is wrong, the second
D)
one is correct
A) 10%
B) 30%
C) 50% 1st, 2nd and 3rd answers are
A)
correct
D) 70%
B) 1st and 3rd answers are correct
E) 90%
C) 2nd and 4th answers are correct
Show comment and answer
INT - 8.6 D) only 4th answer is correct
Which of the following agents facilitate E) all of the answers are correct
biliary excretion? Show comment and answer
INT - 8.9
1) CCK Which of the following does decrease
2) secretin the spasm of the Oddi sphincter?
3) bile salts
4) adrenalin 1) nitrates
2) chocolate
A)
1st, 2nd and 3rd answers are 3) cholecystokinin
correct 4) morphin
B) 1st and 3rd answers are correct
C) 2nd and 4th answers are correct 1st, 2nd and 3rd answers are
A)
correct
D) only 4th answer is correct
B) 1st and 3rd answers are correct
E) all of the answers are correct
C) 2nd and 4th answers are correct
Show comment and answer
INT - 8.7 D) only 4th answer is correct
Which are the factors predisposing to E) all of the answers are correct
gallstone formation? Show comment and answer
INT - 8.10
1) pregnancy What are the indications of endoscopic
2) sudden dieting sphincterotomy?
3) meal rich in unsaturated fatty
acids 1) choledocholithiasis
4) resection of the small intestine 2) Oddi-sphincter-dyskinesis
3) acute recidive gallbladder attack
A)
1st, 2nd and 3rd answers are resulting in acute pancreatitis
correct 4) carcinoma of the pancreatic head
B) 1st and 3rd answers are correct
C) 2nd and 4th answers are correct 1st, 2nd and 3rd answers are
A)
correct
D) only 4th answer is correct
B) 1st and 3rd answers are correct
E) all of the answers are correct
C) 2nd and 4th answers are correct
Show comment and answer
INT - 8.8 D) only 4th answer is correct
What is the procedure to choose in E) all of the answers are correct
case of incidentally discovered Show comment and answer
gallstones not causing any INT - 8.11
symptomes? Which of the following procedures are
useful for functional examination of the
1) surgical removal of the gallbladder?
gallbladder
2) lithotripsy performed with 1) liver scintigraphy
ultrasound 2) MRCP (magnetic resonance
3) starting a lasting gallstone cholangiopancreatography)
solubilizer medication 3) intravenous urography
4) informing the patient of the 4) cholescintigraphy
gallstones and personalized
therapeutic consideration
1st, 2nd and 3rd answers are 1st, 2nd and 3rd answers are
A) A)
correct correct
B) 1st and 3rd answers are correct B) 1st and 3rd answers are correct
C) 2nd and 4th answers are correct C) 2nd and 4th answers are correct
D) only 4th answer is correct D) only 4th answer is correct
E) all of the answers are correct E) all of the answers are correct
Show comment and answer Show comment and answer
INT - 8.12 INT - 8.15
Which enzyme of the bile contributes What is the most important
to fat digestion? differentiating laboratory parameter in
cases of obstructive jaundice?
1) lipase
2) cholic acid 1) elevated direct bilirubin
3) colipase 2) elevated total bilirubin
4) neither of the aforementioned 3) elevated urinary UBG
4) no UBG in the urine
1st, 2nd and 3rd answers are
A)
correct 1st, 2nd and 3rd answers are
A)
B) 1st and 3rd answers are correct correct
C) 2nd and 4th answers are correct B) 1st and 3rd answers are correct
D) only 4th answer is correct C) 2nd and 4th answers are correct
E) all of the answers are correct D) only 4th answer is correct
Show comment and answer E) all of the answers are correct
INT - 8.13 Show comment and answer
What is the Courvoisier-sign? INT - 8.16
What is the most important role of the
1) pain under the right ribs + bile during digestion?
jaundice + fever
2) jaundice + melaena + epigastrial 1) exretion of bile acids from the
pain body
3) fever + jaundice + tactile 2) fat emulgeation
resistance in the upper abdomen 3) activation of trypsin
4) tactile, not painful enlarged 4) contributing to lipase’s
gallbladder connection to the fat

1st, 2nd and 3rd answers are 1st, 2nd and 3rd answers are
A) A)
correct correct
B) 1st and 3rd answers are correct B) 1st and 3rd answers are correct
C) 2nd and 4th answers are correct C) 2nd and 4th answers are correct
D) only 4th answer is correct D) only 4th answer is correct
E) all of the answers are correct E) all of the answers are correct
Show comment and answer Show comment and answer
INT - 8.14 INT - 8.17
Which of the following diseases can Which statements are true regarding
cause direct hyperbilirubinaemia? biliary functions and blood cholesterol
levels?
1) Gilbert’s syndrome
2) Rotor syndrome 1) Elevated biliary functions elevate
3) haemolysis blood cholesterol levels.
4) choledocholithiasis 2) Elevated biliary functions
decrease blood cholesterol levels.
3) Elevated biliary functions elevate only in permanently functionally
LDL levels. damaged gallbladders.
4) Bile is the only elimination route
for cholesterol from the body. both the statement and the
A) explanation are true and a causal
1st, 2nd and 3rd answers are relationship exists between them;
A)
correct both the statement and the
B) 1st and 3rd answers are correct B) explanation are true but there is no
C) 2nd and 4th answers are correct causal relationship between them;
D) only 4th answer is correct the statement is true, but the
C)
explanation is false;
E) all of the answers are correct
the statement is false, but the
Show comment and answer D)
explanation itself is true
INT - 8.18
Which of the following may be both the statement and the
E)
complication of gallstones? explanation are false
INT - 8.21
1) obstructive jaundice Sudden dieteing does not predispose
2) acute pancreatitis to gallstone formation, because
3) cholecystitis environment in non-functioning
4) chronic pancreatitis gallbladders favors gallstone
formation.
1st, 2nd and 3rd answers are
A) both the statement and the
correct
B) 1st and 3rd answers are correct A) explanation are true and a causal
relationship exists between them;
C) 2nd and 4th answers are correct
both the statement and the
D) only 4th answer is correct B) explanation are true but there is no
E) all of the answers are correct causal relationship between them;
Show comment and answer the statement is true, but the
INT - 8.19 C)
explanation is false;
Solubizing the gallstone which is
the statement is false, but the
smaller than 2 cm and has been D)
explanation itself is true
formed during pregancy is advised
both the statement and the
later, because pregnancy is only a E)
explanation are false
temporary state predisposing to
gallstone formation. Show comment and answer
INT - 8.22
Laparoscopic cholecystectomy is a
both the statement and the
lower load for the patient compared to
A) explanation are true and a causal
relationship exists between them; open cholecystectomy, because the
abdomen is not opened in a large
both the statement and the
surface.
B) explanation are true but there is no
causal relationship between them;
both the statement and the
the statement is true, but the A) explanation are true and a causal
C)
explanation is false; relationship exists between them;
the statement is false, but the both the statement and the
D)
explanation itself is true B) explanation are true but there is no
both the statement and the causal relationship between them;
E)
explanation are false the statement is true, but the
Show comment and answer C)
explanation is false;
INT - 8.20
the statement is false, but the
All gallstones should be surgically D)
explanation itself is true
removed, because gallstone is formed
both the statement and the both the statement and the
E)
explanation are false B) explanation are true but there is no
Show comment and answer causal relationship between them;
INT - 8.23 the statement is true, but the
C)
Abdominal ultrasound examination can explanation is false;
detect gallbladder carcinoma with the statement is false, but the
great security, therefore gallbladdder D)
explanation itself is true
carcinoma is usually detected at early both the statement and the
stage. E)
explanation are false
Show comment and answer
both the statement and the INT - 8.26
A) explanation are true and a causal Klatskin tumour can be easily removed
relationship exists between them; surgically, because the tumour is
both the statement and the located in the porta hepatis.
B) explanation are true but there is no
causal relationship between them; both the statement and the
the statement is true, but the A) explanation are true and a causal
C)
explanation is false; relationship exists between them;
the statement is false, but the both the statement and the
D)
explanation itself is true B) explanation are true but there is no
both the statement and the causal relationship between them;
E)
explanation are false the statement is true, but the
C)
Show comment and answer explanation is false;
INT - 8.24 the statement is false, but the
Gallstones are generated in the D)
explanation itself is true
gallbladder, because in the gallbladder both the statement and the
the concentration of the bile is at least E)
explanation are false
fivefold.
Show comment and answer
INT - 8.27
both the statement and the The common bile duct (ductus
A) explanation are true and a causal choledochus) usually leads to the
relationship exists between them; duodenum through the minor papilla,
both the statement and the because the duct of Wirsung leads to
B) explanation are true but there is no the duodenum through the ampulla of
causal relationship between them; Vater.
the statement is true, but the
C)
explanation is false; both the statement and the
the statement is false, but the A) explanation are true and a causal
D)
explanation itself is true relationship exists between them;
both the statement and the both the statement and the
E)
explanation are false B) explanation are true but there is no
Show comment and answer causal relationship between them;
INT - 8.25 the statement is true, but the
C)
Treatment of chronic abdominal explanation is false;
complaints occurring after the statement is false, but the
cholecystectomy is easy, because in D)
explanation itself is true
these cases with the introduction of both the statement and the
antacid treatment most symptoms can E)
explanation are false
be ceased.
Show comment and answer
INT - 8.28
both the statement and the Ultrasound scan detects bile duct
A) explanation are true and a causal stones with great certainty, because
relationship exists between them;
ultrasound is absolute reliable in the both the statement and the
E)
detection of gallbladder stones as well. explanation are false
Show comment and answer
both the statement and the INT - 8.31
A) explanation are true and a causal In case of gallstones the first choice of
relationship exists between them; treatment is always medical stone
both the statement and the dissolution, because it is cheap and
B) explanation are true but there is no only 10% of gallstones recur within
causal relationship between them; two years.
the statement is true, but the
C)
explanation is false; both the statement and the
the statement is false, but the A) explanation are true and a causal
D) relationship exists between them;
explanation itself is true
both the statement and the both the statement and the
E) B) explanation are true but there is no
explanation are false
causal relationship between them;
Show comment and answer
INT - 8.29 the statement is true, but the
C)
The surgical solution of bile duct explanation is false;
stenoses is advised, when the the statement is false, but the
D)
intervention with ERCP is unsuccessful, explanation itself is true
because surgical intervention is both the statement and the
E)
cheaper than stent exchanges. explanation are false
Show comment and answer
both the statement and the INT - 8.32
A) explanation are true and a causal Gallstone production is common in
relationship exists between them; small bowel diseases, because in these
both the statement and the cases the reabsorption of bile acids
B) explanation are true but there is no that keep the bile in solution is
causal relationship between them; damaged.
the statement is true, but the
C)
explanation is false; both the statement and the
the statement is false, but the A) explanation are true and a causal
D) relationship exists between them;
explanation itself is true
both the statement and the both the statement and the
E) B) explanation are true but there is no
explanation are false
causal relationship between them;
Show comment and answer
INT - 8.30 the statement is true, but the
C)
Prophylactic antibiotic therapy in explanation is false;
obstructive jaundice decreases the the statement is false, but the
D)
complication rate of ERCP, because the explanation itself is true
assurance of free bile flow is of both the statement and the
E)
secondary importance. explanation are false
Show comment and answer
both the statement and the INT - 8.33
A) explanation are true and a causal What is the first choice of
relationship exists between them; examination?
both the statement and the
B) explanation are true but there is no The 45-year-old woman with known
causal relationship between them; bile stones complains of a cramping,
the statement is true, but the severe abdominal pain under the right
C) ribs irradiating in the scapulae,
explanation is false;
beginning about 30 minutes after
the statement is false, but the
D) eating and associated by nausea.
explanation itself is true
During physical examination she
indicates a strong pain on pressure C) gastric perforation
under the right ribs, and fluctuating D) hydrops vesicae felleae
resistance can be felt here.
E) choledocholithiasis
Show comment and answer
A) abdominal CT
INT - 8.36
B) stomach X-ray What therapuetic interventions are
C) gastroscopy considered?
D) abdominal US
The 45-year-old woman with known
E) chest X-ray
bile stones complains of a cramping,
Show comment and answer severe abdominal pain under the right
INT - 8.34
ribs irradiating in the scapulae,
What lab test helps setting the correct
beginning about 30 minutes after
diagnosis?
eating and associated by nausea.
The 45-year-old woman with known During physical examination she
bile stones complains of a cramping, indicates a strong pain on pressure
severe abdominal pain under the right under the right ribs, and fluctuating
ribs irradiating in the scapulae, resistance can be felt here.
beginning about 30 minutes after
1) inserting a central venous
eating and associated by nausea.
catheter and administering nitrates
During physical examination she
2) laparoscopic cholecystectomy
indicates a strong pain on pressure
3) endoscopic sphincterotomy and
under the right ribs, and fluctuating
stone extraction
resistance can be felt here.
4) complete fasting, conservative
1) white blood cell count medical treatment
2) serum-amylase
3) total urine 1st, 2nd and 3rd answers are
A)
4) direct bilirubin correct
B) 1st and 3rd answers are correct
1st, 2nd and 3rd answers are C) 2nd and 4th answers are correct
A)
correct D) only 4th answer is correct
B) 1st and 3rd answers are correct E) all of the answers are correct
C) 2nd and 4th answers are correct Show comment and answer
D) only 4th answer is correct INT - 8.37
E) all of the answers are correct What is the next examination of
choice?
Show comment and answer
INT - 8.35
The 78-year-old man is admitted to
What is the most probable diagnosis?
the hospital because of significant
The 45-year-old woman with known weight loss and jaundice. Abdominal
bile stones complains of a cramping, US confirms a gallbladder without
severe abdominal pain under the right stone and dilated intrahepatic bile
ribs irradiating in the scapulae, ducts, the extrahepatic bile ducts are
beginning about 30 minutes after not wider.
eating and associated by nausea.
During physical examination she A) iv. cholescintigraphy
indicates a strong pain on pressure B) abdominal CT
under the right ribs, and fluctuating C) ERCP
resistance can be felt here. D) percutaneous transhepatic drainage
Show comment and answer
A) acute pancreatitis
B) gallstone ileus
INT - 8.38 A) bypass surgery
Which laboratory results help in setting
B) ERCP, stent implantation
the diagnosis?
C) duodenal tube
The 78-year-old man is admitted to D) treatment with ursodeoxycholic acid
the hospital because of significant E) bedrest, supportive treatment
weight loss and jaundice. Abdominal Show comment and answer
US confirms a gallbladder without INT - 8.41
stone and dilated intrahepatic bile Typical symptoms of Caroli syndrome:
ducts, the extrahepatic bile ducts are
not wider. 1) recurring cholangitis
2) haematuria
1) normal CRP level 3) shaking chills, fever
2) elevated erythrocyte 4) peptic duodenal ulcer
sedimentation rate
3) high LDH level 1st, 2nd and 3rd answers are
4) elevated CEA level A)
correct
B) 1st and 3rd answers are correct
1st, 2nd and 3rd answers are
A) C) 2nd and 4th answers are correct
correct
D) only 4th answer is correct
B) 1st and 3rd answers are correct
C) 2nd and 4th answers are correct E) all of the answers are correct
Show comment and answer
D) only 4th answer is correct
INT - 8.42
E) all of the answers are correct Morphological characteristics of Caroli
Show comment and answer syndrome:
INT - 8.39
What is the most probable diagnosis? 1) fusiform dilation of intrahepatic
bile ducts
The 78-year-old man is admitted to 2) crooked cystic duct
the hospital because of significant 3) bile duct stenosis
weight loss and jaundice. Abdominal 4) shortened common bile duct
US confirms a gallbladder without
stone and dilated intrahepatic bile 1st, 2nd and 3rd answers are
ducts, the extrahepatic bile ducts are A)
correct
not wider. B) 1st and 3rd answers are correct
C) 2nd and 4th answers are correct
A) acute cholecystitis
D) only 4th answer is correct
B) acute hepatitis
E) all of the answers are correct
C) primary biliary cirrhosis
Show comment and answer
D) bile duct tumour (Klatskin)
INT - 8.43
E) pancreas head carcinoma Criterion of early cholecystectomy in
Show comment and answer case of gallstones:
INT - 8.40
What is the treatment of choice? 1) jaundice
2) previous abdominal operation
The 78-year-old man is admitted to 3) cholangitis not responding to
the hospital because of significant antibiotics
weight loss and jaundice. Abdominal 4) within 24-72 hours after the
US confirms a gallbladder without formation of symptoms
stone and dilated intrahepatic bile
ducts, the extrahepatic bile ducts are 1st, 2nd and 3rd answers are
not wider. A)
correct
B) 1st and 3rd answers are correct
C) 2nd and 4th answers are correct Possible cause of gallbladder
D) only 4th answer is correct hypokinesia:
E) all of the answers are correct
1) diabetes
Show comment and answer 2) extended starvation
INT - 8.44
3) pregnancy
Criterion of the solution of gallstones 4) steroid treatment
with bile acids:
1st, 2nd and 3rd answers are
1) permeable cystic duct A)
correct
2) calcareous gallbladder
3) well contracting gallbladder B) 1st and 3rd answers are correct
4) age above 50 years C) 2nd and 4th answers are correct
D) only 4th answer is correct
1st, 2nd and 3rd answers are E) all of the answers are correct
A)
correct
Show comment and answer
B) 1st and 3rd answers are correct INT - 8.48
C) 2nd and 4th answers are correct Can be typical symptom of tumour of
D) only 4th answer is correct the ampulla of Vater:
E) all of the answers are correct
1) jaundice
Show comment and answer 2) fever
INT - 8.45
3) pruritus
Characteristics of gallbladder polyps: 4) diarrhoea
1) 95 percent are benign
1st, 2nd and 3rd answers are
2) can be detected with ultrasound A)
correct
3) adenomas larger than 10 mm are
precancerous B) 1st and 3rd answers are correct
4) inflamed polyps are common C) 2nd and 4th answers are correct
D) only 4th answer is correct
1st, 2nd and 3rd answers are E) all of the answers are correct
A)
correct
Show comment and answer
B) 1st and 3rd answers are correct INT - 8.49
C) 2nd and 4th answers are correct Possible symptom of
D) only 4th answer is correct postcholecystectomy syndrome:
E) all of the answers are correct
1) constipation
Show comment and answer 2) weight loss
INT - 8.46
3) nausea
Characteristics of Mirizzi’s syndrome: 4) diarrhoea
1) obstructive icterus
1st, 2nd and 3rd answers are
2) normal bilirubin level A)
correct
3) dilated hepatic duct
4) diarrhoea B) 1st and 3rd answers are correct
C) 2nd and 4th answers are correct
1st, 2nd and 3rd answers are D) only 4th answer is correct
A)
correct E) all of the answers are correct
B) 1st and 3rd answers are correct Show comment and answer
C) 2nd and 4th answers are correct INT - 8.50
D) only 4th answer is correct Possible cause of postcholecystectomy
syndrome:
E) all of the answers are correct
Show comment and answer 1) cystic duct remnant syndrome
INT - 8.47
2) bile duct stenosis
3) remaining bile duct stone 3) duodenal diverticulum
4) gallstone ileus 4) ileocecal valve

1st, 2nd and 3rd answers are 1st, 2nd and 3rd answers are
A) A)
correct correct
B) 1st and 3rd answers are correct B) 1st and 3rd answers are correct
C) 2nd and 4th answers are correct C) 2nd and 4th answers are correct
D) only 4th answer is correct D) only 4th answer is correct
E) all of the answers are correct E) all of the answers are correct
Show comment and answer Show comment and answer
INT - 8.51 INT - 8.54
Characteristics of cholelithiasis: Predisposing factors of acute
acalculous cholecystitis:
1) can be detected in 10 percent of
the population 1) prolonged starvation
2) more common in women thn in 2) immobility
men 3) hemodynamic instability
3) its incidence increases with age associated with severe abdominal
4) its prevalence does not change diseases
with age 4) nonalcoholic steatohepatitis

1st, 2nd and 3rd answers are 1st, 2nd and 3rd answers are
A) A)
correct correct
B) 1st and 3rd answers are correct B) 1st and 3rd answers are correct
C) 2nd and 4th answers are correct C) 2nd and 4th answers are correct
D) only 4th answer is correct D) only 4th answer is correct
E) all of the answers are correct E) all of the answers are correct
Show comment and answer Show comment and answer
INT - 8.52 INT - 8.55
Characetristics of juxtapapillary May be the complication of
diverticulum: choledocholithiasis:

1) can be cause of chronic liver 1) chronic cholangitis


disease 2) biliary pancreatitis
2) endoscopic papillotomy is 3) secondary biliary cirrhosis
contraindicated 4) liver cyst
3) is often associated with bile duct
malformations 1st, 2nd and 3rd answers are
A)
4) raises the risk of bile duct stones correct
B) 1st and 3rd answers are correct
1st, 2nd and 3rd answers are C) 2nd and 4th answers are correct
A)
correct
D) only 4th answer is correct
B) 1st and 3rd answers are correct
E) all of the answers are correct
C) 2nd and 4th answers are correct
Show comment and answer
D) only 4th answer is correct

E) all of the answers are correct NEPHROLOGY
Show comment and answer
INT - 8.53
INT - 10.1
The most common location of
The prevalence of diabetic
obstruction in gallstone ileus:
nephropathy in Type 1 diabetes
mellitus:
1) hepatic flexure
2) colonic diverticulum
A) below 5% The progression of diabetic
B) 20–30% nephropathy may be accelerated by:
C) 80–90%
A) euglycaemia
D) it develops in all patients
B) increased protein intake
Show comment and answer
INT - 10.2
C) reduced salt intake
In diabetic patients treated with D) antihypertensive therapy
metformin, metformin should not be Show comment and answer
administered prior to any scheduled INT - 10.7
examination using contrast agent if In diabetic patients treated with
eGFR <30 ml/min/1,73m2: metformin, metformin therapy can be
continued after the contrast agent
A) administration should be continued examination:
administration should be suspended
B) A) on the day of the examination
only on the day of the examination
administration should be suspended on the next day, irrespective of
C) B)
by two days prior to the examination renal function
administration should be suspended 48 hours after the examination, if
C)
D) by one week prior to the GFR is below 30 ml/min/1,73m2
examination 48 hours after the examination, if
D)
Show comment and answer GFR is above 30 ml/min/1,73m2
INT - 10.3 Show comment and answer
The most important factor in the early INT - 10.8
diagnosis of diabetic nephropathy: Patients with Type 1 diabetes have to
be screened for diabetic nephropathy:
A) the appearance of hypertension
B) the appearance of microalbuminuria A) upon establishing the diagnosis
the increase of serum creatinine approx. 5 years after the diagnosis
C) B)
level was established
D) the decrease of eGFR approx. 10-15 years after the
C)
diagnosis was established
Show comment and answer
INT - 10.4 approx. 20-30 years after the
D)
Diabetic nephropathyis reversible: diagnosis was established
Show comment and answer
A) even if eGFR is reduced INT - 10.9
Patients with Type 2 diabetes have to
B) in the stage of macroalbuminuria
be screened for diabetic nephropathy:
C) in the stage of microalbuminuria
even if the serum creatinine is A) upon establishing the diagnosis
D)
increased
approx. 5 years after the diagnosis
Show comment and answer B)
was established
INT - 10.5
approx. 10-15 years after the
It is fundamental in the treatment of C)
diagnosis was established
diabetic nephropathy to give:
approx. 20-30 years after the
D)
renin–angiotensin–aldosterone diagnosis was established
A) Show comment and answer
system (RAAS) inhibitors
INT - 10.10
B) alpha-blockers
Diabetic nephropathy is likely to have
C) high-dose thiazide diuretics developed in a diabetic patient with
D) direct vasodilator antihypertensives proteinuria
Show comment and answer
INT - 10.6
in the absence of diabetic is a glomerulonephritis caused by
A) C)
retinopathy NSAIDs
if the patient has diabetic is an acute kidney injury caused by
B) D)
retinopathy but no haematuria steroidal anti-inflammatory drugs
if the patient has diabetic Show comment and answer
C) INT - 10.15
retinopathy and haematuria
D) it is likely in all the above cases The most appropriate method for the
early detection of analgesic
Show comment and answer
nephropathy:
INT - 10.11
Metformin is contraindicated in diabetic
patients: A) abdominal ultrasound
B) CT-angiography
A) if the eGFR is below30 ml/min C) MR-angiography
B) if the patient is obese D) retrograde pyelography
C) if the patient has hypertension Show comment and answer
INT - 10.16
D) if the patient has thyroid disease
Risk factors for acute kidney injury
Show comment and answer
exclude:
INT - 10.12
Nephrotic syndrome is defined as:
A) old age
A) significant proteinuria B) intravenous contrast agents
B) hypalbuminaemia C) NSAIDs abuse
C) predisposition to oedema D) increased fluid intake
the concurrent presence of all three Show comment and answer
D) INT - 10.17
factors above
In acute kidney injury, renal cause
Show comment and answer
INT - 10.13
may be suspected in the case of:
In case of the presence of glomerular
type red blood cells in the urine isolated or marked increase of the
A)
sediment: serum carbamide nitrogen level
B) bladder retention
renal stone is suspected, the patient marked increase of the serum
A) should be referred first to urology creatinine level in conjunction with
C)
examination less marked increase of the serum
tumour is suspected, the patient carbamide nitrogen level
B) should be referred first to urology decreased turgor of the skin and dry
D)
examination tongue
glomerular disease is suspected, Show comment and answer
C) the patient should be referred first INT - 10.18
to nephrology examination In the case of acute kidney injury and
uroinfection is suspected, the glomerular haematuria or the presence
D) patient should be referred first to of RBC cylinders in the urine:
urology examination
Show comment and answer pre-renal causes should be
A)
INT - 10.14 searched
Analgesic nephropathy: renal cause, e.g. acute
B)
glomerulonephritis is suspected
is an acute kidney injury caused by post-renal cause should be
A) C)
NSAIDs searched
is a chronic kidney disease caused Show comment and answer
B)
by NSAIDs INT - 10.19
The most common cause of acute B) answers 1. and 3. are correct
kidney injury out of the following: C) answers 2. and 4. are correct
D) only answer 4. is correct
A) pre-renal causes
E) all 4 answers arecorrect
B) renal causes
Show comment and answer
C) post-renal causes INT - 10.23
Show comment and answer Causes of symmetrical leg oedema
INT - 10.20 may be:
The most common causes of primary
nephrotic syndrome: 1) marked proteinuria e.g. in
nephrotic syndrome
1) membranous glomerulonephritis 2) malnutrition, malabsorption
2) minimal change disease 3) liver failure
3) focal segmental 4) peripheral artery disease
glomerulosclerosis
4) IgA nephropathy A) answers 1., 2. and 3. are correct
B) answers 1. and 3. are correct
A) answers 1., 2. and 3. are correct
C) answers 2. and 4. are correct
B) answers 1. and 3. are correct
D) only answer 4. is correct
C) answers 2. and 4. are correct
E) all 4 answers arecorrect
D) only answer 4. is correct
Show comment and answer
E) all 4 answers arecorrect INT - 10.24
Show comment and answer A typical sign of systemic, vasculitis
INT - 10.21 involving the kidneys may be:
In the case of glomerular type
haematuria after respiratory infection 1) acute kidney injury often
the following pathology/pathologies accompanied by haematuria
may be suspected: 2) urinary obstruction
3) appearance of purpura
1) minimal change disease 4) concurrent presence of diabetic
2) IgA nephropathy retinopathy
3) membranous glomerulonephritis
4) acute, post-streptococcal A) answers 1., 2. and 3. are correct
glomerulonephritis
B) answers 1. and 3. are correct
C) answers 2. and 4. are correct
A) answers 1., 2. and 3. are correct
D) only answer 4. is correct
B) answers 1. and 3. are correct
E) all 4 answers arecorrect
C) answers 2. and 4. are correct
Show comment and answer
D) only answer 4. is correct
INT - 10.25
E) all 4 answers arecorrect The most common cause of secondary
Show comment and answer hypertension:
INT - 10.22
If RAAS-inhibitor therapy was 1) Cushing syndrome
commenced for treating diabetic 2) acromegaly
nephropathy, after the initiation, the 3) hyperthyreoidism
following should be controlled: 4) hypertension of renal origin

1) serumcreatinine and eGFR A) answers 1., 2. and 3. are correct


2) blood lipids
B) answers 1. and 3. are correct
3) serum potassium level
4) RBC sedimentation rate C) answers 2. and 4. are correct
D) only answer 4. is correct
A) answers 1., 2. and 3. are correct E) all 4 answers arecorrect
Show comment and answer B) answers 1. and 3. are correct
INT - 10.26
C) answers 2. and 4. are correct
Choose the correct answers from the
following statements characterising D) only answer 4. is correct
acute post-streptococcal E) all 4 answers arecorrect
glomerulonephritis! Show comment and answer
INT - 10.29
1) the serum complement level The following renal-replacement
decreases therapy modalities may be chosen in
2) the serum ASO titer decreases end-stage renal failure:
3) oedema around the eyes can be
observed 1) haemodialysis
4) hypertension is never present 2) peritoneal dialysis
3) kidney transplantation
A) answers 1., 2. and 3. are correct 4) cyclophosphamide therapy
B) answers 1. and 3. are correct
A) answers 1., 2. and 3. are correct
C) answers 2. and 4. are correct
B) answers 1. and 3. are correct
D) only answer 4. is correct
C) answers 2. and 4. are correct
E) all 4 answers arecorrect
D) only answer 4. is correct
Show comment and answer
INT - 10.27 E) all 4 answers arecorrect
Choose the correct answers from the Show comment and answer
following statements suggesting INT - 10.30
causes of anaemia as a result ofchronic Causes of systemic leg oedema may
renal failure! be:

1) decreased erytropoietin 1) heart failure


production 2) decompensated liver cirrhosis
2) decreased iron and 3) nephrotic syndrome
B12 absorption 4) none of the above
3) uraemic toxins depressing the
bone marrow A) answers 1., 2. and 3. are correct
4) increased life-span of RBC can be B) answers 1. and 3. are correct
observed C) answers 2. and 4. are correct
D) only answer 4. is correct
A) answers 1., 2. and 3. are correct
E) all 4 answers arecorrect
B) answers 1. and 3. are correct
Show comment and answer
C) answers 2. and 4. are correct INT - 10.31
D) only answer 4. is correct Which of the following may indicate
E) all 4 answers arecorrect renal involvement is SLE?
Show comment and answer
INT - 10.28 1) skin lesions
Out of the following, which ultrasound 2) bacteriuria
result indicates chronic kidney disease 3) neurological signs
in an average-sized adult? 4) proteinuria

1) the length of kidneys is 80 mm A) answers 1., 2. and 3. are correct


2) hyper-reflective parenchyma B) answers 1. and 3. are correct
3) wave-like appearance of the C) answers 2. and 4. are correct
surface
D) only answer 4. is correct
4) 12–14 mm thick parenchyma
E) all 4 answers arecorrect
A) answers 1., 2. and 3. are correct Show comment and answer
INT - 10.32
Which of the following factors may be What characterises acute
responsible for the development of tubulointerstitial nephritis from among
diabetic nephropathy? the following?

1) glycation end products 1) fever


2) hyperfiltration 2) eosinophilia/eosinophiluria
3) genetic factors 3) sterile pyuria
4) decreased negative charge of the 4) hypertension
glomerular basal membrane
A) answers 1., 2. and 3. are correct
A) answers 1., 2. and 3. are correct B) answers 1. and 3. are correct
B) answers 1. and 3. are correct C) answers 2. and 4. are correct
C) answers 2. and 4. are correct D) only answer 4. is correct
D) only answer 4. is correct E) all 4 answers arecorrect
E) all 4 answers arecorrect Show comment and answer
Show comment and answer INT - 10.36
INT - 10.33 What characterises renovascular
When does bacteriuria require hypertension?
treatment in young female patients?
1) bruits are always noticeable with
1) in the case of complaints physical examination
suggestive of a urinary tract infection 2) angiography is always needed as
2) in the case of leukocyturia the gold standard
3) in pregnancy 3) it is the most common form of
4) if haematuria is also present secondary hypertension
4) it most frequently develops on
A) answers 1., 2. and 3. are correct the grounds of atherosclerosis
B) answers 1. and 3. are correct
C) answers 2. and 4. are correct A) answers 1., 2. and 3. are correct
D) only answer 4. is correct B) answers 1. and 3. are correct
E) all 4 answers arecorrect C) answers 2. and 4. are correct
Show comment and answer D) only answer 4. is correct
INT - 10.34 E) all 4 answers arecorrect
Which of the following can be used for Show comment and answer
measuring the glomerular filtration INT - 10.37
rate? Which of the following results indicate
good prognosis in acute renal injury?
1) 24-hour endogenous creatinine
clearance 1) polyuria
2) clearance measured with an 2) anuria
isotope 3) urine specific gravity: 1005
3) estimated GFR 4) urine specific gravity: 1012
4) concentrating ability test
A) answers 1., 2. and 3. are correct
A) answers 1., 2. and 3. are correct B) answers 1. and 3. are correct
B) answers 1. and 3. are correct C) answers 2. and 4. are correct
C) answers 2. and 4. are correct D) only answer 4. is correct
D) only answer 4. is correct E) all 4 answers arecorrect
E) all 4 answers arecorrect Show comment and answer
Show comment and answer
INT - 10.35
INT - 10.38 1) insulin + glucose infusion
Which of the following factors can 2) iv. calcium
attenuate the progression of chronic 3) furosemide and physiological
kidney disease? saline infusion
4) ion-exchange resin
1) RAAS inhibitor therapy
2) dietary protein restriction A) answers 1., 2. and 3. are correct
3) regular blood pressure control
B) answers 1. and 3. are correct
4) statin therapy
C) answers 2. and 4. are correct
A) answers 1., 2. and 3. are correct D) only answer 4. is correct
B) answers 1. and 3. are correct E) all 4 answers arecorrect
C) answers 2. and 4. are correct Show comment and answer
INT - 10.42
D) only answer 4. is correct
It is important to know the patient’s
E) all 4 answers arecorrect kidney function when ordering a new
Show comment and answer medication, as the pharmacokinetics of
INT - 10.39 several medications and thereby, the
Which of the following needs to be dosage depends on the kidney
addressed in the conservative therapy function.
of chronic renal failure?
both are correct, there is a causal
1) anaemia A)
relationship between the two,
2) metabolic acidosis
both are correct, there is no causal
3) hyperkalaemia B)
relationship between the two,
4) hyponatraemia
the first is correct, the second is
C)
A) answers 1., 2. and 3. are correct incorrect
B) answers 1. and 3. are correct the first is incorrect , the second is
D)
correct
C) answers 2. and 4. are correct
E) both are incorrect
D) only answer 4. is correct
Show comment and answer
E) all 4 answers arecorrect INT - 10.43
Show comment and answer Haematuria is the most common
INT - 10.40 symptom in minimal change
Which antidiabetic can be given to nephropathy, therefore the patient has
patients with declined kidney function to be protected from physical
(eGFR<30 ml/min/1,73m2), including exercise/burden.
patients receiving dialysis therapy?
both are correct, there is a causal
1) gliquidone A)
relationship between the two,
2) metformin
both are correct, there is no causal
3) insulin B)
relationship between the two,
4) glimepiride
the first is correct, the second is
C)
incorrect
A) answers 1., 2. and 3. are correct
the first is incorrect , the second is
B) answers 1. and 3. are correct D)
correct
C) answers 2. and 4. are correct
E) both are incorrect
D) only answer 4. is correct
Show comment and answer
E) all 4 answers arecorrect INT - 10.44
INT - 10.41 In the case of non-dialysed patients
Which of the following can be used in with declined kidney function thiazide-
the conservative treatment of type diuretics should be given for
hyperkalaemia?
diuretic therapy, becauseat this stage to the urinary flow have to be looked
loop-diuretics are ineffective. for.

both are correct, there is a causal both are correct, there is a causal
A) A)
relationship between the two, relationship between the two,
both are correct, there is no causal both are correct, there is no causal
B) B)
relationship between the two, relationship between the two,
the first is correct, the second is the first is correct, the second is
C) C)
incorrect incorrect
the first is incorrect , the second is the first is incorrect , the second is
D) D)
correct correct
E) both are incorrect E) both are incorrect
Show comment and answer Show comment and answer
INT - 10.45 INT - 10.48
In patients over 50 suffering from In acute kidney injury, pre- and post-
membranous glomerulonephritis renal causes have to be excluded, as
cancer screening is necessary, since in acute kidney injury may develop in
older age tumour antigen-containing elderly patients with the use of
immune complexes more frequently NSAIDs.
cause membranous
glomerulonephritis. both are correct, there is a causal
A)
relationship between the two,
both are correct, there is a causal both are correct, there is no causal
A) B)
relationship between the two, relationship between the two,
both are correct, there is no causal the first is correct, the second is
B) C)
relationship between the two, incorrect
the first is correct, the second is the first is incorrect , the second is
C) D)
incorrect correct
the first is incorrect , the second is E) both are incorrect
D)
correct Show comment and answer
E) both are incorrect INT - 10.49
Show comment and answer In Type 1 diabetes mellitus, initial
INT - 10.46 symptom of the renal involvement is
Acute diffuse poststreptococcal the increase of glomerular filtration
glomerulonephritis is characterised by rate (GFR), which decreases to its
fever and inflammatory symptoms, original value as glucose metabolism
therefore the patient has to be treated normalizes in the majority of patients.
with antipyretics.
both are correct, there is a causal
A)
both are correct, there is a causal relationship between the two,
A)
relationship between the two, both are correct, there is no causal
B)
both are correct, there is no causal relationship between the two,
B)
relationship between the two, the first is correct, the second is
C)
the first is correct, the second is incorrect
C)
incorrect the first is incorrect , the second is
D)
the first is incorrect , the second is correct
D)
correct E) both are incorrect
E) both are incorrect Show comment and answer
Show comment and answer INT - 10.50
INT - 10.47 In the case of diabetic kidney disease,
The main symptom of patients with RAAS-inhibitor therapy is given due to
nephrotic syndrome is pyuria, abnormal albuminuria, as RAAS-
therefore, changes causing obstruction
inhibitors decrease the intraglomerular
pressure. both are correct, there is a causal
A)
relationship between the two,
both are correct, there is a causal both are correct, there is no causal
A) B)
relationship between the two, relationship between the two,
both are correct, there is no causal the first is correct, the second is
B) C)
relationship between the two, incorrect
the first is correct, the second is the first is incorrect , the second is
C) D)
incorrect correct
the first is incorrect , the second is E) both are incorrect
D)
correct Show comment and answer
E) both are incorrect INT - 10.54
Show comment and answer Abnormal albuminuria is inadequate
INT - 10.51 for screening the diabetic and
Stages of the renal alterations are hypertonic renal damage, because the
similar in Type1 and Type 2 diabetes range of abnormal albuminuria is >30
mellitus, as both types of diabetes mg/day.
mellitus develop as the result of
severely damaged insulin production of both are correct, there is a causal
A)
the pancreas. relationship between the two,
both are correct, there is no causal
both are correct, there is a causal B)
A) relationship between the two,
relationship between the two, the first is correct, the second is
both are correct, there is no causal C)
B) incorrect
relationship between the two, the first is incorrect , the second is
the first is correct, the second is D)
C) correct
incorrect E) both are incorrect
the first is incorrect , the second is Show comment and answer
D)
correct Association question
E) both are incorrect Link the symptoms indicated by
Show comment and answer numbers with the matching
INT - 10.52 pathological clinical pictures (with
Nephropathy may be present at the capital letters)!
time of the diagnosis in Type 2
diabetes, as diabetic nephropathy INT - purpura -
belongs to the so-called 10.55
macroangiopathies. INT - acute kidney injury -
10.56
both are correct, there is a causal INT - glomerular type -
A)
relationship between the two, 10.57 haematuria
both are correct, there is no causal INT - nephrotic syndrome -
B) 10.58 without the presence of a
relationship between the two,
the first is correct, the second is diabetic retinopathy
C)
incorrect
A) minimal change disease
the first is incorrect , the second is
D) B) vasculitis
correct
C) urinary obstruction
E) both are incorrect
D) diabetic nephropathy
Show comment and answer
INT - 10.53
In general practice, CKD-epi equation
is used for estimating the kidney
function, as its determination requires
the collection of 24-hour urine sample.
Association question C) both of the above
Link the symptoms indicated by D) none of the above
numbers with the most matching Association question
pathological clinical pictures (with Match the therapeutic possibilities
capital letters)! marked by numbers with the clinical
picture marked by capital letters in the
INT - proteinuria above 3.5 - treatment of which the given
10.59 g/day therapeutic possibility is the most
INT - glomerular type - appropriate treatment of choice.
10.60 haematuria
INT - pyuria + bacteriuria - INT - management of -
10.61 10.67 homeostatic/volume
INT - erythropoietin therapy - balance
10.62 INT - potassium-sparing diuretic -
10.68
A) renal anaemia INT - erythropoietin -
B) nephrotic syndrome 10.69
C) nephritic syndrome INT - dialysis -
D) uroinfection 10.70
Association question
Link the symptoms indicated by A) renal anaemia
numbers with the most matching B) pre-renal acute kidney injury
pathological clinical pictures (with C) severe acute kidney injury
capital letters)! D) nephrotic syndrome
Association question
INT - proteinuria above 3.5 - Match conditions marked by numbers
10.59 g/day with the most appropriate concept!
INT - glomerular type -
10.60 haematuria INT - screening of albuminuria -
INT - pyuria + bacteriuria - 10.71
10.61 INT - acute post-streptococcal -
INT - erythropoietin therapy - 10.72 glomerulonephritis
10.62 INT - decrease in GFR -
10.73
A) renal anaemia INT - decreased proteinuria -
B) nephrotic syndrome 10.74
C) nephritic syndrome
D) uroinfection A) cardiovascular progression
Association question B) cardiovascular regression
Link the symptoms indicated by C) early screening for cardiovascular
numbers with the most matching disease
pathological clinical pictures (with D) no increase in cardiovascular risk
capital letters)!
ENDOCRINOLOGY
INT - low glomerular filtration -

10.63 rate
INT - 11.1
INT - kidneys are of normal size -
Main manifestations of multiple
10.64 or enlarged
endocrine neoplasia type 1 (MEN1):
INT - it develops unnoticeably, -
10.65 almost symptomless
pheochromocytoma,
INT - the kidneys’ concentrating -
A) hyperparathyroidism, Leydig-cell
10.66 ability is not disturbed testicle tumor
A) acute kidney injury
B) chronic renal insufficiency
pheochromocytoma, D) treatment with dopamin antagonist
B) hyperparathyroidism, pituitary treatment with somatostatin
adenoma E)
analogue
hyperparathyroidism, pancreas Show comment and answer
C) neuroendocrine tumor, pituitary INT - 11.5
adenoma In case of a patient receiving thyroid
pheochromocytoma, medullary hormone substitution due to a thyroid
D) thyroid carcinoma, pituitary ablation therapy of papillary thyroid
adenoma carcinoma, laboratory results show a
hyperparathyroidism, suppression of TSH level and normal
E) pheochromocytoma, pituitary free T4 and free T3 levels. Is it
adenoma necessary to modify the dose of the
Show comment and answer thyroid hormone substitution (l-
INT - 11.2 thyroxin)?
The appropriate test to certify
acromegaly: the dose of l-thyroxin is appropriate,
A)
no modification is needed
investigation of the diurnal rhythm reduction of the daily dose of l-
A) of serum growth hormone (GH) B) thyroxin is necessary due to the
concentration suppressed TSH
investigation of the serum growth increase of the daily dose of l-
B) hormone (GH) concentration in C) thyroxin is necessary due to the
morning’ hours suppressed TSH
investigation of the serum growth no modification of the dose of l-
C) hormone (GH) concentration during D) thyroxin is needed, but we shall
oral glucose tolerance test give thyreostatics in addition
investigation of the serum growth Show comment and answer
hormone (GH) concentration after INT - 11.6
D)
administration of growth hormone- We can use it as a serum marker of
releasing hormone (GHRH) medullary thyroid carcinoma:
Show comment and answer
INT - 11.3 investigation of serum
It is characteristic for subclinical A)
thyreoglobulin
primary hypothyroidism: B) investigation of serum calcitonin
C) investigation of serum osteocalcin
clinical signs of hypothyroidism with
A) normal serum thyroid hormone investigation of serum parathyroid
D)
levels hormone
the hormone levels refer to primary Show comment and answer
INT - 11.7
B) hypothyroidism without clinical
signs Endocrine disorders causing diarrhea,
except:
low free T4 and free T3 levels with
C)
normal serum TSH
A) medullary thyroid carcinoma
normal free T4 and free T3 levels
D) B) gastrinoma
with elevated serum TSH
C) carcinoid tumor
Show comment and answer
INT - 11.4 D) pheochromocytoma
Primary therapy of prolactin-producing Show comment and answer
pituitary microadenoma: INT - 11.8
The appropriate test for the
A) pituitary surgery clarification of Cushing’s syndrome,
B) pituitary irradiation except:
C) treatment with dopamin agonist
investigation of 24 hour urinary free there is no connection between the
A) D) pituitary macroadenoma and the
cortisol (UFC) excretion
investigation of serum cortisol after double vision
B) administration of low dose Show comment and answer
dexamethasone INT - 11.12
investigation of the diurnal rhythm It is characteristic for the lack of
C) growth hormone (GH) in adults,
of serum cortisol
except:
investigation of serum cortisol in
D)
morning hours
it is associated with unfavorable
investigation of the diurnal rhythm A)
E) lipid profile
of salivary cortisol
it is associated with increased water
Show comment and answer B) content of the body and water
INT - 11.9
retention
Radioiodine treatment can be used
against the following diseases, except: it is associated with unfavorable
C)
fat/muscle mass ratio
after the surgery of papillary thyroid it is associated with weakness and
A) D)
carcinoma discomfort
after the surgery of follicular thyroid Show comment and answer
B) INT - 11.13
carcinoma
Diseases with polyuria, except:
after the surgery of medullary
C)
thyroid carcinoma
A) primary aldosteronism
autonomous thyroid nodule causing
D) B) primary hyperparathyroidism
hyperthyroidism
C) diabetes insipidus
E) Graves’ disease
D) diabetes mellitus
Show comment and answer
INT - 11.10 SIADH (syndrome of inappropriate
E)
It is characteristic for primary antidiuretic hormone secretion)
aldosteronism: Show comment and answer
INT - 11.14
suppression of plasma renin and It can be the cause of growth hormone
A) (GH) deficiency in adults:
high plasma aldosteron
high plasma renin and high plasma
B) A) pituitary tumor
aldosteron
normal plasma renin and high B) skull trauma
C)
plasma aldosteron C) radiotherapy
high plasma aldosteron, plasma D) pituitary apoplexia
D)
renin is not relevant E) all of the above
Show comment and answer Show comment and answer
INT - 11.11 INT - 11.15
In case of a patient with pituitary Diseases with hirsutism, except:
macroadenoma, double vision evolved.
What can be the reason for that? A) Cushing’s disease
B) hypopituitarism
suprasellar spread of the pituitary
A) C) polycystic ovary syndrome
macroadenoma
lateral spread of the pituitary 21-hydroxylase deficiency form of
D)
B) macroadenoma to the cavernous congenital adrenal hyperplasia
sinus Show comment and answer
INT - 11.16
downward spread of the pituitary
C) macroadenoma and penetration Hereditary tumor syndromes with
into the sphenoid sinus pheochromocytoma:
1) von Hippel–Lindau syndrome 3) hCG (human chorionic
2) multiple endocrine neoplasia type gonadotropin)
2 4) FSH (follicle-stimulating
3) neurofibromatosis type 1 hormone)
4) multiple endocrine neoplasia type
1 A) answers 1., 2., and 3 are correct
B) answers 1. and 3. are correct
A) answers 1., 2., and 3 are correct C) answers 2. and 4. are correct
B) answers 1. and 3. are correct D) only answer 4 is correct
C) answers 2. and 4. are correct E) all four answers are correct
D) only answer 4 is correct Show comment and answer
E) all four answers are correct INT - 11.20
Show comment and answer Which tumors are associated with
INT - 11.17 elevated serum chromogranin A
Which are tumor suppressor genes of concentration?
the followings?
1) carcinoid tumor
1) RET 2) gastrinoma
2) MEN1 (menin protein encoding 3) pheochromocytoma
gene) 4) insulinoma
3) RAS
4) VHL A) answers 1., 2., and 3 are correct
B) answers 1. and 3. are correct
A) answers 1., 2., and 3 are correct C) answers 2. and 4. are correct
B) answers 1. and 3. are correct D) only answer 4 is correct
C) answers 2. and 4. are correct E) all four answers are correct
D) only answer 4 is correct Show comment and answer
E) all four answers are correct INT - 11.21
Show comment and answer Applicable treatment in thyrotoxic
INT - 11.18 crisis:
It may be a symptom of Graves’
disease: 1) thyrostatics
2) iodine preparation
1) diarrhea 3) beta-adrenergic receptor blocker
2) tachycardia 4) plasmapheresis
3) pretibial myxedema
4) fine hand tremor A) answers 1., 2., and 3 are correct
B) answers 1. and 3. are correct
A) answers 1., 2., and 3 are correct C) answers 2. and 4. are correct
B) answers 1. and 3. are correct D) only answer 4 is correct
C) answers 2. and 4. are correct E) all four answers are correct
D) only answer 4 is correct Show comment and answer
E) all four answers are correct INT - 11.22
Show comment and answer Typical in subacute granulomatous
INT - 11.19 thyroiditis (de Quervain’s thyroiditis):
Which are glicoprotein hormones of the
followings? 1) it is associated with the painful
swelling of the thyroid gland and fever
1) TSH (thyroid-stimulating 2) it is common cause of permanent
hormone) (definitive) primary hypothyroidism
2) LH (luteinizing hormone) 3) ESR is elevated
4) it often recurs after healing
A) answers 1., 2., and 3 are correct
1) hypernatremia, that leads to
B) answers 1. and 3. are correct
severe neurological symptoms
C) answers 2. and 4. are correct 2) antidiuretic hormone (ADH,
D) only answer 4 is correct vasopressin) overdosage can be one of
E) all four answers are correct its causes
Show comment and answer 3) its treatment can be water
INT - 11.23 administration through a nasogastric
In case of differentiated thyroid cancer tube
radioiodine ablation 4) its long-term prognosis is usually
good, because other regulating
1) is applicable after total (near systems may compensate the role of
total) thyroidectomy the thirst center
2) is applicable instead of total
(near total) thyroidectomy A) answers 1., 2., and 3 are correct
3) the condition of its application is B) answers 1. and 3. are correct
high serum TSH level (that can be
C) answers 2. and 4. are correct
achieved by the discontinuation of
thyroid supplementation for a certain D) only answer 4 is correct
time in patients receiving thyroid E) all four answers are correct
supplementation, or the administration Show comment and answer
of TSH product) INT - 11.26
4) is applicable before total (near It is common in acromegaly:
total) thyroidectomy
1) diabetes mellitus
A) answers 1., 2., and 3 are correct 2) hypertension
3) sleep apnea
B) answers 1. and 3. are correct
4) osteoarthrosis
C) answers 2. and 4. are correct
D) only answer 4 is correct A) answers 1., 2., and 3 are correct
E) all four answers are correct B) answers 1. and 3. are correct
Show comment and answer C) answers 2. and 4. are correct
INT - 11.24
D) only answer 4 is correct
Which diseases are associated with
hyponatremia? E) all four answers are correct
Show comment and answer
1) SIADH (syndrome of INT - 11.27
inappropriate antidiuretic hormone Which form of congenital adrenal
secretion) hyperplasia is associated with
2) diabetes insipidus hypertension and hypokalemia?
3) Addison’s disease
4) primary aldosteronism 1) steroid 21-hydroxylase deficiency
2) 11β-hydroxylase deficiency
A) answers 1., 2., and 3 are correct 3) defect of the gene encoding
STAR protein (steroidogenesis acute
B) answers 1. and 3. are correct
regulatory protein)
C) answers 2. and 4. are correct 4) 17α-hydroxylase/17, 20-lyase
D) only answer 4 is correct enzyme deficiency
E) all four answers are correct
Show comment and answer A) answers 1., 2., and 3 are correct
INT - 11.25 B) answers 1. and 3. are correct
The injury of the thirst center of the C) answers 2. and 4. are correct
hypothalamus may cause primary
D) only answer 4 is correct
adipsia. What are the characteristics of
this? E) all four answers are correct
Show comment and answer Association question
INT - 11.28 Connect the various symptoms
It is typical in Turner syndrome: (indicated with numbers) with the
disease (indicated with letters) that it
1) the syndrome is caused by the is most associated with!
total absence of an X chromosome,
thus the karyotype of these patients is INT - 11.30 polyuria -
always 45,X INT - 11.31 scant body hair -
2) apart from the typical features INT - 11.31 scant body hair -
(short stature, pterygium colli, low INT - 11.32 tall stature -
posterior hairline, low-set ears, short INT - 11.33 round, “moon” face -
metatarsus, cubitus valgus) cardial
and renal malformations are often A) diabetes insipidus
associated B) Cushing’s disease
3) the cause of the typical primary C) Klinefelter syndrome
amenorrhea is the decreased LH and D) panhypopituitarism
FSH production of the pituitary Association question
4) in childhood for the treatment of Connect the various symptoms
short stature growth hormone (GH) (indicated with numbers) with the
products are applicable disease (indicated with letters) that it
is most associated with!
A) answers 1., 2., and 3 are correct
B) answers 1. and 3. are correct INT - nephrolithiasis -

C) answers 2. and 4. are correct 11.34


INT - nephrolithiasis -
D) only answer 4 is correct
11.34
E) all four answers are correct INT - osteoporosis -
Show comment and answer 11.35
INT - 11.29 INT - osteoporosis -
It is typical in Klinefelter syndrome: 11.35
INT - galactorrhea and -
1) the syndrome is caused by an 11.36 amenorrhea
extra X chromosome, therefore the INT - flush episodes -
karyotype of these patients is always 11.37
47,XXY
2) apart from the typical external A) prolactin-producing pituitary
features (tall stature, signs of adenoma
hypogonadism, gynecomastia, obesity, B) carcinoid tumor
small testicles) the elevated risk of C) primary hyperparathyroidism
diabetes mellitus and cardiovascular D) multiple endocrine neoplasia type 1
disease is also common (MEN1 syndrome)
3) the cause of the hypogonadism Association question
typical for the disease is the decreased Connect the various symptoms
LH and FSH production of the pituitary (indicated with numbers) with the
4) hypogonadism associated with disease (indicated with capital letters)
the disease can be treated with that it is most associated with!
testosterone products
INT - 11.38 hypogonadism -
A) answers 1., 2., and 3 are correct INT - 11.38 hypogonadism -
B) answers 1. and 3. are correct INT - 11.39 pericardial effusion -
C) answers 2. and 4. are correct INT - 11.40 anosmia -

D) only answer 4 is correct INT - 11.41 tetany -

E) all four answers are correct


A) Kallmann syndrome
Show comment and answer B) Klinefelter syndrome
C) severe untreated primary C) acromegaly
hypothyroidism D) Addison’s disease
D) severe untreated Association question
hypoparathyroidism Connect the various diseases
Association question (indicated with letters) with the most
Connect the various symptoms appropriate treatment (labelled with
(indicated with numbers) with the numbers)!
disease (indicated with capital letters)
that it is most associated with! INT - alpha-adrenergic -
11.50 receptor blocker
INT - 11.38 hypogonadism - INT - glucocorticoid -
INT - 11.38 hypogonadism - 11.51
INT - 11.39 pericardial effusion - INT - l-thyroxin -
INT - 11.40 anosmia - 11.52
INT - 11.41 tetany - INT - propyl-thiouracyl -
11.53
A) Kallmann syndrome
B) Klinefelter syndrome A) pheochromocytoma
C) severe untreated primary B) congenital adrenal hyperplasia
hypothyroidism C) panhypopituitarism
D) severe untreated D) hyperthyreoidism
hypoparathyroidism Association question
Association question Connect the diseases (indicated with
Connect the various diseases letters) with the most appropriate
(indicated with letters) with the most diagnostic serum- and urine
appropriate treatment (labelled with parameters (indicated with numbers)!
numbers)!
INT - TRAK (TSH receptor -
INT - bromocriptine - 11.54 antibody)
11.42 INT - parathyroid hormone- -
INT - spironolactone - 11.55 related protein (PTH-RP)
11.43 INT - anti-TPO (thyroid -
INT - octreotide (somatostatin- - 11.56 peroxidase antibody)
11.44 analogue) INT - 24-hour urine 5- -
INT - desmopressin - 11.57 Hydroxyindoleacetic acid
11.45 (5-HIAA)

A) secondary hyperaldosteronism A) Hashimoto’s thyroiditis


B) carcinoid tumor B) carcinoid tumor
C) prolactin producin pituitary C) Graves’ disease
adenome D) hypercalcaemia associated with
D) central diabetes insipidus malignant diseases
Association question Association question
Connect the various diseases Connect the diseases (indicated with
(indicated with letters) with the most numbers) with the most appropriate
appropriate treatment (labelled with diagnostic methods (indicated with
numbers)! letters)!

INT - 11.46 proton pump inhibitor - ) INT - pheochromocytoma - )


INT - 11.47 mitotane - ) 11.58
INT - 11.48 fludrocortisone - ) INT - Cushing’s disease - )
INT - 11.49 pegvisomant - ) 11.59
INT - aldosterone-producing - )
A) adrenocortical carcinome 11.60 adenoma
B) gastrinoma
INT - insulinoma - ) INT - 11.66
11.61 It is not advised to give beta-
adrenergic receptor blocker without
A) selective adrenal vein sampling for alpha-adrenergic receptor blocker to
hormone determination patients suffering from
B) inferior petrosal sinus sampling for pheochromocytoma, because the beta-
hormone determination adrenergic receptor blocker given
C) somatostatin receptor scintigraphy alone may provoke a
(octreoscan) pheochromocytoma crisis through the
D) metaiodobenzylguanidine (MIBG) inhibition of the presynaptic beta2-
scintigraphy adrenergic receptor.
Association question
Connect the diseases (indicated with both are correct, with cause-effect
A)
numbers) with the most appropriate relationship
diagnostic methods (indicated with both are correct, without cause-
letters)! B)
effect relationship
the first clause is correct, the
INT - pheochromocytoma - C)
second clause is incorrect
11.58
the first clause is incorrect, the
INT - Cushing’s disease - D)
second clause is correct
11.59
E) both are incorrect
INT - aldosterone-producing -
11.60 adenoma Show comment and answer
INT - 11.67
INT - insulinoma -
Non-pituitary tumors of the sellar
11.61
region (e.g. craniopharyngioma) may
cause the compression of the
A) selective adrenal vein sampling for
hypothalamus and/or the pituitary
hormone determination
stalk, therefore in these cases prolactin
B) inferior petrosal sinus sampling for
secretion and serum-prolactin levels
hormone determination
decrease due to the decreased
C) somatostatin receptor scintigraphy
production of hypothalamic trophic
(octreoscan)
hormones and/or the inhibited
D) metaiodobenzylguanidine (MIBG)
transport of trophic hormones through
scintigraphy
Association question the pituitary stalk.
Connect the various diseases
(indicated with letters) with the gene both are correct, with cause-effect
A)
(indicated with numbers) that’s relationship
determination is most helpful in the both are correct, without cause-
B)
diagnosis! effect relationship
the first clause is correct, the
C)
INT - RET (rearranged during - second clause is incorrect
11.62 transfection gene) the first clause is incorrect, the
INT - MEN1 (menin protein - D)
second clause is correct
11.63 coding gene) E) both are incorrect
INT - VHL (VHL protein coding -
Show comment and answer
11.64 gene) INT - 11.68
INT - PRKAR1A (protein kinase -
In contrast to Addison’s disease,
11.65 type I regulatory subunit secondary adrenal insufficiency
alpha coding gene) presenting as part of
panhypopituitarism is rarely treated
A) Carney complex with mineralocorticoids, because in
B) multiple endocrine neoplasia type 2 secondary adrenal insufficiency
C) multiple endocrine neoplasia type 1 substantially higher doses of
D) von Hippel–Lindau syndrome
glucocorticoid substitution are used E) both are incorrect
than for Addison’s disease. Show comment and answer
INT - 11.71
both are correct, with cause-effect Hyperprolactinemia causes
A)
relationship hypogonadism, because the increased
both are correct, without cause- level of serum prolactin inhibits the
B)
effect relationship secretion of hypothalamic
the first clause is correct, the gonadotropin-releasing hormone
C) (GnRH) and pituitary gonadotropins.
second clause is incorrect
the first clause is incorrect, the
D) both are correct, with cause-effect
second clause is correct A)
relationship
E) both are incorrect
both are correct, without cause-
Show comment and answer B)
INT - 11.69
effect relationship
The biologically active 1α,25- the first clause is correct, the
C)
dihydroxyvitamin-D (calcitriol) is second clause is incorrect
produced in the proximal tubule of the the first clause is incorrect, the
D)
kidney, therefore by patients suffering second clause is correct
from primary hypoparathyroidism with E) both are incorrect
normal renal function, hypocalcemia Show comment and answer
can be treated with vitamin D, and INT - 11.72
there is no indication of the For patients taking chronic adrenal
administration of activated vitamin D hormone replacement therapy for
(calcitriol). Addison’s disease, the daily
glucocorticoid dose has to be increased
both are correct, with cause-effect temporarily in case of an infection with
A)
relationship high fever, because infectious diseases
both are correct, without cause- with fever raise the glucocorticoid
B)
effect relationship demand.
the first clause is correct, the
C) both are correct, with cause-effect
second clause is incorrect A)
the first clause is incorrect, the relationship
D) both are correct, without cause-
second clause is correct B)
E) both are incorrect effect relationship
Show comment and answer the first clause is correct, the
C)
INT - 11.70 second clause is incorrect
The adult height of patients suffering D)
the first clause is incorrect, the
from the simple virilizing form of second clause is correct
congenital adrenal hyperplasia due to E) both are incorrect
21-hydroxylase deficiency is above Show comment and answer
average, because the permanently INT - 11.73
high level of testosterone during Aldosterone overproduction causes
childhood enhances bone development sodium retention, therefore
and maturation. hypertension is the concomitant of
diseases with secondary
A)
both are correct, with cause-effect hyperaldosteronism.
relationship
both are correct, without cause- both are correct, with cause-effect
B) A)
effect relationship relationship
the first clause is correct, the both are correct, without cause-
C) B)
second clause is incorrect effect relationship
the first clause is incorrect, the the first clause is correct, the
D) C)
second clause is correct second clause is incorrect
the first clause is incorrect, the both are correct, without cause-
D) B)
second clause is correct effect relationship
E) both are incorrect the first clause is correct, the
C)
Show comment and answer second clause is incorrect
INT - 11.74 the first clause is incorrect, the
D)
Growth hormone (GH) has no second clause is correct
physiological role after the age of E) both are incorrect
reaching adult height, therefore in Show comment and answer
adulthood the treatment of GH INT - 11.77
deficiency due to panhypopituitarism is In women with untreated primary
not justified. hypothyroidism, besides elevated
serum TSH level, high serum prolactin
both are correct, with cause-effect level and consecutive galactorrhea-
A)
relationship amenorrhea syndrome are also typical,
both are correct, without cause- because the enhanced thyrotropin-
B)
effect relationship releasing hormone (TRH) due to
the first clause is correct, the decreased thyroid function triggers the
C)
second clause is incorrect secretion of both TSH and prolactin.
the first clause is incorrect, the
D) both are correct, with cause-effect
second clause is correct A)
E) both are incorrect relationship
Show comment and answer both are correct, without cause-
B)
INT - 11.75 effect relationship
Regarding glucocorticoid efficacy, the first clause is correct, the
C)
prednisolone has a weaker effect than second clause is incorrect
hydrocortisone, therefore for the the first clause is incorrect, the
D)
chronic hormone replacement therapy second clause is correct
of primary and secondary adrenal E) both are incorrect
insufficiency the drug of choice is Show comment and answer
hydrocortisone. INT - 11.78
The treatment of secondary
both are correct, with cause-effect hyperaldosteronism due to bilateral
A)
relationship adrenal hyperplasia (idiopathic
both are correct, without cause- hyperaldosteronism) is not surgical
B)
effect relationship (bilateral adrenalectomy), but
the first clause is correct, the medicinal (spironolactone, potassium-
C)
second clause is incorrect sparing diuretics), because after
the first clause is incorrect, the bilateral adrenalectomy performed
D) because of secondary
second clause is correct
E) both are incorrect hyperaldosteronism Nelson syndrome
(aggressive ACTH-producing adenoma)
Show comment and answer
INT - 11.76 may evolve.
Sufficient testosterone replacement for
men suffering from panhypopituitarism both are correct, with cause-effect
A)
abolishes hypogonadism and relationship
oligospermia/aspermia associated with both are correct, without cause-
B)
the disease, because serum effect relationship
testosterone level becomes normal the first clause is correct, the
C)
after the introduction of hormone second clause is incorrect
replacement. the first clause is incorrect, the
D)
second clause is correct
both are correct, with cause-effect E) both are incorrect
A)
relationship
Show comment and answer
INT - 11.79 myelofibrosis, chronic myeloid
Increased ACTH secretion triggers not B) leukaemia, polycythaemia vera,
only cortisol production, but also thrombocythaemia
adrenal androgen production, therefore thrombocythaemia, chronic myeloid
in women suffering from adrenocortical C) leukaemia, polycythaemia vera,
carcinoma the symptoms of both hairy cell leukemia
Cushing’s syndrome and polycythaemia vera, hairy cell
hyperandrogenism may evolve. D) leukemia, agranulocytosis,
myelofibrosis
both are correct, with cause-effect Show comment and answer
A)
relationship INT - 13.2
both are correct, without cause- The most common hereditary
B)
effect relationship thrombophilia:
the first clause is correct, the
C)
second clause is incorrect A) antithrombin deficiency
the first clause is incorrect, the B) protein C deficiency
D)
second clause is correct activated protein C
E) both are incorrect C)
resistence/Leiden-mutation
Show comment and answer D) protein S deficiency
INT - 11.80
Show comment and answer
For patients with Addison’s disease on INT - 13.3
chronic adrenal hormone replacement Baseline aPTT prolonged with 12
therapy the daily dose of seconds, which can not be corrected
hydrocortisone pill is given in several with addition of normal plasma. You
equally divided amounts in equal time should suggest a background
intervals throughout the day, because
the aim of the treatment is to sustain a A) dysfibrinogenaemia
constantly stable, unchanged serum
lupus-
hydrocortisone (i.e. cortisol) level
B) anticoagulant/antiphospholipid
throughout the 24 hours of the day. syndrome
C) F XIIIdeficiency
both are correct, with cause-effect
A) D) antithrombin deficiency
relationship
both are correct, without cause- Show comment and answer
B) INT - 13.4
effect relationship
Which form of acute myeloid leukemia
the first clause is correct, the
C) is most likely, if there is a severe
second clause is incorrect
general easy bruising, signs of acute
the first clause is incorrect, the DIC. Marrow smear contains Auer rods
D)
second clause is correct
in myeloid precursory cells,
E) both are incorrect cytogenetics reveal translocation 15/17

HEMATOLOGY A) M1 undifferentiated myeloblastic
B) M2 differentiated myeloblastic
NT - 13.1 C) M3 promyelocytic
Please select from the list which D) M4 myelomonocytic
contains the four classic
E) M5 monocytic
myeloprolifrative neoplams
Show comment and answer
INT - 13.5
Hodgkin disease, chronic myeloid
Principal, fundamental induction
A) leukemia, polycythemia vera, hairy
therapy agents
cell leukemia
vincristin, prednisolon, cytosin- pathologic lymph node at the neck
A) D) and supraclavicular region with
arabinosid
vincristin, cytosin-arabinosid, fever and weight loss
B) Show comment and answer
procarbazin
INT - 13.9
antracyclin type agent, cytosin-
C) Please identify first line chemotherapy
arabinosid
protocol of Hodkgin lymphoma
D) antracyclin, vincristin, prednisolon
E) procarbazin, hydroxyurea CVP (cyclophosphamid + vincristin
A)
F) interferon, vincristin, prednisolon + prednisolon)
Show comment and answer CAF (cyclophosphamid +
INT - 13.6
B)
adriablastin + fluorouracil)
thrombocytopenia, direkt bilirubin TAD/HAM (thioguanin + Ara-C +
accumulation, oligo-anuria, apathy C) daunorubicin/high dose Ara-C +
mitoxantron)
school aged kid, fever following ABVD (adriamycin + bleomycin +
couple of day a recent diarrhoea, D)
vinblastin + dacarbazin)
moderate jaundice, mild renal
A) Show comment and answer
failure, leukocytosis,
INT - 13.10
thrombocytopenia, neurological
signs and cognitive dysfunction Please identify a non Hodgkin
lymphoma in which lymphadenopathy
elderly lady, hypofibrinogenaemia,
B) is not charateristic finding
splenomegaly, acut DIC
gastric cancer patient. A) hairy cell leukemia
C) hyperfibrinogenaemia. low LDH
activity, Coombs direect positive B) Waldenström-macroglobulinaemia
Show comment and answer C) Burkitt-lymphoma
INT - 13.7 D) multiple myeloma
Which alteration associates frequently E) mantle-cell lymphoma
with congenital spherocytosis Show comment and answer
INT - 13.11
A) arteriosclerosis obliterans The most probable background of
B) immunodeficiency hypochromic microcyctic anemia
C) gallstone disease
D) hypertension A) pernicious anemia
E) diarrhoea B) anemia sideropenica
F) constipation C) hyperthyreodism
Show comment and answer D) agranulocytosis
INT - 13.8 Show comment and answer
Which means Hodgin III/A Ann Arbor INT - 13.12
stage Hypersplenic hemopoesis can be
characterised as:
pathologic lymphnodes both sides
A) of the neck, axillary without fever leukocytosis with left shift
A)
and weight loss splenomegaly
pathological lymphnodes at the pancytopenia, portal hypertension,
B) neck and inguinal region with fever B) normal or increased reticulocyte
and weight loss count
pathological lymphnodes at the C) thrombocytosis, splenomegaly
C) neck and inguinal region without erythrocytosis, thrombocytosis,
fever and weight loss D)
leukocytosis, splenomegaly
Show comment and answer
INT - 13.13 of 1 mg/kg Prednisolon 80%
Which hermolytic disorder is associated achieves durable remissions
with pancytopenia In chronic ITP spontaneous
remissions are uncommon, first line
A) spherocytosis C)
1 mg/kg Prednisolon achieves
B) immunhemolytic anemia durable remissions in 10-15%
C) sickle cell trait Any kind of remisssion needs
D)
paroxysmal nocturnal combined immusuppressive therapy
D)
hemoglobinuria Show comment and answer
INT - 13.18
E) thalassaemia minor
Please identify the antidote of
Show comment and answer
unfractionated (Na) heparin
INT - 13.14
Non pegylated factor concentrate half
life in Hemophilia A A) Vitamin K 20 mg iv.
B) Dicynon 20 mg iv.
A) 2 hours C) ε-aminocapronic acid 4–8 g iv.
B) 6 hours D) protamine sulphate
C) 12 hours E) Styptanon
D) 24 hours F) DDAVP
E) 48 hours Show comment and answer
INT - 13.19
Show comment and answer
INT - 13.15 Megaloblastic (pernicious) anemia
Mild von Willebrand case with characteristics
moderate bleeding or prevention only
one of the agents efficient 1) Weak gastric acid output, only
after pentagstrin stimulation.
A) pentoxyphyllin 2) Pentagastrin refractory
achlorhydria in manifest anemia cases.
B) calcium dobesilat
3) Normoblast accumulation in bone
DDAVP- (desamino-D-arginin- marrow.
C)
vasopressin-) infusion 4) Presence of giant
D) somatostatin metamyelocytes in the marrow and
Show comment and answer blood.
INT - 13.16
In chilhood acute ITP prognosis: 1st, 2nd and 3rd answers are
A)
correct
If thrombocytopenia severe, B) 1st and 3rd answers are correct
A) spontaneous improvement is
C) 2nd and 4th answers are correct
uncommon
D) only 4th answer is correct
Spontaneous recovery is the most
B) E) all of the answers are correct
common
Improvement might be achieved Show comment and answer
C) INT - 13.20
only with 1-2 mg/kg prednison
Megaloblastic anemia typical findings
Show comment and answer
INT - 13.17
1) Mild hemolysis with moderately
In chronic ITP (non post infectious
increased indirect bilirubin
event) the prognosis
2) Leukocytosis and trombocytosis.
3) Mild leukopenia and
Spontaneous remissions are the
A) thrombocytopenia.
rule
4) Normal LDH activity and
Spontaneous remissions are hypocellular bone marrow
B)
uncommon, but after a short course
1st, 2nd and 3rd answers are present, myeloid maturation is blocked
A) at early stage.
correct
B) 1st and 3rd answers are correct 4) There is a pancytopenia and
severe bone marrow hypocellularity
C) 2nd and 4th answers are correct
D) only 4th answer is correct 1st, 2nd and 3rd answers are
E) all of the answers are correct A)
correct
Show comment and answer B) 1st and 3rd answers are correct
INT - 13.21
C) 2nd and 4th answers are correct
The definition of severe hemophilia
D) only 4th answer is correct
1) FVIII activity is 5–10%. E) all of the answers are correct
2) FVIII activity is less than 1% Show comment and answer
3) Petechiae and Rumpell Leede test INT - 13.24
positivity are frequently observed. General principles of coumarol
4) Spontaneous haemarthros, joint monitoring
bleedings are developing
1) Check thrombin time.
1st, 2nd and 3rd answers are 2) Check prothrombin time.
A) 3) INR prophylactic range between
correct
B) 1st and 3rd answers are correct 4-5
4) INR prophylactic range between
C) 2nd and 4th answers are correct
2–3
D) only 4th answer is correct
E) all of the answers are correct 1st, 2nd and 3rd answers are
A)
Show comment and answer correct
INT - 13.22 B) 1st and 3rd answers are correct
Identify the correct response
C) 2nd and 4th answers are correct
combinations in hemophilia A
D) only 4th answer is correct
1) plasma von Willebrand normal. E) all of the answers are correct
2) plasma von Willebrand-faktor Show comment and answer
decreased. INT - 13.25
3) Prolonged aPTT which can be FVII deficiency typical findings
corroected with normal plasma mixing
4) aPTT is prolonged, but thrombin 1) Rare conditions, not inducing
time is even more prolonged. bleeding events.
2) Rare condition with severe
1st, 2nd and 3rd answers are bleedings
A) 3) Prolonged aPTT, thrombin time,
correct
B) 1st and 3rd answers are correct along with prothrombin time, which all
can be corrected by addition of normal
C) 2nd and 4th answers are correct
plasma
D) only 4th answer is correct
E) all of the answers are correct 1st, 2nd and 3rd answers are
A)
Show comment and answer correct
INT - 13.23 B) 1st and 3rd answers are correct
True features of agranulocytosis
C) 2nd and 4th answers are correct
(febrile neutopenia)
D) only 4th answer is correct
1) Mostly drug associated. E) all of the answers are correct
2) Mostly idiopathic. Show comment and answer
3) No erythropetic or
thrombopoietic abnormalities are
INT - 13.26 6) thalidomide, revlimide
Typical attributes of chronic 7) antracyclines
lymphocytic leukemia 8) methotrexate

1) long disease course, with good 2nd, 5th and 6th answers are
therapeutic responses, no refractory A)
correct
cases
3rd, 6th and 8th answers are
2) durable course, rather indolent B)
correct
features, sometimes only parrtial
2nd, 4th and 7th answers are
responses to therapies, CD5/CD19 C)
correct
coexpressed cell count exceeds 5 g/L
in peripheral blood Show comment and answer
INT - 13.29
3) immunglobulins are elevated
ITP attributes:
4) low immunoglobulin levels
1) Isolated thrombocytopenia
1st, 2nd and 3rd answers are 2) pancytopenia and
A)
correct
thrombocytopenia.
B) 1st and 3rd answers are correct 3) Significant splenomegaly.
C) 2nd and 4th answers are correct 4) Splenomegaly is usually absent,
D) only 4th answer is correct however, steroid refractory cases may
E) all of the answers are correct respont with 60-70% durable
remission rate to splenectomy
Show comment and answer
INT - 13.27 5) Template bleeeding time
What is/are diagnostic criterium of prolonged.
chronic lymphocytic leukemia 6) Prolonged clotting times.

1) Fever, painful lymphadenopathy, A) 1st, 4th and 5th answers are correct
elevated CRP B) 2nd and 3rd answers are correct
2) Leukocytosis with left shift 2nd, 3rd and 6th answers are
3) Monoclonal gammopathy C)
correct
4) CD5/CD19 coexpression cell Show comment and answer
count exceeds 5 G/L in peripheral INT - 13.30
blood Heparin complications

1st, 2nd and 3rd answers are 1) Heparin induced


A)
correct thrombocytopenia (HIT) is a mild
B) 1st and 3rd answers are correct complication, may inducse some easy
C) 2nd and 4th answers are correct bruising, injection site reactions
2) HIT is frequently sever
D) only 4th answer is correct
complication, runs high mortality,
E) all of the answers are correct serious bleeding and simultaneous
Show comment and answer progressive thrombosis may develop
INT - 13.28 3) HIT is more common with low
Therapies which are able to prolong molecular weight heparins
multiple myeloma survival 4) HIT is more common with
unfractionated heparin dervatives.
1) Per os melphalan plus
5) Heparin may induce cholestatic
prednisolon
enzym elevations
2) bortezomib, proteasome
6) Heparin may induce
inhibitors
transaminase elevation.
3) Vincristin
4) Dexamethason
1st., 2nd and 3rd answers are
5) high dose melphalan With A)
correct
autologous stem cell transplantation
1st., 3rd and 5th answers are 1) Coumarol
B)
correct 2) Pentoxyphyllin
2nd., 4th and 6th answers are 3) Aspirin
C) 4) clopidogrel
correct
Show comment and answer 5) tissue plasminogen activator
INT - 13.31 6) low molecular weight heparin
Poor prognostic moments in Hodgkin 7) ε-aminokapron acid
lymphoma: 8) DDAVP (desamino-D-arginin-
vasopressin)
1) lymphoid depletion type
2) lymphoid predominant A) 1st and 2nd answers are correct
3) AnnArbor IV stage 3rd, 4th, 5th and 6th answers are
4) Ann Arbor I stage B)
correct
5) Age over 45 C) 7th and 8th answers are correct
6) Age under 45
D) all of the answers are correct
7) Interim PET CT shows
progression Show comment and answer
INT - 13.34
8) Stagnation or moderate
Thrombolytic therapy is absolutely
improvement on interim PET-CT
indicated
1st, 3rd, 5th and 7th answers are 1) phlegmasia coerulea dolens
A)
correct 2) submassive pulmonary embolism
B) 2nd and 4th answers are correct 3) deep vein thrombosis progressing
C) 6th and 8th answers are correct to femoral vein
2nd, 4th, 6th and 8th answers are 4) thrombotic signs within 12 hours
D) time frame
correct
Show comment and answer 5) a Inferior caval vein thrombosis,
INT - 13.32 affecting kidney veins + oligoanuria
Iron deficiency anemia can be 6) femoropopliteal thrombosis under
characterised the age of 20
7) thrombosis during pregnancy
1) MCV-85–100 fl range 8) upper limb deep vein thrombosis
2) az MCV-less than normal
3) low ferritin levels due to empty A) 1st and 5th answers are correct
iron stores B) 2nd and 6th answers are correct
4) reduced ferritin level is unusual
C) 3rd and 7th answers are correct
in iron deficiency
5) glossitis along with nail D) 4th and 8th answers are correct
abnormalitys might develop Show comment and answer
6) a histamin refractory INT - 13.35
achlorhydria is a diagnostic criterium Polycythemia vera typical
characteristics
1st, 2nd and 3rd answers are
A) 1) leukocytosis, thrombocytosis Jak
correct
V617F mutation negative,
2nd., 3rd and 5th answers are splenomegaly
B)
correct 2) leukocytosis, thrombocytosis,
C) 1st, 4th and 6th answers are correct moderate splenomegaly JAK-V613F
D) 4th, 5th and 6th answers are correct mutation
Show comment and answer 3) normal leukocyte and platelet
INT - 13.33 counts, oxigen saturation under 90%
Please select agents convenient to 4) after a longer period
treat acute or subacute arterial myelofibrotic transformation may
thrombosis occurr
5) Needs no intervention in early 3) lupus-
stage, phlebotomy is indicated, if anticoagulant/antiphospholipid-
elevated platelets counts are persistent antibody syndrome
6) Early combined 4) antithrombin deficiency
antacyclin+cytosin arabinoside based 5) unstable angina
combined induction therapy seems to 6) Prinzmetal angina
the most efficacious, so this could be 7) long lasting thrombocytosis
selected 8) reduced blood viscosity
9) immobilisation
A) 1st and 5th answers are correct
B) 3rd and 6th answers are correct A) 1st, 4th and 6th answers are correct
C) 2nd and 4th answers are correct 2nd, 3rd, 5th and 7th answers are
B)
correct
Show comment and answer
INT - 13.36 4th, 6th, 8th and 9th answers are
C)
identify typical coumarol complications correct
Show comment and answer
1) thrombocytopenia INT - 13.39
2) skin or intestinal necrosis Acute DIC characteristics
3) diarrhoea, abdominal distension
4) agranulocytosis 1) thrombocytopenia
5) if INR over 5, spontaneous 2) normal platelet count
bleedings 3) shortened PT/aPTT
6) Spontaneous bleedings with INR 4) Prolonged PT/aPTT
over 2 5) hypofibrinogenaemia
6) elevated antithrombin
A) 1st, 3rd and 4th answers are correct
B) 2nd and 5th answers are correct A) 1st, 4th and 5th answers are correct
C) 4th and 6th answers are correct 2nd, 3rd and 6th answers are
B)
correct
Show comment and answer
INT - 13.37 Show comment and answer
Unfractionated heparin monitoring INT - 13.40
feasible tools Chronic DIC features

1) prothrombin time 1) prolonged aPTT


2) aPTT 2) slightly shortened aPTT
3) global clotting time measured 3) thrombotic events
with appropriate bedside device 4) severe bleedings
4) urea lysability test 5) hypofibrinogenaemia
5) APC- resistance 6) elevated D-dimer
7) severe thrombocytopenia
A) 1st and 4th answers are correct
A) 1st and 4th answers are correct
B) 1st and 5th answers are correct
B) 1st, 5th and 7th answers are correct
C) 2nd and 3th answers are correct
2nd, 3rd and 6th answers are
D) 4th and 5th answers are correct C)
correct
Show comment and answer
Show comment and answer
INT - 13.38
INT - 13.41
Please select condition predisposing to
Typical backgrounds of acute DIC
arterial thrombosis
1) Thrombotic thrombocytopenic
1) FV. Leiden mutation
purpura acute DIC is aways present
2) hyperhomocysteinaemia
2) major cranial trauma
3) intrauterin fetal death
4) prostatic cancer
5) Romano–Ward-syndrome 3rd, 5th and 9th answers are
C)
6) multiple myeloma correct
7) acute promyelocytic leukemia Show comment and answer
INT - 13.44
A) 1st and 5th answers are correct Typical features of chronic myeloid
2nd, 3rd, 4th and 7th answers are leukemia
B)
correct
1) Modest splenomegaly
C) 5th and 6th answers are correct
2) Major splenomegaly
Show comment and answer 3) The entire granulocyte
INT - 13.42
maturation stages cells are
FXIII deficiency typically cause
represented in peripheral blood
1) severe bleeding tendency 4) Only blasts and matured
2) predisposing to thrombosis neutrophils can be seen in peripheral
3) prolonged bleeding time blood smears, intermediary forms are
4) repeated spontanous abortions absent
5) increased keloid scars at surgical 5) Imatinib or second third
wounds generation tyrosike kinase inhibitors
6) inreased likelyhood of glaucoma achieve a complete haematological,
cytogenetic and even molecular
A) 1st, 4th and 5th answers are correct remission in the majoritiy of chronic
phase patients, resulting in huge
2nd., 3rd and 6th answers are
B) survival benefit
correct
6) Optimal treatment of chronic
Show comment and answer phase is prednisolon +
INT - 13.43
cyclophosphamid
Characteristics of chronic myeloid
leukemia
A) 1st, 4th and 6th answers are correct
1) survival without specific 2nd, 3rd and 5th answers are
B)
treatment is around 5 years correct
2) survival without specific Show comment and answer
treatment is around 12 years INT - 13.45
3) untreated cases average survival Reactive leukocytosis might be
is 1 years expected
4) Cromosome 12 trisomy is a
hallmark finding 1) Bacterial bronchopneumonia
5) 15,19 translocation is typical 2) B virus hepatitis
6) By definition Philadelphia 3) Monoucleosis infectiosa
chromosome and bcr/abl 4) exsiccosis
rearrangerment is present 5) anaemia perniciosa
7) A chronic, accelerated, and 6) cholecystitis acuta
blastic phase typicallly develops
without specific treatment 1st, 2nd and 5th answers are
A)
8) the disease remains in chronic correct
phase during the entire clinical course B) 1st, 5th and 6th answers are correct
9) blastic phase is a typical finding 1st, 3rd, 4th and 6th answers are
C)
in early accelerated cases, but very correct
uncommon with early chronic phase Show comment and answer
cases INT - 13.46
Leukopenia might be expected
A) 1st, 6th and 7th answers are correct
2nd, 4th and 8th answers are 1) hyperthyreodism
B) 2) purulent meningitis
correct
3) abscedent pneumonia
4) AIDS premanifest stage 5) Willebrand-disease
5) acute víral hepatitis 6) unfractionated heparin therapy
6) autoimmune diseases
7) endocarditis infectiva A) 1st and 2nd answers are correct
B) 2nd and 4th answers are correct
1st, 2nd and 6th answers are 3rd, 4th and 6th answers are
A) C)
correct correct
2nd, 6th and 7th answers are Show comment and answer
B)
correct INT - 13.50
1st, 4th, 5th and 6th answers are aPTT will be prolonged
C)
correct
Show comment and answer 1) immun thrombocytopenic
INT - 13.47 purpura
Eosinophilia might be expected 2) acute DIC
3) Unfractionated heparin therapy
1) Churg Strauss syndrome 4) Low molecular weight heparin
2) Loeffler-endocarditis administration
3) sphaerocytosis 5) lupus-anticoagulant case
4) chronic myeloid leukemia, 6) clopidogrel therapy
accelerated phase 7) Antithrombin deficiency
5) plasmocytoma
6) aplastic anemia A) 1st and 4th answers are correct
2nd, 3rd and 5th answers are
A) 1st, 3rd and 5th answers are correct B)
correct
1st, 2nd and 4th answers are C) 6th and 7th answers are correct
B)
correct
Show comment and answer
2nd, 3rd and 5th answers are INT - 13.51
C)
correct Typical findings in hemolysis
3rd, 5th and 6th answers are
D)
correct 1) direct bilirubin elevated
Show comment and answer 2) indirect bilirubin elevated
INT - 13.48 3) ubg reaction increased in urine
Bleeding time will be prolonged 4) bilirubin positívity in urine
5) high reticulocyte count
1) ITP (immun thrombocytopenic 6) low reticulocyte count
purpura)
2) aspirin treatment A) 1st and 4th answers are correct
3) Coumarol administration B) 4th and 6th answers are correct
4) haemophilia A
C) 1st, 4th and 6th answers are correct
5) haemophilia B
6) von Willebrand-disease 2nd, 3rd and 5th answers are
D)
correct
1st, 2nd and 6th answers are Show comment and answer
A) INT - 13.52
correct
Hemolysis suggestive findings
3rd, 4th and 5th answers are
B)
correct 1) Coombs-positivity
Show comment and answer 2) pruritus
INT - 13.49 3) hypocellular hemopoiesis
Clotting time will prolonged 4) elevated LDH activity
5) high GOT/GPT ratio
1) Glanzmann-thrombasthenia
6) pleiochrom, dark stool
2) aspirin user
3) coumarol treatment
A) 1st, 4th and 6th answers are correct
4) haemophilia A
2nd, 3rd and 5th answers are 6) chronic myeloid leukemia
B)
correct 7) thombocythemia essentialis
C) 2nd, and 3rd answers are correct 8) mycosis fungoides
D) 3rd and 5th answers are correct 9) Sezary-syndrome
Show comment and answer
INT - 13.53
A) 1st and 2nd answers are correct
Diseases associted with hemolysis 3rd, 4th, 6th and 7th answers are
B)
correct
1) plasmocytoma C) 5th, 8th and 9th answers are correct
2) acute ITP (immun Show comment and answer
thrombocytopenic purpura) INT - 13.56
3) paroxysmal nocturnal Macrocytic anemia can be found
haemoglobinuria
4) aplastic anemia 1) hyperthyroidism
5) sphaerocytosis 2) anemia perniciosa
6) Dubin–Johnson-syndrome 3) anemia sideropenica
7) anemia perniciosa 4) hypothyroididms
8) thrombotic thrombocytopenic 5) spherocytosis
purpura/Moschcowitz-syndrome
9) Glanzmann-thrombasthenia A) 1st and 2nd answers are correct
B) 2nd and 4th answers are correct
A) 1st and 4th answers are correct C) 3th and 5th answers are correct
B) 4th, 6th and 9th answers are correct Show comment and answer
3rd, 5th, 7th and 8th answers are INT - 13.57
C)
correct Normocytic anemia should be expected
Show comment and answer
INT - 13.54 1) chronic iron deficiency
Two of the following conditions may be 2) B12-malabsorption
associated with hemolysis 3) folic acid impaired absorption
4) recent major bleeding
1) ulcerative colitis 5) some cancers
2) Crohn-disease
3) blood group mismatched blood A) 1st and 2nd answers are correct
transfurion B) 2nd and 3th answers are correct
4) severe praeecclampsy (HELLP)
C) 4th and 5th answers are correct
5) myelofibrosis
6) polycythaemia vera Show comment and answer
INT - 13.58
7) thrombocythaemia essentialis
Severe anemia signs and complaints
A) 1st and 7th answers are correct 1) central cyanosis
B) 2nd and 6th answers are correct 2) peripheral cyanosis
C) 3rd and 4th answers are correct 3) accidental ejection type systolic
D) 5th and 6th answers are correct murmur
4) tinnitus
Show comment and answer
INT - 13.55 5) dizzyness
Identify chronic myeloproliferative 6) high fever
neoplasm among the listed conditions 7) dyspnea
8) hypotension
1) hemochromatosis 9) vomiting
2) Waldenström-macroglobuliemia
3) polycythaemia vera 1st, 2nd, 6th and 8th answers are
A)
4) myelofibrosis correct
5) megaloblastic anemia
2nd, 6th, 8th and 9th answers are 3) refractory anemia with blasts
B)
correct 4) 5q- syndrome
3rd, 4th, 5th and 7th answers are 5) hemochromatosis
C) 6) ITP
correct
Show comment and answer
INT - 13.59 A) 1st and 2nd answers are correct
Immuno-hemolytic characteristic 1st, 2nd and 5th answers are
B)
findings correct
C) 3rd and 4th answers are correct
1) Coombs positivity
Show comment and answer
2) GOT elevated INT - 13.62
3) alcaline phosphatase elevated You can find elevated iron plasma
4) LDH elevated levels
5) splenectomy helps in 50%
6) splenectomy helps in most 1) myeloma multiplex
instances 2) hemochromatosis
7) a splenectomy clearly useless 3) Willebrand disease in females
4) hemolytic crisis
A) 1st, 4th and 5th answers are correct 5) aplastic anemia
B) 3rd and 7th answers are correct 6) agranulocytosis
3rd, 6th and 7th answers are
C) A) 1st and 6th answers are correct
correct
Show comment and answer B) 1st, 5th and 6th answers are correct
INT - 13.60 C) 2nd and 4th answers are correct
Three statements are correct in diffuse Show comment and answer
large B cell lymphoma INT - 13.63
The following two lymphomas are
1) indolent lymphoma
classified as aggressive
2) aggressive lymphoma
3) antiCD20 based 1) hairy cell leukemia
immunochemotherapy achieves 2) chronic lymphoid leukaemia
definitive cure in substantial part of 3) diffuse large B-cell lymphoma
patients 4) Burkitt-lymphoma
4) antiCD20 based 5) plasmocytoma
immunochemotherapy should be
applied, but it never achieves final cure A) 1st and 2nd answers are correct
5) In refractory/relapsed cases high
B) 2nd and 5th answers are correct
dose chemotherapy/autologous
transplant should be performed C) 3rd and 4th answers are correct
6) Diffuse large B cell lymphoma is D) 4th and 5th answers are correct
CD20 negative, so anti CD20 therapy Show comment and answer
is not applicable INT - 13.64
Two lymphomas are indolent
2nd, 3th and 5th answers are
A) 1) chronic lymphoid leukemia
correct
2) small B cell gastric MALT-
B) 3rd and 5th answers are correct
lymphoma
C) 4th and 5th answers are correct 3) Burkitt-lymphoma
D) 2nd and 6th answers are correct 4) diffuse large B-cell lymphoma
Show comment and answer
INT - 13.61 A) 1st and 2nd answers are correct
Myelodysplastic conditions are B) 2nd and 4th answers are correct
1) chronic myeloid leukemia C) 3rd and 4th answers are correct
2) myelofibrosis Show comment and answer
INT - 13.65 4) Bone marrow hypercellular,
Feasible methods to detect empty iron peripheral cytopenia is due to blocked
stores maturation
5) In aplastic anemia you see
1) serum iron level hypoplastic bone marrow with
2) Prussian blue stain, bone marrow increased reticulocyte acount
3) Shilling-test 6) Antithymocyte globulin +
4) Ham-test cyclosporin A immunosuppression may
5) serum-ferritin achieve 60-70% improvement,
6) β2-microglobulin sometimes cure
7) PAS-reaction, bone marrow 8) Final cure can be achieved with
allogeneous bonemarrow
A) 1st, 3rd and 7th answers are correct transplantation, but it carries high risk
3rd, 4th and 6th answers are if compatible sibling donor is not
B)
correct available
C) 2nd and 5th answers are correct
Show comment and answer A) 1st, 4th and 7th answers are correct
INT - 13.66 1nd, 5th and 8th answers are
B)
Typical hematological findings and correct
alterations in AIDS 2nd, 3rd, 6th and 8th answers are
C)
correct
1) leukocytosis
Show comment and answer
2) polyglobulia INT - 13.68
3) thombocytosis Important viruses in aplastic anemia
4) thrombocytopathia etiopathogenesis
5) reduced CD4 cell count
6) increased CD4 count 1) Epstein–Barr-vírus
7) blood count alterations and 2) cytomegalovirus
obstipation 3) parvovirus
8) in AIDS premanifest stage 4) hepatitis A vírus
lymphadenopathy is frequent, but later 5) HIV virus
the nodes shrink spontaneously 6) hepatitis B, C vírus
9) increased incidence of primary 7) mumpsvírus
cerebral lymphoma
1st, 2nd and 4th answers are
A) 5th, 8th and 9th answers are correct A)
correct
2nd, 3rd, 6th, 7th and 9th answers 2nd, 4th and 7th answers are
B) B)
are correct correct
Show comment and answer 3rd, 5th and 6th answers are
INT - 13.67 C)
correct
Characteristics of severe aplastic
Show comment and answer
anemia INT - 13.69
Select correct features of
1) Selective erythropoiesis
agranulocytosis (febrile neutropenia)
deficiency, maintained white blood cell
and platelet generation 1) Bone marrow aplasia, involving
2) Pancytopenia, decreased all cell lines
myeloid, erythroid and platelet 2) Bone marrow: Large number of
generation early myeloid precursors, blocked in
3) Bone marrow hypocellular, maturation, other cell lines are
mostly fatty areas, with some preserved
lymphocytes 3) Normal hemopoiesis in bone
marrow
4) Agranocytosis improves with the 6) Aggressive course is typical in
cessation of provocative drug. vast majority
5) This improvement can be slightly
quicker with bone marrow cytokin A) 1st, 3rd and 6th answers are correct
administration 2nd, 4th and 5th answers are
6) Agranulocytosis is not a drug B)
correct
induced condition, so cessation of Show comment and answer
previous drugs is unnecessary. INT - 13.72
7) Large dose fresh frozen plasma Coumarol overdose induced major
administration helps agranulocytotic bleeding can be stopped by only three
patients of the choices

1st, 6nd and 7th answers are 1) protamin sulhphate


A)
correct 2) Low molecular weight heparin
2nd, 4th and 5th answers are 3) ε-aminocaproic acid
B)
correct 4) Fresh frozen plasma
3rd, 6th and 7th answers are 5) DDAVP
C) 6) Novoseven (rFVIIa) iv.
correct
Show comment and answer 7) prothrombin complex concentrate
INT - 13.70
You should apply perioperative 1st, 2nd and 5th answers are
A)
anticoagulant prophylaxis correct
2nd, 3rd and 4th answers are
B)
1) all surgery lasting more than 20 correct
minutes C) 4th, 6th and 7th answers are correct
2) 45 minutes or longer operation, Show comment and answer
even if laparoscopic INT - 13.73
3) all cases with genera l Melyik a legvalószínűbb betegség?
anaesthesia
4) neoplasma surgery a 16 years old girl admitted to your
5) hip joint or fracture surgery ward, looks really sick. History no
6) all upper limb bone surgery meaningful previous event, she had
regular sport activity, but 8 days ago
A) 1st, 3rd and 6th answers are correct she had transient mild fever and
2nd, 4nd and 5th answers are diarrhoea for 2 days. Last day she had
B)
correct fever again, headache, dizziness, mild
Show comment and answer confusion, she coul not even recognise
INT - 13.71 her sister. Small petechiae and
Correct statements in mantlecell suffusiones appeared. You observe
lymphoma somnolence, mild jaundice. No
meningeal signs. Liver, spleen,
1) Dense CD 20+ lymphoma so anti lymphnodes are normal sized. Platelet
CD20 based intensive count 10 G/L, WBC 15 G/L urea 11
immunochemotherapy should be mmol/l, hemostasis screening tests
applied noirmal.
2) CD20 negative, so anti CD 20
therapy is not necessary A) acute myeloid leukemia
3) Autologous transplant should be B) acute lymphoid leukemia
performed in first remission
acute immun thrombocytopenic
4) Indolent course is frequent in C)
purpura (ITP)
which watch and wait approach is
Thrombotic mikroangiopathia, HUS
enough D)
(hemolytic uremic syndrome)
5) indolent lymphoma
E) acute víral hepatitis
F) acute DIC Which are the most useful disease
Show comment and answer specific diagnostic tools
INT - 13.74
Which are the most appropriate a 16 years old girl admitted to your
examination in this case to prove ward, looks really sick. History no
suggested diagnosis meaningful previous event, she had
regular sport activity, but 8 days ago
a 16 years old girl admitted to your she had transient mild fever and
ward, looks really sick. History no diarrhoea for 2 days. Last day she had
meaningful previous event, she had fever again, headache, dizziness, mild
regular sport activity, but 8 days ago confusion, she coul not even recognise
she had transient mild fever and her sister. Small petechiae and
diarrhoea for 2 days. Last day she had suffusiones appeared. You observe
fever again, headache, dizziness, mild somnolence, mild jaundice. No
confusion, she coul not even recognise meningeal signs. Liver, spleen,
her sister. Small petechiae and lymphnodes are normal sized. Platelet
suffusiones appeared. You observe count 10 G/L, WBC 15 G/L urea 11
somnolence, mild jaundice. No mmol/l, hemostasis screening tests
meningeal signs. Liver, spleen, noirmal.
lymphnodes are normal sized. Platelet
count 10 G/L, WBC 15 G/L urea 11 1) Bone marrrow myeloblast flow
mmol/l, hemostasis screening tests cytometry
noirmal. 2) t15,17 translocation, suggesting
acute promyelocytic leukemia, to
1) Bone marrow – myeloblast indicate retinooid therapy
accumulation 3) Durable increment of platelet
2) Bone marrow – lymphoblast counts after platelet transfusion, a sin
accumulation ITP foreign platelets are not affected
3) Bone marrow – accumulation by antibodies
4) GOT-/GPT 4) test if patient plasma provokes
5) HbsAg screening platelet aggregation in control
6) LDH-activity specimen
7) peripheral blood smear, 5) verotoxin ELISA
leukocytosis analysis 6) ADAM TS13 activity or
8) peripheral blood smear, complementactivation measurement
lymphoblast search 7) TFPI measurement (tissue Factor
9) peripheral blood smear, inhibitor)
fragmentocyte (fragmented sred blood
cells) screen A) 1st and 7th answers are correct
10) Platelet surface immunglobulin B) 5th and 6th answers are correct
detection C) 3rd and 7th answers are correct
11) kidney function analysis
D) 4th and 5th answers are correct
1st, 4th and 11th answers are E) all of the answers are correct
A) Show comment and answer
correct
INT - 13.76
2nd, 5th and 10th answers are
B) Which therapeutic approach is the
correct
most appropriate
6th, 9th and 11th answers are
C)
correct a 16 years old girl admitted to your
7th, 8th and 10th answers are ward, looks really sick. History no
D)
correct meaningful previous event, she had
Show comment and answer regular sport activity, but 8 days ago
INT - 13.75 she had transient mild fever and
diarrhoea for 2 days. Last day she had
fever again, headache, dizziness, mild 3rd, 4th and 5th answers are
C)
confusion, she coul not even recognise correct
her sister. Small petechiae and Show comment and answer
suffusiones appeared. You observe INT - 13.78
somnolence, mild jaundice. No Which more recent therapy can also be
meningeal signs. Liver, spleen, helpful
lymphnodes are normal sized. Platelet
count 10 G/L, WBC 15 G/L urea 11 a 16 years old girl admitted to your
mmol/l, hemostasis screening tests ward, looks really sick. History no
noirmal. meaningful previous event, she had
regular sport activity, but 8 days ago
A) AML induction chemoterapy she had transient mild fever and
B) ALL induction chemotherapy diarrhoea for 2 days. Last day she had
fever again, headache, dizziness, mild
C) retinoic acid administration
confusion, she coul not even recognise
D) Antibiotics her sister. Small petechiae and
high dose plasma suffusiones appeared. You observe
E)
administration/plasmaapheresis somnolence, mild jaundice. No
F) aspirin meningeal signs. Liver, spleen,
Show comment and answer lymphnodes are normal sized. Platelet
INT - 13.77 count 10 G/L, WBC 15 G/L urea 11
What interventions are contraindicated mmol/l, hemostasis screening tests
int he acute phase of this disease noirmal.

a 16 years old girl admitted to your A) heparin


ward, looks really sick. History no autologous bone marrow
meaningful previous event, she had B)
transplantation
regular sport activity, but 8 days ago
allogeneous bone marrow
she had transient mild fever and C)
transplantation
diarrhoea for 2 days. Last day she had
D) iv. anti cd20 monoclonal antibody
fever again, headache, dizziness, mild
confusion, she coul not even recognise E) high dose interferon
her sister. Small petechiae and Show comment and answer
suffusiones appeared. You observe INT - 13.79
somnolence, mild jaundice. No With adequate diagnosis and therapy
meningeal signs. Liver, spleen, what is the correct prognosis
lymphnodes are normal sized. Platelet
count 10 G/L, WBC 15 G/L urea 11 a 16 years old girl admitted to your
mmol/l, hemostasis screening tests ward, looks really sick. History no
noirmal. meaningful previous event, she had
regular sport activity, but 8 days ago
1) thrombocyte transfusion she had transient mild fever and
2) colony stimulators (GCSF, GM- diarrhoea for 2 days. Last day she had
CSF) fever again, headache, dizziness, mild
3) prednisolon confusion, she coul not even recognise
4) antibiotics her sister. Small petechiae and
5) diuretics suffusiones appeared. You observe
6) clopidogrel somnolence, mild jaundice. No
7) aspirin meningeal signs. Liver, spleen,
lymphnodes are normal sized. Platelet
A) 1st, 6th and 7th answers are correct count 10 G/L, WBC 15 G/L urea 11
mmol/l, hemostasis screening tests
2nd, 6th and 7th answers are
B) noirmal.
correct
90% mortality remains unchanged- her sister. Small petechiae and
A) suffusiones appeared. You observe
in spite of therapeutic efforts
B) 5 years survival 15–30% somnolence, mild jaundice. No
meningeal signs. Liver, spleen,
with steroids+splenectomy you can
lymphnodes are normal sized. Platelet
C) achieve definitive cure, sometimes
with residual easy bruising count 10 G/L, WBC 15 G/L urea 11
mmol/l, hemostasis screening tests
defintive cure rate is 90–95%, some
D) noirmal.
cases meay have recidive episodes
Spontaneous improvement 80%, A) retinoin receptor anomaly
E)
with treatment it is almost 100%
B) previous alkylator therapy
Show comment and answer
INT - 13.80 C) previous irradiation
This disease might have recidives more D) hypertension
likely in ADAMTS13 or complement
E)
alteration
a 16 years old girl admitted to your F) multidrug-resistane
ward, looks really sick. History no
Show comment and answer
meaningful previous event, she had
INT - 13.82
regular sport activity, but 8 days ago Which intervention is strictly
she had transient mild fever and contraindicated in this disease
diarrhoea for 2 days. Last day she had
fever again, headache, dizziness, mild a 16 years old girl admitted to your
confusion, she coul not even recognise ward, looks really sick. History no
her sister. Small petechiae and meaningful previous event, she had
suffusiones appeared. You observe regular sport activity, but 8 days ago
somnolence, mild jaundice. No she had transient mild fever and
meningeal signs. Liver, spleen, diarrhoea for 2 days. Last day she had
lymphnodes are normal sized. Platelet fever again, headache, dizziness, mild
count 10 G/L, WBC 15 G/L urea 11 confusion, she coul not even recognise
mmol/l, hemostasis screening tests her sister. Small petechiae and
noirmal. suffusiones appeared. You observe
somnolence, mild jaundice. No
A) pneumococcus-pneumonia meningeal signs. Liver, spleen,
Philadelphia-chromosome anomaly lymphnodes are normal sized. Platelet
B)
present count 10 G/L, WBC 15 G/L urea 11
lg H gene rearrangement is mmol/l, hemostasis screening tests
C)
persistent noirmal.
D) pregnancy
E) aspirin use A) Red blood cell-transfusion
F) Taking coumarols B) Platelet transfusion
Show comment and answer C) splenectomy
INT - 13.81 colony stimulators, fdue to
D)
Dominant pathogentic factor in this leukemogenicity
disease Show comment and answer
INT - 13.83
a 16 years old girl admitted to your What is the most likely diagnosis
ward, looks really sick. History no suspected
meaningful previous event, she had
regular sport activity, but 8 days ago You see a 62 years old male, with
she had transient mild fever and cherry sized neck, axillary, inguinal
diarrhoea for 2 days. Last day she had painless lymphadenopathy. Moderate
fever again, headache, dizziness, mild wekness and weightloss. he had some
confusion, she coul not even recognise mild fever and upper airway infections
recently, Spleen enlarged by 6 cm. White Blood cell count 23 G/L, 76%
White Blood cell count 23 G/L, 76% small matured lymphocyte, oither
small matured lymphocyte, oither forms are normal. Hemoglobin 105 g/l,
forms are normal. Hemoglobin 105 g/l, platelet count normal. Reduced IgG
platelet count normal. Reduced IgG
A) Epstein–Barr-víral serology
A) lymphogranulomatosis Epstein–Barr-vírus genomic
B)
B) chronic lymphoid leukemia analysis in malignant cells
C) acute lymphoid leukemia Bcr-Abl rearrangement analysis,
C)
D) chroic myeloid leukemia even if cytogenetics are negative
E) mononucleosis infectiosa D) Philadelphia chomosome detection
Show comment and answer PAS and acid phosphatase
E)
INT - 13.84 citochemistry
What are the most helpful diagnostic F) peripheral blood FISH analysis
tools in this case Show comment and answer
INT - 13.86
You see a 62 years old male, with What clinical signs, complications you
cherry sized neck, axillary, inguinal should check and foresee for longer
painless lymphadenopathy. Moderate period
wekness and weightloss. he had some
mild fever and upper airway infections You see a 62 years old male, with
recently, Spleen enlarged by 6 cm. cherry sized neck, axillary, inguinal
White Blood cell count 23 G/L, 76% painless lymphadenopathy. Moderate
small matured lymphocyte, oither wekness and weightloss. he had some
forms are normal. Hemoglobin 105 g/l, mild fever and upper airway infections
platelet count normal. Reduced IgG recently, Spleen enlarged by 6 cm.
White Blood cell count 23 G/L, 76%
1) lymph node biopsy small matured lymphocyte, oither
2) Gumprecht shadows in peripheral forms are normal. Hemoglobin 105 g/l,
blood smears platelet count normal. Reduced IgG
3) Bone marrow examination
4) citogenetics screening for 1) slowly progressing
Philadelphia-chromosome lymphadenomegaly
5) peripheral blood flowcytometry. 2) splenic rupture
CD5/CD19 coexpressed cell count 3) hepatitis like symptoms
measurement 4) myocarditis like symptoms
5) increased likelyhood of infections
A) 1st and 3rd answers are correct 6) transformation into accelerated
B) 2nd and 4th answers are correct or blastic phase
C) 2nd and 5th answers are correct 7) transformation into myelofibrosis
8) After the cure of
D) 4th and 5th answers are correct
lymphogranulomatosis secondary
E) all of the answers are correct cancer might develop
Show comment and answer
INT - 13.85 A) 2nd and 4th answers are correct
What diagnostic tool is the most useful
B) 6th and 7th answers are correct
in this condition to define prognosis
C) 3rd and 8th answers are correct
You see a 62 years old male, with D) 1st and 5th answers are correct
cherry sized neck, axillary, inguinal Show comment and answer
painless lymphadenopathy. Moderate INT - 13.87
wekness and weightloss. he had some Mennyi a betegség átlagos túlélése
mild fever and upper airway infections kezelés mellett?
recently, Spleen enlarged by 6 cm.
You see a 62 years old male, with C) 4th and 6th answers are correct
cherry sized neck, axillary, inguinal Show comment and answer
painless lymphadenopathy. Moderate INT - 13.89
wekness and weightloss. he had some What is the adequate therapy in this
mild fever and upper airway infections case
recently, Spleen enlarged by 6 cm.
White Blood cell count 23 G/L, 76% You see a 62 years old male, with
small matured lymphocyte, oither cherry sized neck, axillary, inguinal
forms are normal. Hemoglobin 105 g/l, painless lymphadenopathy. Moderate
platelet count normal. Reduced IgG wekness and weightloss. he had some
mild fever and upper airway infections
A) spontaneous cure in all recently, Spleen enlarged by 6 cm.
One or two years in spite of White Blood cell count 23 G/L, 76%
B) small matured lymphocyte, oither
therapeutic efforts
Four years chronic phase, followed forms are normal. Hemoglobin 105 g/l,
C) platelet count normal. Reduced IgG
by one year accelerated phase
Fove year survival is 55–60% in
D) hydroxyurea first line, may consider
Hodgkin lymphoma
interferons which might induce
8–15 average survivial, however A)
cytogenetic response of Phildelphia
E) final cure can be achieved even abnormality
with modern therapies
ALL-standard agents: antracyclin,
Show comment and answer B) steroid, vincristin, methotrexate
INT - 13.88
polychemotherapy
Which alteration carries important
in mononucleosis observation is
prognostic relevance
C) enough without any intervention
You see a 62 years old male, with mononucleosis
cherry sized neck, axillary, inguinal COPP (cyclophosphamide +
painless lymphadenopathy. Moderate D) vincristin + procarbazin +
wekness and weightloss. he had some prednisolon) procoll, 6 cycles
mild fever and upper airway infections watch and wait, if progression
recently, Spleen enlarged by 6 cm. (quick doubling of white blood cell
White Blood cell count 23 G/L, 76% count, gross lymphadenopathy, B
small matured lymphocyte, oither signs) first line should be
E)
forms are normal. Hemoglobin 105 g/l, Fludarabine-Cyclophosphamid-
MabThera immunchemotherapy, in
platelet count normal. Reduced IgG
elderly, moderately frail patients
1) histological subtype, as lymphoid MabThera+bendamustin
depletion cases run poor prognosis Show comment and answer
2) In ALL Philadelphia chromosome INT - 13.90
means very bad prognosis Which 3 alteration may help the most
3) lymphocyte doubling time shorter during followup
than 6 month
You see a 62 years old male, with
4) anemia, thrombocytopenia,
cherry sized neck, axillary, inguinal
medullary infiltration
painless lymphadenopathy. Moderate
5) male sex in itself has worse
wekness and weightloss. he had some
prognosis in Hodgkin
mild fever and upper airway infections
6) In chronic lymphocytic leukemia
recently, Spleen enlarged by 6 cm.
FISH p53 positivity especaillay bad
White Blood cell count 23 G/L, 76%
prognosis
small matured lymphocyte, oither
forms are normal. Hemoglobin 105 g/l,
A) 1st and 3rd answers are correct
platelet count normal. Reduced IgG
B) 3rd and 6th answers are correct
1) extent of splenomegaly
2) extent and progression of
lymphadenomegalia 26 years old pregnant lady, 16 weeks
3) kinetics of white blood cell count of pregnancy arrives to your office.
increase Progressive swelling and pain
4) immunglobulin levels developed in he right femoral region,
5) progression of FISH alterations four days ago. Pregnancy goes on
6) EBV-antibody -titer changes normally otherwise. Doppler
7) serum-copper levels ultrasound reveals newly developed
deep vein thrombosis in right femoral
A) 1st, 5th and 7th answers are correct vein and popliteal vein. The patient
B) 5th, 6th and 7th answers are correct mother and sister alsohad thrombtoic
episodes.
2nd, 3rd and 5th answers are
C)
correct
A) Low molecular weight heparin
D) 1st and 5th answers are correct
B) coumarol derivatives
E) all of the answers are correct
C) dabigatran
Show comment and answer
INT - 13.91 D) rivaroxaban
What other examinations should Show comment and answer
should be performed before starting INT - 13.93
anticoagulant therapy? Wich anthrombotic therapy should be
selected in this case?
26 years old pregnant lady, 16 weeks
of pregnancy arrives to your office. 26 years old pregnant lady, 16 weeks
Progressive swelling and pain of pregnancy arrives to your office.
developed in he right femoral region, Progressive swelling and pain
four days ago. Pregnancy goes on developed in he right femoral region,
normally otherwise. Doppler four days ago. Pregnancy goes on
ultrasound reveals newly developed normally otherwise. Doppler
deep vein thrombosis in right femoral ultrasound reveals newly developed
vein and popliteal vein. The patient deep vein thrombosis in right femoral
mother and sister alsohad thrombtoic vein and popliteal vein. The patient
episodes. mother and sister alsohad thrombtoic
episodes.
1) Iv. phlebography, as all types of
anticoagulants are extremely only aspirin can be applied in
A)
dangerous during pregnancy, to pregnancy
ascertain more your diagnosis Low molecular weight heparin,
B)
2) aPTT therapeutic dose
3) fibrinogen level couumarin, as all heparins are
4) Factor VIII. activity C)
contraindicated in pregnant patients
5) baseline platelet count D) thrombolytic treatment
6) Bleeding time measurememnt
E) surgical thrombectomy
A) 1st and 3rd answers are correct Show comment and answer
INT - 13.94
B) 2nd and 5th answers are correct Considering all this, what are the
C) 4th and 6th answers are correct feasible options
D) 1st and 5th answers are correct
E) all of the answers are correct 26 years old pregnant lady, 16 weeks
of pregnancy arrives to your office.
Show comment and answer
Progressive swelling and pain
INT - 13.92
developed in he right femoral region,
Which anticoagulat is what does not
four days ago. Pregnancy goes on
cross placental barrier and can be
normally otherwise. Doppler
adminitered safely during pregnancy?
ultrasound reveals newly developed
deep vein thrombosis in right femoral Progressive swelling and pain
vein and popliteal vein. The patient developed in he right femoral region,
mother and sister alsohad thrombtoic four days ago. Pregnancy goes on
episodes. normally otherwise. Doppler
ultrasound reveals newly developed
1) aspirin deep vein thrombosis in right femoral
2) aspirin + coumarol vein and popliteal vein. The patient
3) Unfractionated-heparin 1000– mother and sister alsohad thrombtoic
1200 U/hour, infusion episodes.
4) immobilisation, elastic bandage
on the thrombotic leg 1) Coumarols
5) Low molecular weight heparin 2) aspirin
twice/day 100–120 U/kg 3) Low molecular weight heparin
subcutaneously. profilactic dose once daily 1
6) streptokinase or tissue 4) elastic bandage
plasminogen activator 5) prophilaxis is not necessary
7) pentoxyphyllin 6) prohylaxis is dangerous in
pregnancy, so not recommended
A) 1st and 3rd answers are correct
B) 2nd and 7th answers are correct A) 1st and 3rd answers are correct
C) 4th and 5th answers are correct B) 3rd and 4th answers are correct
D) 3rd and 5th answers are correct C) 5th and 6th answers are correct
E) all of the answers are correct D) 3rd and 5th answers are correct
Show comment and answer E) all of the answers are correct
INT - 13.95 Show comment and answer
identify the most dangereous INT - 13.97
complication of your therapy In her case suspicion of thrombophilic
condition shoud be an issue. What is
26 years old pregnant lady, 16 weeks the most like hereditary thrombophilia
of pregnancy arrives to your office. based on statistics?
Progressive swelling and pain
developed in he right femoral region, 26 years old pregnant lady, 16 weeks
four days ago. Pregnancy goes on of pregnancy arrives to your office.
normally otherwise. Doppler Progressive swelling and pain
ultrasound reveals newly developed developed in he right femoral region,
deep vein thrombosis in right femoral four days ago. Pregnancy goes on
vein and popliteal vein. The patient normally otherwise. Doppler
mother and sister alsohad thrombtoic ultrasound reveals newly developed
episodes. deep vein thrombosis in right femoral
vein and popliteal vein. The patient
A) erosion, gastric bleeding mother and sister alsohad thrombtoic
B) herparin induced thrombocytopenia episodes.
C) coumarol-necrosis
A) antithrombin deficiency
D) hypofibrinogenemia
B) protein C deficiency
Show comment and answer
INT - 13.96 activated protein C resistance/F. V.
C)
After the acute thrombotic period Leiden-mutation
which type of anticoagulant D) protein S deficiency
prophylaxis should be applied in this homozygous
E)
case? hyperhomocysteinemia
Show comment and answer
26 years old pregnant lady, 16 weeks
of pregnancy arrives to your office.
INT - 13.98 Show comment and answer
So the most common thrombophilia INT - 14.3
was proved. What prophylactic Which immunosuppressive agent MAY
guideline is recommended in her case? NOT be used for SLE therapy?

26 years old pregnant lady, 16 weeks A) Cyclophosphamide


of pregnancy arrives to your office. B) Methotrexate
Progressive swelling and pain
C) Belimumab
developed in he right femoral region,
four days ago. Pregnancy goes on D) Adalimumab
normally otherwise. Doppler E) Azathioprine
ultrasound reveals newly developed Show comment and answer
deep vein thrombosis in right femoral INT - 14.4
vein and popliteal vein. The patient Which is NOT a complication of
mother and sister alsohad thrombtoic systemic sclerosis?
episodes.
A) Diffuse pulmonary fibrosis
all cases whoi had once DVT B) Honeydew melon stomach
A) lifelong anticoagulant prophylactic C) Esophageal stricture
approach recommended.
D) Pulmonary hypertension
All cases with previous DVT should
E) Fingertip-ulcer
receive couple of years
B) anticoagulant prophylaxis, after Show comment and answer
some years may go for cautious INT - 14.5
dose reduction Which is NOT an ANCA-associated
In all such cases a 6 mont vasculitis?
C)
prophylaxis is enough
A) Polyarteritis nodosa
As all prophylactic efforts are
D) ineffective, their use is not B) Churg-Strauss syndrome
recommended C) Granulomatous vasculitis
D) Microscopic polyangiitis
CLINICAL IMMUNOLOGY E) Wegener granulomatosis
Show comment and answer
INT - 14.1 INT - 14.6
Which is NOT a side-effect of chronic Which antibody is specific for
steroid therapy? Antiphospholipid syndrome?

A) Hypertension A) Anti-apexin V.
B) Myopathy B) Anti-beta2 microglobulin I.
C) Hyperpotassemia C) Anti-beta2 glycoprotein I
D) Cataract D) Anti-cardiotliptin
E) Glaucoma E) Anti-lupus anticoagulant
Show comment and answer Show comment and answer
INT - 14.2 INT - 14.7
Which autoantibody is NOT specific for Which autoantibody is specific for
poly-dermatomyositis? MCTD?

A) Anti-Ku A) Anti-PM-Scl
B) Anti-Mi2 B) Anti-U1 RNP
C) Anti-SRP C) ANCA
D) Anti-Sm D) Anti-SS-A
E) Anti-Jo1 E) Anti-dsDNA
Show comment and answer A) SLE
INT - 14.8
B) Polymyositis
Which is true about immunofluorescent
staining of antinuclear antibody? C) Systemic sclerosis
D) MCTD
A) The staining is nucleolar in SLE E) Sjögren’s syndrome
The staining is homogenous in Show comment and answer
B)
MCTD INT - 14.13
The staining is nucleolar in systemic Which autoimmune disease is NOT
C) commonly associated with B-cell
sclerosis
D) The staining is homogenous in RA lymphoma?
The staining is against centromer in
E) A) SLE
myositis
B) Systemic sclerosis
Show comment and answer
INT - 14.9 C) Rheumatoid arthritis
Which is NOT a polysystemic D) Polymyositis
autoimmune disease? E) Sjögren’s syndrome
Show comment and answer
A) Polymyositis INT - 14.14
B) Systemic sclerosis Which is NOT a symptom of
C) Sclerosis multiplex antiphospholipid syndrome?
D) Sjögren’s syndrome
A) Habitual abortion
E) MCTD
B) Frequent migraine
Show comment and answer
INT - 14.10 C) Thrombocytosis
Which statement is NOT typical in D) Increased risk of thromboembolism
systemic autoimmune diseases? E) Livedo reticularis
Show comment and answer
A) Familiar aggregation INT - 14.15
B)
Sometimes medicines or chemicals Which medicine DO NOT causes SLE?
can trigger them
Antinuclear autoantibodies are A) Etanercept
C)
frequent B) Procainamide
D) Males are affected mainly C) Infliximab
The disease can worsen or improve D) Hydralazine
E)
during pregnancy
E) Rituximab
Show comment and answer
INT - 14.11
Show comment and answer
INT - 14.16
Which genetic constellation is typical in
In what percent of patients with
autoimmune diseases?
undifferentiated autoimmune diseases
(NDC) develops definitive autoimmune
A) HLA-DQ2-DR4
disease?
B) HLA-A1-B8-DR3
C) HLA-A8-B27-DR4 A) 5-10 %
D) HLA-B7-B27-DR3 B) 15-20 %
E) HLA-A1-B7-DR4 C) 30-40 %
Show comment and answer D) 50-60 %
INT - 14.12
E) 70-80 %
Which of the following is the most
frequently occurring systemic Show comment and answer
INT - 14.17
autoimmune disease?
In which case can be NDC diagnosed?
C) Membranous lupus nephritis
In the presence of three typical Mesangial proliferative lupus
A) D)
clinical symptoms nephritis
In the presence of one typical E) Focal minimal lupus nephritis
B) clinical symptom and two
Show comment and answer
autoantibody positivity’s INT - 14.22
In the presence of two clinical In which disease elevated CRP level is
C)
symptoms and two autoantibodies NOT typical?
In the presence of two clinical
D)
symptoms and one autoantibody A) Rheumatoid arthritis
In the presence of one clinical B) Sjögren’s syndrome
E) symptom and three typical C) SLE
autoantibodies
D) Intercurrent infection
Show comment and answer
INT - 14.18 E) Dermatomyositis
Which process is NOT typical in the Show comment and answer
pathogenesis of SLE? INT - 14.23
Which is NOT a typical cause of death
A) DNA repair disorder in SLE?
B) DNA methylation
A) Cardiovascular disease
C) Tissue citrullination
B) Kidney failure
D) Increased B-cell activity
C) Central nervous system lupus
E) Decreased regulatory T-cell activity
D) Pulmonary fibrosis
Show comment and answer
INT - 14.19 E) Secondary tumors
Which biological therapy is appropriate Show comment and answer
for the therapy of SLE? INT - 14.24
Which pathogenetic process DOES NOT
A) Etanercept plays a role in the development of
systemic sclerosis?
B) Infliximab
C) Belimumab
A) Microvascular disorders
D) Adalimumab
B) Immunopathological disorders
E) Tocilizumab
C) Tissue citrullination
Show comment and answer
D) Collagen deposition in the organs
INT - 14.20
Accelerated atherosclerosis and E) Macrovascular disorders
increased cardiovascular mortality are Show comment and answer
NOT typical in which disease? INT - 14.25
Against which antigen is oriented Anti-
A) Rheumatoid arthritis Scl70 in scleroderma?
B) SLE
A) RNA polymerase I
C) Sjögren’s syndrome
B) DNA polymerase I
D) Antiphospholipid syndrome
C) DNA topoisomerase III
E) Systemic sclerosis
D) RNA polymerase III
Show comment and answer
INT - 14.21 E) DNA topoisomerase I
Which is NOT a real histological type of Show comment and answer
lupus nephritis? INT - 14.26
Which is NOT a diagnostic criterion of
A) Diffuse lupus nephritis systemic sclerosis?
B) Sclerotizing glomerulonephritis
A) Sclerodactylia
B) Paroxysmal scleroderma C) Positive Rose-Bengal test
C) Star-shaped scars on the fingertips D) Schirmer-test <5 mm/5 min
D) Bibasilar pulmonary fibrosis E) Positive salivary gland scintigraphy
E) Esophageal dismotility Show comment and answer
Show comment and answer INT - 14.31
INT - 14.27 Which agent is NOT appropriate for
Which medicine is NOT appropriate for treating glandular symptoms of
treating the vascular symptoms of Sjögren’s syndrome?
systemic sclerosis?
A) N-acetylcysteine
A) Nitroglycerin B) Tear point occlusion
B) Calcium antagonists Methylcellulose containing artificial
C)
C) Angiotensin II receptor antagonists tear
D) Beta receptor blockers D) Infliximab
E) Sildenafil E) Pilocarpine
Show comment and answer Show comment and answer
INT - 14.28 INT - 14.32
Which is NOT among the clinic Which disease is NOT an element of
pathological subtypes of inflammatory MCTD?
muscle diseases?
A) Rheumatoid arthritis
A) Adult polymyositis B) NDC
B) Tumor-associated myositis C) Polymyositis
C) Juvenile poly-dermatomyositis D) SLE
D) Inclusion body myositis E) Systemic sclerosis
E) Ossification myositis Show comment and answer
INT - 14.33
Show comment and answer
INT - 14.29 Which disease constitutes diagnostic
Which is NOT among the diagnostic criteria for MCTD according to Alarcon-
criteria of inflammatory muscular Segovia?
diseases according to Bohan and
Peter? A) Pulmonary fibrosis
B) Swelling of the back of the hand
Elevated levels of CK and LDH C) Pulmonary arterial hypertension
A)
enzymes D) Esophageal dismotility
Intensifying weakness of distal E) Swelling of the fingers
B)
muscles
Show comment and answer
Positive muscle biopsy with INT - 14.34
C)
inflammation, necrosis INT-14.35. Which systemic vasculitis
D) EMG disorders affects the medium-sized vessels?
Skin symptoms specific for
E)
dermatomyositis A) Churg-Strauss syndrome
Show comment and answer B) Takayasu arteritis
INT - 14.30
C) Kawasaki disease
Which is NOT among the American-
D) Cryoglobulinemic vasculitis
European Consensus Criteria for
Sjögren’s syndrome? E) Polymyalgia rheumatica
Show comment and answer
A) Anti-fodrin antibody INT - 14.35
Which systemic vasculitis affects the
Anti-SS-A and/or anti-SS-B
B) medium-sized vessels?
antibody
A) Churg-Strauss syndrome B) 2. and 4. are correct
B) Takayasu arteritis C) only 5. is correct
C) Kawasaki disease D) 1., 3. and 5. are correct
D) Cryoglobulinemic vasculitis E) all answers are correct
E) Polymyalgia rheumatica Show comment and answer
INT - 14.39
Show comment and answer
INT - 14.36 Diagnostic criteria of Sjögren’s
Histopathological forms of lupus syndrome according to the American-
nephritis: European Consensus system:

1) Focal lupus nephritis 1) Recurrent foreign body sensation


2) Membranous lupus nephritis in the eyes
3) Sclerotizing glomerulonephritis 2) Positive ultrasound of salivary
4) Minimal mesangial nephritis glands
5) Diffuse lupus nephritis 3) Dry eyes for more than 3 months
4) Schirmer-test <3mm/3min
A) 1. and 3. are correct 5) Positive sialography
B) 2. and 4. are correct
A) 1. and 3. are correct
C) only 5. is correct
B) 2. and 4. are correct
D) 1., 3. and 5. are correct
C) only 5. is correct
E) all answers are correct
D) 1., 3. and 5. are correct
Show comment and answer
INT - 14.37
E) all answers are correct
Classification criteria of SLE: Show comment and answer
INT - 14.40
1) Iron deficiency anemia B-cell inhibitor appropriate for therapy
2) ANA positivity of SLE:
3) Anti-ENA antibody
4) Leukopenia <4G/l 1) Rituximab
5) Lymphocytosis 2) Tocilizumab
3) Certolizumab pegol
A) 1. and 3. are correct 4) Infliximab
5) Belimumab
B) 2. and 4. are correct
C) only 5. is correct A) 1. and 3. are correct
D) 1., 3. and 5. are correct B) 2. and 4. are correct
E) all answers are correct C) only 5. is correct
Show comment and answer D) 1., 3. and 5. are correct
INT - 14.38
Classification criteria of E) all answers are correct
antiphospholipid syndrome: Show comment and answer
INT - 14.41
1) Vascular thrombosis Classification criteria of Henoch-
2) Anti-cardiolipin IgA positivity Schönlein purpura:
3) Abortion at or before 10th week
of pregnancy 1) Palpable purpura
4) Premature birth before the 20th 2) ANCA positivity
week 3) Abdominal angina
5) Lupus anticoagulant positivity 4) Lymphocytes in the wall of small
twice, at least 12 weeks interval vessels
between the two examinations. 5) Age <21 years at the beginning
of the disease
A) 1. and 3. are correct
A) 1. and 3. are correct 5) Rituximab
B) 2. and 4. are correct
A) 1. and 3. are correct
C) only 5. is correct
B) 2. and 4. are correct
D) 1., 3. and 5. are correct
C) only 5. is correct
E) all answers are correct
D) 1., 3. and 5. are correct
Show comment and answer
INT - 14.42 E) all answers are correct
Which is true? Show comment and answer
INT - 14.45
1) Wegener-granulomatosis is a Organ symptoms of systemic sclerosis:
vasculitis affecting the small vessels
2) Polyarteritis nodosa is an ANCA- 1) Pulmonary arterial hypertension
associated vasculitis 2) Apical pulmonary fibrosis
3) Microscopic polyangiitis is an 3) Watermelon-stomach
ANCA-associated vasculitis 4) Discoid skin symptoms
4) Takayasu-arteritis affects the 5) Subcutaneous calcinosis
medium-sized vessels
5) Churg-Strauss syndrome is not A) 1. and 3. are correct
ANCA-associated B) 2. and 4. are correct
C) only 5. is correct
A) 1. and 3. are correct D) 1., 3. and 5. are correct
B) 2. and 4. are correct E) all answers are correct
C) only 5. is correct

D) 1., 3. and 5. are correct
INTENSIVE THERAPY, EM MED,
E) all answers are correct
TOXICOLOGY
Show comment and answer
INT - 14.43

INT - 15.1
Which are NOT a classification criterion
The current 2015 European
of granulomatous vasculitis:
Resuscitation Council (ERC) adult Basic
1) Granulomatous inflammation in Life Support (BLS) guideline advises
the wall of small vessels the procedures in the following order:
2) ANCA positivity
3) Fix infiltration or nodules in the unresponsive – no pulse detected –
lungs resuscitation/emergency team
A)
contacted – 30 chest compressions
4) Chronic sinusitis
– 2 breaths, 30 chest compressions
5) Micro hematuria
unresponsive – call for help –
A) 1. and 3. are correct secure the airway – no breathing
detected – resuscitation/emergency
B) 2. and 4. are correct B)
team contacted – 30 chest
C) only 5. is correct compressions – 2 breaths, 30 chest
D) 1., 3. and 5. are correct compressions
E) all answers are correct unresponsive – call for help –
Show comment and answer secure the airway – no breathing
INT - 14.44 detected – resuscitation/emergency
C)
Appropriate medicines for therapy of team alerted – 15 chest
ANCA-associated vasculitides: compressions – 2 breaths, 15 chest
compressions
1) Cyclophosphamide unresponsive – call for help –
2) Azathioprine secure the airway – no breathing
D)
3) Sulfamethoxazole detected – resuscitation/emergency
4) Methotrexate team alerted – 2 breaths, 30
compressions – 2 breaths, 30 Targeted temperature management
compressions is advised most strongly after
D)
Show comment and answer cardiac arrest with a shockable
INT - 15.2 rhythm.
The current 2015 European Show comment and answer
Resuscitation Council (ERC) adult Basic INT - 15.5
Life Support (BLS) guideline permits During cardiopulmonary resuscitation
cessation of chest compressions if the (CPR), if no high-risk intervention (e.g.
following occurs: Intravenous cannulation) is performed,
what are possible transmittable
A) the defibrillator is charging diseases or agents?
B) pulse is detected
Staphylococcus aureus,
the patient start moving, breathing,
C) A) Streptococcus pyogenes, SARS,
opens eyes
meningococcal meningitis
D) if the AED does not advise shock
B) CMV
Show comment and answer
INT - 15.3
C) HIV, HBV, HCV
Which of the following statements is D) Mycoplasma, Chlamydia
false regarding chest compressions Show comment and answer
(according to the 2015 European INT - 15.6
Resuscitation Council Adult Basic Life When assessing a patient with the
Support guideline)? ABCDE algorithm, the letters refer to
the following, except:
optimal frequency of chest
A) Acute assessment of
compressions is 100-120/min A)
depth of compressions should be 5- consciousness
B) B) Breathing assessment
6 cm in adults
position of chest compressions: C) Circulation assessment
C) middle of the chest, lower half of D) Disability (neurological) assessment
sternum E) Exposure assessment
CPR providers should be changed Show comment and answer
over after 5 minutes, because INT - 15.7
D)
fatigue decreases compression Myocardial oxygen supply is influenced
quality by the following, except:
Show comment and answer
INT - 15.4
A) diastolic time
The following are all part of the 2015
B) arterial partial oxygen pressure
European Resuscitation Council (ERC)
adult Advanced Life Support guideline, C) hemoglobin concentration
except for one: D) vagal tone
E) diastolic blood pressure
Check carotid pulse immediately Show comment and answer
A)
after delivering shock. INT - 15.8
When treating VF/pulseless VT A 32-year-old male is brought in to the
adrenaline 1mg should be given emergency room by relatives after
after the 3. shock, after suffering an electric shock, he is
B) compressions have been resumed, unconscious and breathing. Which one
the dose can be repeated after 3-5 of the following actions is necessary?
minutes (in alternating cycles of
CPR). intubation if Glasgow Coma Scale is
Routine use of atropine is not A)
below 8
C) advised in asystole and PEA B) securing a central line
(pulseless electrical activity).
defibrillation with 200J if asystole is When deciding on a transfusion,
C)
present A) universal trigger hemoglobin (Hb)
D) echocardiography levels should be used.
Show comment and answer Transfusion is seldom indicated if
B)
INT - 15.9 Hb>10g/dL.
A patient with pneumonia treated in Transfusion is always indicated if
the ICU with mechanical ventilation C)
Hb<6g/dL.
develops increasing tachycardia, If normovolemia is present, as
decreasing blood pressure on the fifth D) much as an 80% red blood cell loss
treatment day. Which of the following is survivable.
is not the probable cause? Show comment and answer
INT - 15.13
A) septic shock In regards to the oxygen-hemoglobin
B) anaphylactic shock dissociation curve, the following are
normal hemodynamic true except:
C)
consequences of arousal reaction
consequences of acute cardiac Increased pCO2 shifts the curve to
D) A)
ischemia the right.
Show comment and answer Decreased 2,3-disfosfoglycerate
INT - 15.10 B) (2,3 DGP) concentration shifts the
53-year-old female is admitted with curve to the left.
acute right sided hemiplegia, Decreased proton concentration
C)
deteriorating consciousness, shift the curve to the left.
respiratory distress, hypertension to Increased FiO2 (inspired oxygen
D)
the ICU with symptom onset in less fraction) shifts the curve to the right
than an hour. She is intubated and Show comment and answer
ventilated. Which of the following is INT - 15.14
incorrect? In the treatment of an unconscious
patient with severe hemorrhagic
A) Acute head CT scan is indicated. shock, the first and most important
Head of the bed should be elevated step is:
B)
to 30°.
Goal of ventilation is controlled transfuse with compatible red blood
C) A)
hypercapnia. cell
Thrombolysis with rt-PA (iv. place a central line to replace fluid
D) 0,9mg/kg in 60 minutes) might be B) and monitor central venous
indicated. pressure
Show comment and answer rapid infusion of crystalloids and/or
C)
INT - 15.11 colloids
What is the primary goal of packed red D) secure the airway
blood cell transfusion? Show comment and answer
INT - 15.15
A) Restoring intravascular volume. According to current guidelines, when
B) Normalizing cardiac output. is acute percutaneous coronarography
C) Improving tissue perfusion. indicated in ST elevation myocardial
infarction? (ESC STEMI guideline 2012)
D) Improving oxygen delivery.
Show comment and answer Typical chest pain and ST elevation
INT - 15.12
A) or probable new onset left bundle
Which of the following is false branch block (LBBB).
regarding transfusions?
Typical chest pain and wall motion
B)
abnormality on echocardiography.
Typical chest pain or LBBB and C) respiratory rate > 25/min
C)
elevated ectoenzymes. WBC > 12 000/mm3 or <
ST elevation or probable new onset D) 4000/mm3 or > 10% immature
LBBB, elevated ectoenzymes and neutrophils
D)
wall motion abnormality on Show comment and answer
echocardiography. INT - 15.20
Show comment and answer A 65-year-old male is admitted to the
INT - 15.16 ICU with community acquired
Most effective way to diagnose cardiac pneumonia. After 30mL/kg Ringer
tamponade: infusion, mechanical ventilation
initiation, microbiological sampling and
A) Electrocardiogram initiation of fluoroquinolone therapy his
B) Echocardiography vitals are as follows: blood pressure
C) Chest X-ray. 85/40 mmHg, heart rate 110/min,
SatO2 96%, respiratory rate: 28/min,
D) Right heart catheterization
T: 39,3C. By exact definition, he
Show comment and answer suffers from
INT - 15.17
All of the statements regarding
A) sepsis
succinylcholine use are true except for
one. Mark the false statement: B) severe sepsis
C) septic shock
Succinylcholine is contraindicated in D) ventilator associated pneumonia
A) hyperkalemia, burn patient, Show comment and answer
paraplegia. INT - 15.21
Side effects include muscle pain, According to the Surviving Sepsis
B) bradycardia, malignant Campaign the following should be
hyperthermia. achieved within an hour of the initial
Effects of succinylcholine can be treatment of a septic patient:
C)
counteracted with neostigmine.
Succinylcholine can be used for the microbiological sampling and
D) intubation of patients with a full A) starting of adequate antibiotic
stomach. treatment
Show comment and answer identifying the microorganism by
B)
INT - 15.18 PCR
Which of the following is not part of C) 2000mL iv. crystalloid
first line treatment for anaphylaxia? D) surgical source control
Show comment and answer
A) oxygen INT - 15.22
B) infusion therapy Pharmacological treatment of a patient
C) vasoactive treatment with septic shock includes:
D) antihistamines
Show comment and answer Vancomycin, if shock has been
A)
INT - 15.19 present for more than two days
Which of the following are SIRS high dose methylprednisolone, if
B)
(Systemic Inflammatory Response shock in unresponsive to iv. fluids
Syndrome) criteria (as per American insulin, if blood glucose is
C)
College of Chest Physicians and >10mmol/L
Society of Critical Care Medicine D) dobutamine, if tachycardia persists
ACCP/SCCM 1991 Consensus)? Show comment and answer
INT - 15.23
A) temperature > 37°C The goal of fluid resuscitation in a
B) heart rate > 80/min septic patient is:
A) CVP > 20 mmHg oxygen therapy does not improve
B) MAP > 65 mmHg A) outcomes in chronically hypoxic
patients
C) diuresis > 1,5mL/kg/h
oxygen therapy reduces respiratory
D) ScvO2 > 30% B)
drive
Show comment and answer
INT - 15.24
oxygen therapy results in
C) CO2 retention because of
Nutrition goal for a patient treated with
diminished hypoxic vasoconstriction
urosepsis and septic shock includes:
anaerobe metabolism is more
D)
A) Enteral nutrition. energy efficient
B) Parenteral nutrition. Show comment and answer
INT - 15.28
C) Reduced protein enteral nutrition. NIV (noninvasive ventilation) is an
Enteral nutrition with parenteral important treatment option in acute
D) supplementation if goal calorie exacerbation of chronic obstructive
intake is not reached within day 2. pulmonary disease (COPD) with the
Show comment and answer right indications because:
INT - 15.25
23-year-old female is admitted to the it has fewer complications than
ICU because of asthma. Invasive A)
invasive ventilation
mechanical ventilation is started. it results in less mucus dehydration
Which of the following is not true B)
than invasive ventilation
regarding initial ventilation settings?
with appropriate settings 80% of
C)
patients report improving symptoms
Minimizing dynamic hyperinflation is
A) key, so expiratory time should be D) it unloads respiratory muscles
long and PEEP should be low. Show comment and answer
INT - 15.29
High inspiratory pressures should
The following is not true regarding
be avoided at all costs, since
B) acute kidney injury (AKI):
pressure correlates with barotrauma
and mortality.
origin in intensive care is usually
FiO2 should be adjusted to reach a A)
C) prerenal
SatO2 >94%
intraabdominal pressures above 25
Minute ventilation should be as low B)
mmHg promote AKI
D) as possible to minimize dynamic
hyperinflation. high dose diuretic therapy can stop
C)
progression of AKI
Show comment and answer
INT - 15.26 NSAID therapy can promote
D)
Which of the following blood gas values progression of AKI
is most indicative of acute respiratory Show comment and answer
failure in a patient suffering from acute INT - 15.30
exacerbation of COPD? A mechanically ventilated,
hemodynamically stable and
A) pH < 7,3 normovolemic 24-year-old polytrauma
B) paO2 < 60 mmHg patient has the following parameters
on day 4. following trauma: serum
C) paCO2 > 50 mmHg
creatinine 400 umol/L; diuresis:
D) HCO3 > 30 mmHg 30ml/h, pH 7,3; BE: -11, HCO3:
Show comment and answer 13mmol/L. Which of the following is
INT - 15.27 advised first?
In the treatment of acute exacerbation
of COPD oxygen therapy is carefully A) 1-5ug/kg/min dopamine drip
titrated because:
combination of osmotic and loop
B)
diuretics
at least 2L positive fluid balance to 30/min frequency wide QRS, regular
C)
improve hydration rhythm without P waves. BP
D) renal replacement therapy 70/30Hgmm, SatO2 98%, GCS: 3-5-6.
Show comment and answer The following is advised:
INT - 15.31
When treating a patient with acute A) start BLS
kidney failure in intensive care which B) synchronized DC shock
of the following is true: C) transcutaneous pacing
D) give atropine
antibiotic dose should be adjusted
A) Show comment and answer
according to renal clearance.
INT - 15.35
Dose of LMWH (low molecular A patient with hyperactive nodular
weight heparin) treatment should be goiter is admitted to the ICU 12 hours
B)
increased because of increased after an elective partial thyroidectomy
thrombosis risk. with hyperpyrexia, weakness,
Parenteral nutrition should be confusion, vomiting, atrial fibrillation
C) protein free due to the risk of and hypotension. The most probable
protein intoxication. cause and adequate treatment are as
D) All of the above. follows:
Show comment and answer
INT - 15.32 Malignant hyperthermia; Dantrolene
64-year-old male is admitted to the A)
and supportive therapy.
emergency room with repeated Wound infection and sepsis; iv.
occurrence of syncope. Loss of fluids, microbiological sampling,
consciousness occurs again while B)
vasopressor, combination antibiotic
circulation and ventilation remain therapy.
intact, the monitor shows a 220/min Thyreotoxic crisis; iv. fluid therapy,
frequency regular, narrow QRS methimazole or propylthiouracil,
rhythm. The following is indicated: C)
propranolol, methylprednisolone,
plasma exchange if needed.
A) start BLS Thyreotoxic crisis; reoperation to
D)
B) start amiodarone treatment eliminate residual thyroid tissue.
C) perform synchronized DC shock Show comment and answer
perform ablation therapy in INT - 15.36
D) Acute pancreatitis has a poor prognosis
electrophysiology lab
if
Show comment and answer
INT - 15.33
64-year-old patient with palpitations is A) hematocrit decreases
admitted to the emergency room, the B) Ranson score < 3
monitor shows a 220/min frequency, C) Ca level increases
regular, narrow QRS rhythm. BP is D) the patient is young
130/70mmHg, SatO2 100%, GCS: 4-5-
Show comment and answer
6. The following is advised: INT - 15.37
A 24-year-old male with no previous
A) start BLS medical conditions is admitted to the
B) amiodarone loading ICU with dyspnea, general weakness
C) perform synchronized DC shock after a history of symptoms of
D) adenosine therapy respiratory infection. T: 38,5C, BP:
120/70Hgmm, HR: 120/min, SatO2:
Show comment and answer
INT - 15.34 88%, on 4L/min nasal O2. Chest X ray
64-year-old male is admitted to the shows bilateral homogenous effusions.
emergency room because of weakness What is the first line ventilation
and fainting, the monitor shows a strategy?
INT - 15.41
Immediate active respiratory Which of the following is incorrect
A) physiotherapy to increase mucus regarding the treatment of
clearance pneumothorax (ptx)?
Invasive ventilation with high PEEP
B) Asymptomatic, iatrogenic, 1cm
and volume controlled mode
Noninvasive ventilation in CPAP wide, apical position pneumothorax
C) A)
mode. developed after pleural tap does not
need immediate drainage
Invasive ventilation with high PEEP
D) Tension pneumothorax does not
and pressure support mode. B)
always need to be drained.
Show comment and answer
INT - 15.38 Positive pressure ventilation may
Which of the following promotes C) worsen existing pneumothorax,
promoting tension pneumothorax.
postoperative respiratory failure after
abdominal surgery? Reexpansion pulmonary edema
and hypotension is most often
Combined general and regional D) associated with drainage of total
A) pneumothorax that has been
anesthesia during the operation
present for a longer time.
B) Early postoperative mobilization
Show comment and answer
Smoking cessation in the INT - 15.42
C)
preoperative period (1-2 week prior) Which of the following is true regarding
D) Ongoing epidural analgesia the treatment of acute pancreatitis?
Show comment and answer
INT - 15.39 In case of acute, severe pancreatitis
Which of the following is not correct caused by a biliary stone, ERCP
regarding the treatment of pulmonary A) (endoscopic retrograde
embolism? cholangiopancreatography) is
advised within 72 hours.
Verified PE indicates Nasogastric tube is needed
A) B)
anticoagulation permanently.
Submissive PE warrants Epidural analgesia might mask the
B)
thrombolysis treatment. symptoms of acute abdomen, so
C)
Massive PE with a known use in severe pancreatitis is not
C) thrombolysis contraindication advised.
warrants surgical embolectomy. Because of severe pain nonstreoid
Vena cava inferior filter is indicated D) analgesics need to be
in acute PE (or proximal deep supplemented with morphine.
D)
venous thrombosis) if Show comment and answer
anticoagulation is contraindicated. INT - 15.43
Show comment and answer Which of the following is incorrect
INT - 15.40 regarding the treatment of diabetic
Which of the following is incorrect ketoacidosis?
regarding types of respiratory failures?
Fluid resuscitation started
type I respiratory failure is caused immediately helps moderate
A) contraregulating hormone
by shunt or diffusional damage
B) type II respiratory failure can be A) (nor/epinephrine, glucagon, growth
B) hormone, cortisol) release,
caused by myasthenia gravis
improved renal blood flow and
type III respiratory failure includes promotes glucose clearance
C)
both hypoventilation and shunt
Metabolic acidosis needs to be
type IV respiratory failure presents corrected according to the following
D) B)
in ARDS equation: NaHCO3 mmol = -BE x
Show comment and answer 0,3 x kg
Insulin drip is advised in a dose of
C)
0,1E/kg/hour Parenteral nutrition is advised if
Potassium replacement might be A) enteral nutrition is not possible
needed in doses as high as 0,5 within 3 days.
D)
mmol/kg/hour (2,6g/hour of KCl for 900 mOsm/l osmolarity infusion can
a 70-kg patient) B) be administered through a
Show comment and answer peripheral line.
INT - 15.44 Minimal carbohydrate need is 2g/kg
C)
The following is not true regarding glucose.
therapy of hepatic coma. Parenteral nutrition needs to be
D) supplemented with trace elements
cerebral edema is the most and vitamins daily.
A)
common cause of death Show comment and answer
increased prothrombin time,
B) metabolic acidosis predicts Show comment and answer
unfavorable outcome INT - 15.47
Nutrition needs to mainly include What is the goal blood glucose level for
C) critically ill patients?
aromatic amino acids and glucose.
30-45 ml lactulose is advised 3
D) times daily per os or through NG A) 4-5 mmol/L
tube B) 4-6 mmol/L
Show comment and answer C) 6-8 mmol/L
INT - 15.45 D) <10 mmol/L
22-year-old, previously healthy female
Show comment and answer
lost a significant amount of blood INT - 15.48
(about 700ml) during labor, bleeding A patient admitted because of chest
has been stopped and currently there pain radiating to the left arm and
is no further blood loss. Hb is 95g/L, dyspnea has the following vitals: blood
Hct is 35%, blood pressure is 100/70 pressure 90/60, heart rate: 110/min,
mmHg, heartrate 120/min, SatO2 SatO2: 93%, with bilateral rails over
98%, circulation is centralized, the lungs. Which of the following is not
capillary refill time is 5 secs, indicated?
peripheries are cold, jugular vein
collapsed, oliguria is present. Which
A) Morphine 2-4mg iv.
management is advised?
B) Oxygen 2-6 l/min
Currently no treatment is needed, C) Nitrate drip 1mg/h iv.
the patient is young with no D) Aspirin 100mg po.
A)
comorbidities, bleeding will be Show comment and answer
compensated. INT - 15.49
Blood loss warrants about 2 units of In the following condition noninvasive
B)
typed red blood cell transfusions. rather than invasive ventilation is
2000ml isotonic, balanced advised:
crystalloid therapy is needed with
C)
further therapy based on monitored hypercapnic respiratory failure due
parameters. A) to COPD acute exacerbation
2000ml normal saline is needed caused by bronchitis
D) with further therapy based on symptoms of cardiac
monitored parameters. B) decompensation after a successful
Show comment and answer 20-minute-long CPR
INT - 15.46 respiratory failure manifesting on
Which of the following is incorrect C) the 2. postoperative day after large
regarding the nutritional therapy of abdominal surgery
intensive care patients? D) ARDS
Show comment and answer elective hernia repair under
INT - 15.50 inhalation anesthesia.
A patient on ICU with a history of Endotracheal tube, ventilation for a
ischemic heart disease has been 65-year-old patient with COPD
ventilated because of pneumonia for a C) undergoing elective laparoscopic
week. Newly developed abdominal cholecystectomy under iv.
expansion, tenderness and diffuse pain anesthesia
might be a consequence of: D) All of the above
Show comment and answer
A) calculous cholecystitis INT - 15.54
B) ischemic colitis An hour following extubation of a
C) gastric perforation female ventilated for 12 days for
D) all of the above exacerbation of COPD, tachycardia,
Show comment and answer dyspnea, agitation and disorientation
INT - 15.51 develop. What is the most probable
Spinal anesthesia is ideal for inguinal diagnosis and treatment?
hernia repair, if
A) intravasal hypovolemia, iv. fluids
A) the hernia is incarcerated persistent respiratory failure,
B)
B) the patient has COPD reintubation
the patient has significant aortic persistent respiratory failure,
C) C)
stenosis noninvasive ventilation (NIV)
D) the patient has mental retardation D) delirium, haloperidol therapy
Show comment and answer Show comment and answer
INT - 15.52 INT - 15.55
The following anesthesia plans are Tracheostomy is beneficial with
adequate for the following conditions: protracted ventilation for the following
reasons:
Thiopental-fentanyl iv. anesthesia
for a gynecological operation of a A) per os nutrition is possible
A)
patient with a history of asthma B) mobilization is easier
bronchiale intermittent ventilatory breaks can
C)
Inhalational sevoflurane induction be administered
B) and maintenance for abdominal D) all of the above
exploration for ileus
Show comment and answer
Etomidate-fentanyl iv. induction and INT - 15.56
sevoflurane maintenance for valve A 17-year-old female with no previous
C)
replacement operation for a cordal medical history has been brought to
rupture causing cardiogenic shock the ER by EMT. The patient was found
D) All of the above lying in bed with an empty bottle of
Show comment and answer alprazolam and vodka bottles. At
INT - 15.53 admittance, respiratory arrest is
Adequate methods for maintaining a observed and the patient is intubated.
secure airway in the following What are the probable initial blood gas
procedures are: values?

Oropharyngeal airway, bag-valve- A) metabolic acidosis


mask ventilation for a 65-year-old B) acute respiratory alkalosis
A) female with hypertension,
paroxysmal atrial fibrillation during C) chronic respiratory acidosis
narcosis for elective cardioversion D) acute respiratory acidosis
Laryngeal mask airway, ventilation Show comment and answer
B) INT - 15.57
for a 3-year-old girl undergoing
Which of the following is true for pH 7,46; pO2 60 mmHg; pCO2 58
C)
assist/controlled volume controlled mmHg; HCO3 11 mmol/L; BE -12
ventilation? pH 7,37; pO2 65 mmHg; pCO2 59
D)
mmHg; HCO3 27 mmol/L; BE +4
Inspiratory time is controlled by the Show comment and answer
A)
ventilator and is always the same INT - 15.61
Inspiratory pressure is constant Which of the following does not cause
B)
throughout inspiration high anion gap (AG>16mmol/L)
All breaths are triggered by the metabolic acidosis?
C) patient and therefor there can be
longer periods of apnea large amount of normal saline
A)
Assisted breaths tend to have infusion
D) smaller tidal volumes than B) uremia
controlled breaths C) diabetic ketoacidosis
E) null D) ethylene glycol intoxication
Show comment and answer Show comment and answer
INT - 15.58 INT - 15.62
Hypokalemic, hyperchloremic Arterial blood gas is as follows: pH 6,9,
metabolic alkalosis following pO2 89 mmHg, pCO2 18 mmHg,
gastrointestinal surgery is most often a HCO3 9 mmol/L, BE -18. Anion gap is
result of what? 22mmol/L. Which of the following is
not a likely diagnosis?
A) nutritional therapy
B) acute renal failure A) pancreatic fistula
C) diarrhea B) methanol intoxication
gastric paralysis, prolonged NG C) hepatic failure, lactate accumulation
D)
tube placement D) diabetic ketoacidosis
Show comment and answer Show comment and answer
INT - 15.59 INT - 15.63
Which of the following blood gas values A 37-year-old patient with a history of
are characteristic for a 65-year-old chronic pancreatitis, alcoholism is
smoker suffering from chronic brought in with decreased
bronchitis? consciousness and cardiorespiratory
insufficiency, having been intubated
pH 7,46; pO2 98 mmHg; pCO2 28 and ventilated. Blood pressure 80/40
A)
mmHg; HCO3 24 mmol/L; BE -0,2 mmHg; heart rate 118/min, capillary
pH 7,37; pO2 65 mmHg; pCO2 59 refill time: 3sec. Arterial blood gas
B)
mmHg; HCO3 27 mmol/L; BE +4 values: pH 6,9, pO2 89 mmHg,
pH 7,28; pO2 98 mmHg; pCO2 28 pCO2 18 mmHg, HCO3 9 mmol/L, BE -
C)
mmHg; HCO3 11 mmol/L; BE -12 18. Anion gap: 22mmol/L. The patient
pH 7,46; pO2 60 mmHg; pCO2 58 is oligo-anuric. Urine sample is
D)
mmHg; HCO3 11 mmol/L; BE -12 negative for ketone bodies. Which of
Show comment and answer the following is not indicated?
INT - 15.60
Which of the following seems to be a A) toxicology studies
false blood gas read due to sampling iv. fluid therapy (crystalloids,
mistake? B)
colloids)
C) dobutamine therapy
pH 7,28; pO2 98 mmHg; pCO2 28
A) D) renal replacement therapy
mmHg; HCO3 11 mmol/L; BE -12
Show comment and answer
pH 7,46; pO2 98 mmHg; pCO2 28
B)
mmHg; HCO3 24 mmol/L; BE -0,2
INT - 15.64 Show comment and answer
What isn’t an advantage of continuous INT - 15.68
renal replacement therapy over Which of the following is incorrect
intermittent therapy in critically ill, regarding lung transplantation?
hemodynamically unstable patients?
Donor management influences
A)
Circulatory stability is better outcome of lung transplantation.
A)
achieved. Average survival time after lung
B)
B) Daily costs are lower. transplantation is 15 years.
Permanent negative fluid balance is Organ allocation in Eurotransplant
C) C) is based on LAS (lung allocation
better achieved.
There’s a better chance for kidney score).
D) Initiation of immunosuppression is
recovery. D)
Show comment and answer indicated perioperatively.
INT - 15.65 Show comment and answer
Which of the following is not a INT - 15.69
probable cause of mechanical The following is true concerning
ventilation weaning failure? epidural analgesia:

A) Dystelectasis. Insertion of the epidural catheter


B) Cardiac decompensation. A) can be done through a Touhy
needle.
C) Muscle weakness and fatigue.
Epidural catheter can only be
D) Increased FRC. B)
inserted below the level of L3-4.
Show comment and answer Effective epidural analgesia
INT - 15.66 C)
involves motor blockade.
Which of the following statements is
Epidural analgesia can be used for
incorrect regarding premedication? D)
a maximum of 48 hours.
Oral anticoagulation therapy should Show comment and answer
INT - 15.70
A) be stopped 5 days before large
abdominal surgery. Which of the following is incorrect
regarding infusions?
Insulin dependent diabetic patients
B) need to be switched to oral therapy
perioperatively. Normal saline may cause
A) hyperchloremic acidosis in
Long term beta blocker therapy excessive amounts.
C) should be continuous
perioperatively. Balanced crystalloids are safer for
B)
rehydration.
Discontinuation of antiepileptic
D) therapy perioperatively is Hydroxy-ethyl starch solutions are
dangerous. C) recommended in sepsis,
dehydration and severe trauma.
Show comment and answer
INT - 15.67 Distribution of infusions inside the
The following statement is true D) body depends on their oncotic and
osmotic qualities.
regarding preoperative fasting:
Show comment and answer
INT - 15.71
Breast milk can be consumed up to
A) Regarding transfusions in critically ill
2 hours before surgery.
patients, the following is true:
Water can be consumed up to an
B)
hour before surgery.
Transfusion is indicated below a
Solid food can be consumed up to 4 A)
C) hemoglobin level of 10g/dL.
hours before surgery.
Inotropic therapy is indicated below
Tea can be consumed up to 2 hours B)
D) a hemoglobin level of 7g/dL.
before surgery.
Plasma transfusion should be 3) calcium
C) performed simultaneously in 1:1 4) glucose-insulin-potassium
ratio. infusion
Transfusion is indicated below a 5) forced diuresis
D) 6) β-agonist inhalation
hemoglobin level of 7g/dL.
Show comment and answer
INT - 15.72 1st, 2nd and 4th answers are
A)
Hyperdynamic shock is characterized correct
all of these except: B) 1st, 3rd and 6th answers are correct
C) 3rd and 5th answers are correct
A) Bradycardia D) 1st answer is correct
B) Hypotension Show comment and answer
C) Oliguria INT - 15.76
D) Warm extremities Which of the following is correct
Show comment and answer concerning thrombolysis?
INT - 15.73
Which of the following is incorrect 1) In acute myocardial infarction
regarding paracetamol intoxication? thrombolysis is indicated within 12
hours if PCI is not available.
Hepatic phase starts 1-4 days after 2) In acute stroke thrombolysis is
ingestion with vomiting, jaundice, indicated within 3 hours if bleeding can
A) altered consciousness, be ruled out.
hypoglycemia and kidney 3) Thrombolysis is indicated in
dysfunction. massive pulmonary embolism.
Diagnosis is based on symptoms 4) Thrombolysis is contraindicated
B) since serum level cannot be in ages > 75 years.
measured. 5) Menstrual bleeding is a
Active charcoal within 1 hour of contraindication for thrombolysis.
C) 6) Platelet count below 100 G/L is a
intoxication can improve outcome
contraindication for thrombolysis in
N-acetylcysteine works as an
D) ischemic stroke.
antidote for paracetamol.
Show comment and answer 1st, 2nd, 3rd and 6th answers are
INT - 15.74 A)
correct
Which of the following is true for
regarding parenteral nutrition? B) 1st, 3rd and 6th answers are correct
C) 3rd and 5th answers are correct
Parenteral nutrition infusions only D) 1st, 4th and 5th answers are correct
include macronutrients, so they Show comment and answer
A)
need to be supplemented with INT - 15.77
vitamins and micronutrients. Which of the following are indicative of
It can safely be administered pulmonary embolism?
B)
through central venous lines.
It increases the chance for 1) new onset right bundle branch
C) block
nosocomial infections.
D) All of the above. 2) infarction pneumonia on chest X-
ray
Show comment and answer
3) new onset negative T waves in
INT - 15.75
V1–3
Which of the following is part of
4) elevated pro-BNP
emergency treatment of
5) perfusion defect on
hyperkalemia?
ventilation/perfusion pulmonary
1) hemodialysis scintigraphy
2) plasmapheresis 6) reduced alveolo-areterial oxygen
difference
5) reduced functional residual
1st, 2nd, 3rd and 6th answers are capacity
A)
correct 6) protein rich pulmonary edema
B) 1st, 3rd and 6th are answers correct
1st, 3rd, 4th and 5th answers are A) 1st and 3rd answers are correct
C)
correct 2nd, 4th and 5th answers are
B)
2nd, 3rd, 4th and 5th answers are correct
D)
correct 1st, 3rd, 4th and 6th answers are
C)
Show comment and answer correct
INT - 15.78 D) all of the answers are correct
Which of the following can lead to
distributive shock?
CLINICAL GENETICS
1) sepsis
2) thyrotoxicosis INT - 16.1
3) burn Among the following monogenic
4) left ventricle aneurysm rupture diseases predisposing for
5) cardiac tamponade tumorigenesis which one is not
6) diabetic ketoacidosis autosomal dominantly inherited?

1st, 2nd and 6th answers are A) multiple endocrine neioplasia type 1
A)
correct B) von Hippel–Lindau disease
B) 1st, 3rd and 6th answers are correct C) ataxia teleangiectasia
C) 3rd and 6th answers are correct D) multiple endocrine neioplasia type 2
D) 1st, 3rd and 5th answers are correct E) neurofibromatosis type 1
Show comment and answer Show comment and answer
INT - 15.79 INT - 16.2
Severe aspirin intoxication can cause: At which pregnancy age is
amniocentesis recommended?
1) high anion gap metabolic acidosis
2) hypoglycemia A) 4–6 weeks
3) hypoprothrombinemia B) 10–12 weeks
4) tinnitus
C) 15–17 weeks
5) one marrow depression
6) respiratory alkalosis D) 20–22 weeks
E) 24–26 weeks
1th, 2nd and 4th answers are Show comment and answer
A) INT - 16.3
correct
B) 3rd and 5th answers are correct Among the following chromosome
abnormalities which one is the most
1st, 2nd, 4th and 5th answers are
C) frequent?
correct
D) 2nd and 6th answers are correct
A) Turner-syndrome
Show comment and answer
B) Edwards-syndrome
INT - 15.80
Which of the following is true for C) Down-syndrome
ARDS? D) Patau-syndrome
E) Williams-syndrome
1) therapy refractory hypoxia
Show comment and answer
2) increased dead space INT - 16.4
3) high mortality What does the term penetrance mean?
4) bilateral opacities on the chest X-
ray A) the frequency of a genetic alteration
the frequency of a certain What does the term compound
B) phenotype int he case of a certain heterozygosity mean?
genotype
C) a form of genetic regulation More mutations are responsible for
the expression of small molecular A) a certain disease in autosomal
D) dominantly inherited syndrome.
weight RNAs
E) region of chromosomal regulation The importance of Y chromosome
B)
alterations in sex-linked inheritance.
Show comment and answer
INT - 16.5 In the case of autosomal
What proportion of the human genome recessively inherited disease,
is protein-coding? different mutations on different
C)
allelles of the same gene are jointly
responsible for the develeopment of
A) 3%
the disease.
B) 10%
A rare regulatory mechanism in the
C) 25% D) case of X-linked dominant
D) 40% inheritance
E) 60% The emergence of different
Show comment and answer pheotypes due to different
E)
INT - 16.6 mutations int he case of autosomal
Which one of the following diseases recessively inherited diseases.
has the lowest penetrance? Show comment and answer
INT - 16.10
A) multiple endocrine neoplasia type 1 Which pattern of inheritance is
B) multiple endocrine neoplasia type 2 characteristic to blood relation
between parents?
C) neurofibromatosis type 1
D) cystic fibrosis A) autosomal dominant
E) haemochromatosis B) autosomal recessive
Show comment and answer C) X-linked dominant
INT - 16.7
Which one of the following diseases D) X-linked recessive
does not predispose to E) polygenic
pheochromocytoma? Show comment and answer
INT - 16.11
A) von Hippel–Lindau disease What is the chance of the emergence
of an autosomal recessively inherited
B) Gardner syndrome
disease in the offspring of two
C) multiple endocrine neoplasia type 2
heterozygote carriers?
D) neurofibromatosis type 1
E) inherited paraganglioma syndromes A) 10%
Show comment and answer B) 25%
INT - 16.8
C) 40%
By definition, what is the frequeny of a
D) 50%
mutation?
E) 60%
A) < 1% Show comment and answer
INT - 16.12
B) <5%
What is the chance of emergence of an
C) < 10%
autosomal dominantly inherited
D) < 20% disease in one child of an affected
E) < 30% heterozygote individual?
Show comment and answer
INT - 16.9 A) 10%
B) 25% Which chromosomal abnormality
C) 40% affecting the sex chromosomes has the
highest incidence?
D) 50%
E) 60%
A) Turner syndrome
Show comment and answer
B) Klinefelter syndrome
INT - 16.13
For which mode of inheritance is the C) Triple X sydrome (superfemale)
following statement true: of the D) 46XX male
offspring of an affected male and a E) 46XY female
healthy female, all sons are healthy Show comment and answer
and all daughters are affected? INT - 16.18
For which listed chromosome
A) X-linked recessive abnormality are the following
B) X-linked dominant symptoms characteristic:
C) autosomal recessive immunodeficiency,
hypoparathyroidism, congenital heart
D) autosomal dominant
defect?
E) polygenic
Show comment and answer A) Cri du chat syndrome
INT - 16.14
B) DiGeorge syndrome
Among the following diseases, in which
case does genomic imprinting not C) Down syndrome
constitute to the pathogenesis? D) Turner syndrome
E) Prader–Willi syndrome
A) Prader–Willi syndrome Show comment and answer
B) Angelman syndrome INT - 16.19
C) Beckwith–Wiedemann syndrome Which one of the following symptoms
is not characteristic to Turner
D) cystic fibrosis
syndrome?
Show comment and answer
INT - 16.15
A) short stature
Over what frequency of occurrence in
the general population is a genetic B) streak gonad
variant considered a polymorphism? C) primary amenorrhea
D) asthma bronchiale
A) 1% E) coarctation of the aorta
B) 7% Show comment and answer
C) 10% INT - 16.20
D) 15% Which one of the following symptoms
is not characteristic to testicular
E) 20%
feminization (complete androgen
Show comment and answer inensitivity syndrome, Morris
INT - 16.16
syndrome)?
The trisomy of which chromosome is
the most frequent in live-born new-
A) nromal pubic hair
borns?
46XY with normal female
B)
A) 13 phenotype
B) 17 absent inner genitals, hiányzó belső
C)
női nemi szervek, rövid vagina
C) 19
az androgénreceptor mutációi
D) 21 D)
állnak hátterében
E) 22 X-kromoszómához kapcsolt
Show comment and answer E)
öröklésmenetű
INT - 16.17
Show comment and answer Which one of the following symptoms
INT - 16.21 is not characteristic of cystic fibrosis?
Amog the following diseases which one
is not autosomal dominantly inherited? chronic obstructive pulmonary
A)
disease
A) Marfan syndrome B) deficiency of the exocrine pancreas
B) cystic fibrosis C) diabetes mellitus
C) Huntington’s disease D) meconium ileus in new-borns.
D) familial adenomatosus polyposis male infertility due to absence of
E) hereditary retinoblastoma E)
vas deferens
Show comment and answer Show comment and answer
INT - 16.22 INT - 16.26
Which statement is false for Which one of the following is false
phenylketonuria? regarding Duchenne muscular
distrophy?
A) autosomal recessively inherited
it is the consequence of mutations A) X-linked recessive inheritance
B) of the gene encoding phenylalanine it is the consequence of mutations
hydroxylase B) of the largest human protein,
the main symptoms are diarrhea dystrophin
C)
and secondary malabsorption one manifestation is the
C)
elevated phenylalanine levels cause pseudohypertrophy of the calves
D)
severe neurological symptoms weakness of the muscles begins in
D)
E) it is screened routinely in new-borns adulthood
Show comment and answer elevated serum levels of creatine
E)
INT - 16.23 kinase
Which one of the following hereditary Show comment and answer
metabolic diseases does not affect INT - 16.27
purine metabolism? Which of the following statements is
false for Huntington’s disease?
A) Tay–Sachs disease
B) Niemann–Pick disease it belongs to trinucleotide repeat
A)
C) Hurler syndrome disorders
D) Rett syndrome it follows autosomal dominant
B)
inheritance
E) Lesch–Nyhan syndrome
the disease manifestates in
Show comment and answer C) adulthood, usually after 40 years of
INT - 16.24
age
Which one of the following statements
one manifestation is severe
is false for cystic fibrosis? D)
hypertension
the most common fatal autosomal during familial inheritance, the first
A) recessive disorder among symptoms presents at earlier age in
E)
Caucasian race the offspring of the affected father
(anticipation)
B) the frequency of carriers is 1/25
Show comment and answer
C) the frequency of carriers is 1/500 INT - 16.28
it is the consequence of mutations Which of the following disorder is not a
D) of a protein involved in the transport trinucleotide repeat disorder?
of chloride ions.
ΔF508 is the most common A) fragile X syndrome
E)
mutation. B) myotonic dystrophy
Show comment and answer C) Huntington’s disease
INT - 16.25
D) Friedreich’s ataxia A) vitamin B1
E) Rett syndrome B) vitamin B2
Show comment and answer C) folic acid
INT - 16.29
D) vitamin C
Which one of the following is not
characteristic to diseases of the E) vitamin A
mitochondrial DNA? Show comment and answer
INT - 16.33
they can be inherited from the Which hereditary tumor syndrome
A) does not predispose to breast cancer?
mother and from the father
by a main rule, they can be
B) A) Li-Fraumeni syndrome
inherited only from the mother
B) mutations of BRCA1 gene
main characteristics include the
disorders of the nervous system C) mutations of BRCA2 gene
C)
(including the sensory nervous D) von Hippel–Lindau disease
system) and the muscle system E) Cowden syndrome
there may be wide phenotype Show comment and answer
D)
variability INT - 16.34
heteroplasmy is one of the main Which of the following syndromes is
E)
reasons of phenotype variability not associated with chromosomal
Show comment and answer instability arising from deficient DNA
INT - 16.30 repair?
Which following characteristic is false
for acute intermittent porphyria? A) neurofibromatosis type 1
B) ataxia teleangiectasia
A) autosomal dominant inheritance C) Bloom syndrome
B) high penetrance D) xeroderma pigmentosum
mutations of porphobilinogen E) Fanconi anemia
C) deaminase are responsible for the
disease Show comment and answer
INT - 16.35
severe abdominal pain which At which pregnancy age is chorionic
D) cannot be explained by organic
villus sampling recommended to be
reasons
performed?
vegetative nervous system is often
E)
affected A) 4–6 weeks
Show comment and answer B) 10–12 weeks
INT - 16.31
Which apolipoprotein E allele is linked C) 14–16 weeks
to Alzheimer’s disease? D) 18–20 weeks
E) 24–26 weeks
A) APOA Show comment and answer
B) APOC INT - 16.36
C) APOE1 When is prenatal diagnostical
procedures recommended?
D) APOE2
E) APOE4 A) advanced maternal age
Show comment and answer de novo chromosome abnormality
INT - 16.32 B)
in the previous child
Which of the following vitamin’s
deficiency has been mostly associated one parent carries structural
C)
with the development of neural tube chromosomal alteration
defects? D) in all three of the previous cases
E) none of the previous cases
Show comment and answer heteroygosity may have a protective
INT - 16.37 E)
effect against malaria
Which of the following disorders cannot INT - 16.41
be detected by fetal utrasound Which of the following diseases is
imaging? inherited in autosomal dominant
manner?
A) holoprosencephaly
B) neural tube defects A) multiple andocrine neoplasia type 1
C) omphalocele B) neurofibromatosis type 1
D) Huntington’s disease C) von Hippel–Lindau disease
E) cystic hygroma D) all three of them
Show comment and answer E) neither of them
INT - 16.38
Show comment and answer
Which two chromosomes are affected INT - 16.42
by translocation in the case of Which chromosome is affected in
Philadelphia chromosome? Prader-Willi syndrome?

A) 9 and 22 A) 3
B) 8 and 14 B) 12
C) 9 and 14 C) 15
D) 8 and 22 D) 17
E) 10 and 14 E) 19
Show comment and answer Show comment and answer
INT - 16.39 INT - 16.43
Which one of the following is not a Which of the following is false for Tay-
characteristic of the fragile X Sachs disease?
syndrome?
A) lysosomal storage disease
it causes moderate mental
Lisch nodules of the iris is a
retardation in affected boys and B)
A) characteristic
mild mental retardation in affected
girls cherry-red spot on the retina is a
C)
characteristic
it belongs to trinucleotide repeat
B) D) neurodegenerative disease
disorders
enlarged testicles is a characteristic hexosaminidase A deficiency plays
C) E)
of the disease in affected boys a role int he pathogenesis
congenital heart defects are Show comment and answer
D) INT - 16.44
fequently observed
Which of the following is the most
the disease is the consequence of
E) severe manifestation of thalassaemia?
FMR1 gene mutations
Show comment and answer
A) HbH disease
INT - 16.40
Which one of the following is not a B) hydrops fetalis
characteristic of glucose-6-phosphate C) hemolytic anemia
dehydrogenase deficiency? D) cerebral infarction
E) extramedullary hematopoiesis
A) X-linked recessive inheritance
Show comment and answer
frequently observed in INT - 16.45
B)
Mediterranean countries Which three chromosomal trisomies
predisposes to hemolytic anemia in can be compatible with life therefore
C)
homozygotes being rarely present in live-borns?
D) mental retardation is frequent
A) 15, 17, 21 INT - parathyroid hyperplasia, - )

B) 13, 18, 21 16.52 endocrine pancreatic


tumors, pituitary adenoma
C) 13, 14, 22
INT - medullary thyroid cancer, - )
D) 17, 18, 20 16.53 pheochromocytoma,
E) 7, 9, 13 hyperparathyroidism
Show comment and answer INT - neurofibroma, - )
Association question 16.54 neurofibrosarcoma, brain
Associate the following symptoms with tumor, pheochromocytoma
the appropriate monogenic disease! INT - breast cancer, brain tumor, - )
16.55 adrenocortical cancer,
INT - hemorrhagic diatheses, - sarcomas, leukemia
16.46 hemarthrosis, decisively
boys are affected A) multiple endocrine neoplasia 2A
INT - manifestates in adulthood, - B) neurofibromatosis type 1
16.47 severe neurodegenerative C) von Hippel–Lindau disease
disease, autosomal D) Li–Fraumeni syndrome
dominantly inherited E) multiple endocrine neoplasia type 1
INT - Severe muscle weakness, - Association question
16.48 pseudohypertrophy of the Associate the following symptoms with
calves, mild mental the appropriate chromosomal diseases!
retardation, X-linked
inheritance INT - Mental retardation, severe - )
INT - Chronic obstructive - 16.56 developmental disorder,
16.49 pulmonary disease, congenital heart defects,
pancreatic deficiency, prominent occiput,
meconium ileus in new- abnormally small jaw, low-
borns, infertility secondary set, malformed ears,
to the atresia of the vas affected individuals rarely
deferens survive beyond childhood
INT - tall, dysmorphic stature, - INT - mental retardation, short - )
16.50 alterations of the skeletal 16.57 stature, brachycephaly, flat
structure, ectopic eyepiece, nape, epicanthic fold,
mitral valve prolapse, congenital heart defects
dilation and rupture of the INT - Primary hypogonadism, - )
aorta 16.58 infertility, tall stature,
gynecomastia
A) Marfan syndrome INT - hypotonia and poor feeding - )
B) cystic fibrosis 16.59 during infancy, which turns
C) haemophilia (A and B) to hyperphagia and
D) Duchenne muscular dystrophy extreme obisity during
E) Huntington’s disease childhood, disproportionally
Association question smaller hands and toes,
Associate the following symptoms with cognitive disturbancies,
the appropriate monogenic tumor reproductive sterility
syndrome! INT - Short stature, shield chest, - )
16.60 webbed neck, streak
INT - clear cell renal cell - gonad, cardiovascular
16.51 carcinoma, malformations
pheochromocytoma, retinal
and cerebellar A) Prader–Willi syndrome
hemangioblastoma, B) Turner syndrome
pancreatic cysts and C) Down syndrome
neuroendocrine tumor D) Klinefelter syndrome
E) Edwards syndrome
Association question E) polygenic, multifactorial
Associate the following patterns of inheritance
inheritance with the characteristics! Association question
Associate the following therapeutical
INT - From the relationship of - ) approaches to the following genetic
16.61 affected males and healthy diseases!
females all daughters are
affected, while all sons are INT - congenital hypothyroidism - )
healthy. 50% of an 16.66
affected female’s children INT - haemophilia A and B - )
may inherit the disease 16.67
irrespective of the child’s INT - familial - )
sex. 16.68 hypercholesterolemia
INT - Familial history of the - ) INT - Gaucher’s disease - )
16.62 disease is a characteristic, 16.69 (glucocerebrosidase
however the pattern of deficiency)
inheritance does not follow INT - α-1 antitrypsin deficiency - )
monogenic Medelian 16.70
inheritance. Emergence of INT - phenylketonuria - )
the disease is more 16.71
common in closer than INT - galactosemia - )
further relatives of an 16.72
affected individual.
INT - The disease is much more - ) A) dietary restrictions
16.63 common in males. All B) enzyme replacement therapy
daughters of an affected C) hormone replacement therapy
male will be carrier, while D) inhibitor administration
all sons will be healthy. E) replacement of a specific protein
Heterozygote females are (not enzyme)
usually healthy, however Association question
manifestations to some Associate the following sexual
extent may be present. developmental disorders with the
INT - Typically, affected - ) characteristics!
16.64 individuals are present in
all generations. Every child INT - Classical forms are -
of an affected individual 16.73 associated with 45X0
has 50% chance to inherit kariotype.
the disease irrespective of INT - Disease is due to mutations -
sex. These diseases are 16.74 of the androgen receptor.
less likely to alter life INT - Serum levels of 17α- -
expectancy. 16.75 hydroxyprogesterone are
INT - These diseases are usually - ) elevated.
16.65 severe. Some generations INT - The most frequent disorder -
do not have affected 16.76 affecting sex
individuals. Blood relations chromosomes.
are common in the family
history of the affected A) testicular feminization (Morris
families. Males and females syndrome)
are affected in the same B) 21-hidroxylase deficiency
frequency. (congenital adrenal hyperplasia)
C) Turner syndrome
A) autosomal recessive D) Klinefelter syndrome
B) autosomal dominant
C) X-linked recessive
D) X-linked dominant
Association question A) ECOG performance status 0
Associate the chance of the birth of
B) ECOG performance status 1
affected child with the folowing cases
(assume maximal penetrance)! C) ECOG performance status 2
D) ECOG performance status 3
INT - In the daughters of an - E) ECOG performance status 4
16.77 affected male and a healthy Show comment and answer
female in X-linked INT - 17.3
dominant inheritance What is the relationship between
INT - In the relationship of two - dysplasia and cancer?
16.78 heterozygote in autosomal
recessive inheritance Dysplasia always leads to the
A)
INT - In the relationship of an - development of a cancer.
16.79 affected heterozygote and B) All cancer is preceded by dysplasia.
a healthy individual in
The progression of cancer is
autosomal dominant C)
dysplasia.
inheritance
D) A and C.
INT - In the sons of an affected -
16.80 male and a healthy female E) All of the above.
in X-linked recessive Show comment and answer
inheritance. INT - 17.4
At cellular level tumor growth does not
A) 25% depend on the following:
B) 50%
C) 100% A) Time of cell cycle
D) 0% Growth fraction (proliferating tumor
B)
cell ratio)
CLINICAL ONCOLOGY C) cell death rate inside the tumor
D) Time of duplication of tumor volume
INT - 17.1 E) apoptosis
Who decides to start a complex Show comment and answer
oncology treatment for a newly INT - 17.5
diagnosed cancer patient? Goal of treatment with combined
chemotherapy:
The Surgeon, because the tumor
A)
primarily requires resection. To achieve maximum tumor cell
A)
The Oncologist, because he knows destruction with tolerable toxicity.
B)
best about chemotherapy protocols. To prevent or slow down the
The pathologist, who determined B) development of a resistant cell line
C) the exact diagnosis using against new drugs.
immunohistochemical methods. Increase the therapeutic effect of
D) Radiotherapy specialist. C) the most active drug in
E) All of the above monotherapy.
Show comment and answer D) All of the above.
INT - 17.2 E) None of the above
A patient examined because of weight Show comment and answer
loss and abdominal pain was INT - 17.6
diagnosed with a malignant illness. The The prognosis of the following tumor
patient spends more than half time of was the most significantly improved by
the day in bed. What is the patient's combined chemotherapy:
general condition according to ECOG
PS? A) Small Cell Lung Cancer
B) Medulloblastoma
C) Seminoma B) RECIST-staging system
D) Neuroblastoma C) Forrest-staging system
Show comment and answer D) Bismuth-staging system
INT - 17.7 E) All of the above
Chemotherapy before surgery is
Show comment and answer
important for the treatment of the INT - 17.12
following tumor: In oncological clinical studies
progression free survival (PFS) is often
A) Wilms-tumor evaluated. How do you define this
B) medulloblastoma term?
C) Burkitt-lymphoma
D) retinoblastoma Progression-Free Survival (PFS) is
the time that elapses in a
E) Gallbladder cancer
progressive/metastatic tumor from
Show comment and answer A)
starting treatment (from clinical trial
INT - 17.8
randomization) to repeated tumor
After a curative surgery, the treatment growth and/or death of a patient.
for the prevention of recurrence:
PFS is the time that elapses from
the onset of the therapy of an
A) neoadjuvant B) advanced / metastatic disease (in
B) adjuvant clinical trials from randomization) to
C) palliative the death of a patient.
D) supportive Show comment and answer
INT - 17.13
E) none of the above
Definition of overall survival (OS):
Show comment and answer
INT - 17.9
Overall Survival (OS) is the time
To evaluate the effectiveness of
that elapses in a progressive /
adjuvant treatment, the following
metastatic tumor from starting
parameter is chosen:
A) treatment (from randomization in
clinical trials) to repeatedly
A) The time to relapse confirmed tumor growth and / or
B) Disease free survival death of the patient.
C) Overall survival Overall survival (OS) is the time that
D) All of the above elapses from the onset of tumor
B) treatment (in clinical trials from
Show comment and answer
INT - 17.10
randomization) to the death of a
patient.
In case of treatment of cancer
patients, improvement of the disease Show comment and answer
INT - 17.14
is called:
The overall response rate (ORR) is
A) remission used to express the effectiveness of
treatment in clinical trials. Which
B) stable disease
components does the ORR consist of?
C) progression (CR is the complete response, PR is
D) refractory disesase partial response, SD is stable disease /
E) none of the above and PD is progressive disease /.)
Show comment and answer
INT - 17.11 A) ORR= CR+PR+ SD + PD
Criteria for evaluating the therapeutic B) ORR= CR+PR+ SD
response during treatment of cancer C) ORR= CR+PR
patients:
Show comment and answer

A) Dukes-staging system
INT - 17.15 E) medulloblastoma gene receptor
PRO abbreviation is often used in Show comment and answer
oncology drug development. It is the INT - 17.20
abbreviation of: For which chemotherapeutic agent is
the citrovorum factor or folic acid used
A) Progression Related Outcome as a "rescue" compound for the
B) Patient Reported Outcome protection of normal tissues and thus
Show comment and answer high dosage chemotherapy?
INT - 17.16
What is the RECIST criteria used for in A) methotrexat
oncology and in oncological clinical B) bleomycin
trials? C) cyclophosphamid
D) adriamycin
In solid tumors, the response to
anticancer treatment in imaging E) vincristin
A)
studies is evaluated by these Show comment and answer
criteria. INT - 17.21
RECIST relates to the Mesna may reduce the toxic damage of
B) determination of cell surface the bladder. Which cytostatic agent is
receptors in solid tumors. it used for?
Show comment and answer
INT - 17.17 A) 5-fluorouracil
In ca.10% of cancer patients, B) adriamycin
hypercalcaemia occurs due to tumor C) cisplatin
cells producing materials. Such D) cyclophosphamid
materials are the following, except
E) methotrexat
(indicate the exception):
Show comment and answer
INT - 17.22
production of parathormon-
A) The following side effects may occur
likepeptide
immediately or early after cytotoxic
B) calcitonin
therapy except (indicate the
C) active metabolits of Vitamin-D exception):
D) prostaglandins
E) TGF-α és TGF-β A) nausea / vomiting
Show comment and answer B) urocystitis
INT - 17.18 C) renal failure
Which gene's defect is the most
D) phlebitis
common in human tumors?
E) azoospermia
A) c-raf Show comment and answer
INT - 17.23
B) c-myc
Which phase of the cell cycle is
C) K-ras affected by taxane type cytostatics?
D) p53
E) c-fos A) G0
Show comment and answer B) G1
INT - 17.19 C) S
What is the meaning of MDR?
D) G2
E) M
A) minimal disease residue
Show comment and answer
B) multiple drug resistance
C) maximal dose rate
D) mesenchymal disease risk
INT - 17.24 CA-19-9 tumor marker test is the most
What is the machanism of action of valuable in the follow-up of the
camptothecin cytostatics (irinotecan, following disease:
topotecan)?
A) ovarian cancer
inhibits the depolymerization of B) hepatocellular carcinoma
A)
microtubules
C) testicular cancer
B) topoizomerase II. inhibitors
D) breast cancer
C) topoizomerase I. inhibitors
E) pancreas cancer
Inhibition of DNA function and cell
D) Show comment and answer
division in phase M
INT - 17.29
Show comment and answer CA-15-3 tumor marker test is the most
INT - 17.25
valuable in the follow-up of the
The following is valid for antimitotic
following disease:
chemotherapeutic drugs:
A) ovarian cancer
causes covalent crosslinking of the
A) two polynucleotide chains in the B) hepatocellular carcinoma
dual spiral of DNA C) testicular cancer
they inhibit DNA synthesis through D) breast cancer
B) the inhibition of important enzymes E) pancreatic cancer
of purine or pyrimidine synthesis. Show comment and answer
they inhibit the polymerization of INT - 17.30
C) tubulins, i.e. the formation of mitotic AFP (alfa-foeto-protein) tumor marker
spindles. test is the most valuable in the follow-
They inhibit the depolymerization of up of the following disease:
D) tubulins, i.e. blocking the mitotic
spindle. A) ovarian cancer
E) C and D B) hepatocellular carcinoma
Show comment and answer C) testicular cancer
INT - 17.26
D) breast cancer
Which compound belongs to the strong
opiates? E) pancreatic cancer
Show comment and answer
INT - 17.31
A) codein
CA125 tumor marker test is the most
B) oxycodon
valuable in the follow-up of the
C) tramadol following disease:
D) dextropropoxyphen
Show comment and answer A) ovarian cancer
INT - 17.27 B) hepatocellular carcinoma
Targeted therapies for treatment of
C) testicular cancer
solid tumors except (indicate the
exception): D) breast cancer
E) pancreatic cancer
A) adalimumab Show comment and answer
INT - 17.32
B) imatinib
β-HCG tumor marker test is the most
C) sorafenib
valuable in the follow-up of the
D) cetuximab following disease:
E) sunitinib
Show comment and answer A) ovarian cancer
INT - 17.28 B) hepatocellular carcinoma
C) testicular cancer
D) breast cancer C) renal cancer
E) pancreatic cancer D) chorionepithelioma
Show comment and answer E) osteosarcoma
INT - 17.33 Show comment and answer
In case of prostate cancer, which of INT - 17.40
the following tumor markers should be Which drug was the first approved
followed? „targeted therapy” in the treatment of
hepacullular carcinoma?
A) β-HCG
B) CEA A) sorafenib
C) PSA B) bevacizumab
D) CA-125 C) gefitinib
Show comment and answer D) vandetanib
INT - 17.34 E) ipilimumab
Which factor is not used for monitoring
Show comment and answer
or as prognostic factor in tumors? INT - 17.41
According to clinical studies, the
A) CEA sorafenib treatment in patients
B) PSA diagnosed with primary hepatic cancer
C) β-HCG (hepatocellulray carcinoma),increases
D) TSH the overral survival. Is this true or
E) p53 false?
Show comment and answer
A) true
INT - 17.35
What is the most common histological B) false
type of malignant oesophageal cancer? Show comment and answer
INT - 17.42
A) leiomyosarcoma Choose the correct answer!
B) epithelial carcinoma
The mortality of colon cancer is
C) adenocarcinoma A)
constantly increasing
D) rhabdomyosarcoma The mortality of colon cancer has
E) melanoma been decreasing since the
B)
Show comment and answer introduciton of effective
INT - 17.36 chemotherapy
What is the most common primary The mortality of colon cancer was
tumor site in Krukenberg tumor? rapidly increasing between 1965
C)
and 2000, since then it has been
A) breast cancer stagnant
B) gallbladder cancer The mortality of colon cancer hasn’t
C) rectum cancer D) changed significantly in the past few
decades
D) pancreas cancer
In first world countries,colon cancer
E) gastric cancer
E) has the highest mortality rate of all
Show comment and answer cancers
INT - 17.37
Show comment and answer
In case of haematogenous
INT - 17.43
dissemination, the following tumors
Chemotherapy drugs used in the
usually give a pulmonary metastases
treatment of colorectal cancer, except:
first, except one type of tumor:
A) 5-fluorouracil
A) gastric cancer
B) capecitabine
B) breast cancer
C) oxaliplatin Carcinoid syndrome is present in
D) irinotecan E) less than 10% of all neuroendocrine
tumors.
E) docetaxel
Show comment and answer
Show comment and answer INT - 17.48
INT - 17.44
The following malignant tumors,during
Treatment option in the therapy of
hematogenous spreading metastatize
rectal cancer:
at first to the liver, except:
A) total mesorectal excision
A) gastric cancer
B) neoadjuvant chemotherapy
B) colon cancer
C) adjuvant chemotherapy
C) panreatic cancer
D) VEGF inhibitor drugs
D) adrenal gland carcinoma
E) all of the above
E) gallblader cancer
Show comment and answer
INT - 17.45
Show comment and answer
INT - 17.49
In which malignant disease ”DNA
Of the following factors which ones
mismatch repair error” is the most
should be measured-calculating the
frequent?
Notthingham Prognostic Index?
A) Li–Fraumeni-sydrome
A) size of the tumor in centimeter
B) MEN-2 syndrome
B) histological grade
C) retinoblastoma
C) lymph node status
hereditary, non polyposis colon
D) D) all of the above three factors
cancer (HPNCC)
Show comment and answer
E) Wilms-tumor
INT - 17.50
Show comment and answer During the histological evaluation of
INT - 17.46
breast cancer we use the „NPI”
Which of the following malignant abbreviation. What does it stand for?
cancers don’t ususally metastatize to
the bones?
A) Norfolk Predictive Index
B) Notthingam Prognostic Index
A) thyroid cancer
Show comment and answer
B) breast cancer
INT - 17.51
C) renal cancer In the case of breast cancer which NPI
D) colon cancer score is considered as poor prognosis?
E) prostate cancer
Show comment and answer A) below 3,14
INT - 17.47 B) above 5,4
Correct statements regarding the Show comment and answer
neuroendocrine tumor (NET), except: INT - 17.52
Which of the following ones is NOT a
A)
Neuroendocrine tumors are rare risk factor of breast cancer?
entities
The incidence of neuroendocrine A) no children
B) tumors has been increasing in the B) early menarche, late menopause
past 30 years.
C) promiscuity
Most of the neuroendocrine tumors
D) BRCA1 mutation
C) are classified as "non-functioning"
types. E) sister’s breast cancer
Neuroendocrine tumor most Show comment and answer
D) INT - 17.53
typically affects the pancreas.
What is the most common sypmtom of The survival rate of renal cancer-
the intraductal papilloma of the breast? thanks to the widely used screening
A)
methods- has increased in the last
peau d’orange symptom (orange few years.
A)
peel symptom) In the case of obvious imaging test
enlarged lymph node at the findings, which suggest malignant
B) B)
unilateral side tumor, surgical therapy can be
bloody fluid discharging from the indicated.
C)
nipple In the treatment of renal cancer,
D) ekzematous skin lesion C) surgery is the only definitive and
possibly curative option.
painful,red, swollen, palpable mass
E) In the case of metastatic.renal
in the breast
cancer palliative nephrectomy
Show comment and answer
D) should be performed, even if the
INT - 17.54
patients are in good general
Which type of breast cancer affects
condition
typically both sides?
In the case of small peripheric
tumor of the kidney laparoscopic
A) invasive ductuscarcinoma
partial neprehctomy is an option.
B) medullar carcinoma E) The most common indication of
C) mucinosus carcinoma radiotherapy is the palliative
D) lobular carcinoma treatment of intracranial and bone
metastasis.
E) papiliar carcinoma
Show comment and answer
Show comment and answer
INT - 17.59
INT - 17.55
Diagnostic methods in the screening of
In the case of an 22-years-old female
prostate cancer:
patient, a mobile, non-painful palpable
mass was discovered in the right
breast with a diameter of 2,5 cm, and A) rectal digital examination
no change in size during the menstrual B) PSA-test
cycle. What is the most probable C) PET-CT
diagnosis? transrectal ultrasound-guided
D)
biopsy
A) fat necrosis E) A, B and D
B) fibroadenoma Show comment and answer
C) carcinoma phylloides INT - 17.60
D) mastopathia fibrocystica The gold standard for treatment of
bone metastases of prostate cancer is:
E) tubularis carcinoma
Show comment and answer
A) GnRh analogue drugs
INT - 17.56
Which one is not a paraneoplastic B) radiotherapy
symptom of renal cancer? C) antiandrogen drugs
D) bisphophonates
A) anaemia E) chemotherapy
B) weight loss Show comment and answer
C) fever, subfebrility INT - 17.61
D) tiredness and fatigue Choose the CORRECT answer!
E) hematuria
In spite of the increasing incidence
Show comment and answer of prostate cancer, the mortality
INT - 17.58 A)
caused by the disease has been
Choose the INCORRECT answer! slightly decreasing.
Increased risk was observed with E) Ewing-sarcoma
family history of the disease, as Show comment and answer
B) hereditary genes which can cause INT - 17.64
susceptibility to prostate cancer Which type of germ-cell testicular
were indentified. tumor cannot be classified into the
Although prostate carcinoma can be non-seminoma group?
observed in 30-40 % of men above
C) the age of 55, the disease is A) choriocarcinoma
ususally diagnosed over the age of
B) Yolksac tumor
65.
C) teratoma
In 95% of the cases, prostate
cancer orginates from the D) spermatocytic
D)
“peripherial zone”, and it is usually E) polyembrioma
multifocal. Show comment and answer
E) All statements are true. INT - 17.65
Show comment and answer Which cell-type does the majority of
INT - 17.62 testicular cancers originate from?
Choose the INCORRECT asnwer!
A) Leydig-cell
After radical prostectomy, if the B) Sertoli-cell
PSA level is above 0,4 ng/ml, C) germ cell
A)
hormone and/or radiotherapy is
D) endothel cell
necessary.
E) epithelial cell
Treatment with antiadnrogene drug
should be introduced few days Show comment and answer
B) INT - 17.66
before applying the LHRH analogue
at the star of hormone therapy Which one is the most common
testicular germ-cell tumor?
In the case of metastatic prostate
cancer systemic chemotherapy
C) A) teratoma
should be started as soon as
possible. B) endodermal sinus tumor
During intermittent hormone C) embryonal carcinoma
therapy, in the "treatment holidays" D) choriocarcinoma
D)
the sexual potential of the patient
E) seminoma
usually is restored.
Show comment and answer
When prostate cancer metastatizes INT - 17.67
to the bones, hormone and Choose the INCORRECT answer!
E)
bisphosphonate therapy is the first
choice treatment.
In the case of cryptorchism the
Radiotherapy in prostate can be A) incidence of testicular cancer is 10-
F) used for curative or palliative 40 times higher.
purpose as well.
Testicular cancer is the most
Show comment and answer B) common malignancy among young
INT - 17.63
adults.
Elder man is hospitalized with back
The most common type of testicular
pain, X-ray describes oval masses in
among infants and children are the
the lumbal spine, and PSA-level is
teratoma and the yolksac tumors,
elevated What is the most likely C)
between 15 and 35 years the non-
diagnosis? seminoma, between 40 and 60 the
seminoma.
A) metastatic thyroid cancer
Testicular cancer is bilateral in the
B) multiple myeloma D)
1-7% of all cases.
C) metastatic prostate cancer Regular self-exams are useful for
E)
D) metastatic pacreatic cancer early detection of testicular cancer.
In the case of testicular cancer monochemotherapy is equivalent to
,swollen, enlarged testes, abnormal the irradiation of the paraaortic
F) inguinal lymph nodes can be region.
generally palpable during physical Show comment and answer
examination. INT - 17.70
Show comment and answer Which one is the most important risk
INT - 17.68 factor of the transitional-cell carcioma
Choose the CORRECT answer! of the bladder?

The additon of cisplatin A) alcohol


chemotherapy to the treatment of B) smoking
A)
testicular cancer led to excellent
results. C) HPV infection
In the case of metastatic testicular D) E. coli infection
cancer surgery is not E) HBV infection
B)
recommended, only systemic Show comment and answer
chemotherapy and radiotherapy. INT - 17.71
In the case of advanced testicular Wich one is NOT typical to the familiar
C) cancer prior to surgery, neoadjuvant ovarian cancer?
chemotherapy is recommended.
In the case of relapse of testicular Family history of two or more
A)
cancer salvage chemotherapy is breast/ovarian tumor
D)
useless since the disease is It is usually diagnosed at young
B)
threapy-resistent. age.
Since testicular seminoma is highly- C) It is an agressive cancer.
sensitive to radiotherapy, after- The genotype of the disease is
surgery irradiation of paraaortic and D)
E) already well-known.
parailiac lymph node region on the
Show comment and answer
same side is part of standard
INT - 17.73
treatment.
Typical of germ-cell ovarian cancer:
Show comment and answer
INT - 17.69
A) It is more frequent in old age.
Choose the INCORRECT answer!
It is often discovered during
B)
pregnancy.
Choosing the most adequate
treatment centre is essential in the C) It is always bilateral.
A) aspect of complex therapy since It is usually resistent to chemo- and
D)
testicular cancer has excellent radiotherapy.
healing results. Show comment and answer
In the majority of testicular cancer INT - 17.74
B) cases the first step of treatment is Which one is NOT part of the
surgical high-castration. treatment of the epithelial ovarian
In the case of testicular seminoma cancer?
the cumulative dose of
supplementary irradiaton of the A) combined systemic chemotherapy
C)
primary lymph node regions has hysterectomy, bilateral salpingo-
been decreasing in the last few B)
oophorectomy, omentectomy
years.
C) P32 intraabdominal irradiation
In the case of stage I. testicular
D) intraperitoneal chemotherapy
D) cancer, after surgery only regular
check-ups are necessary. Show comment and answer
INT - 17.75
In the case of early testicular
The incidence of cervical cancer:
E) seminoma, according to recent
studies, the carboplatin-based
It is the most frequent among In case of endometrium cancer:
A)
women in their thirties and forties.
The primary risk factor is HPV 15 most frequent symptom is the onset
B) A)
and 17 infection. of ascites
It is usually occurs among women B) can be prevented by screening
C)
of higher social-economical status. In case of a postmenopausal
It is usually discovered in older age C) hemorrhage you should think about
D) this
and at early stage.
Show comment and answer curettage is no longer used in the
D)
INT - 17.76 diagnosis
Choose the correct answer regarding Show comment and answer
cervical cancer: INT - 17.81
In the treatment of uterine cancer
The most common type is the
A) The most commonly used treatment
planocellular carcinoma A)
the adenocarcinoma is extremely is surgery
B) pre- and postoperative radiotherapy
rare B)
the neuroendocrine type has good is indispensable
C) hormone therapy and
prognosis C)
the incidence of planocellular chemotherapy are equally effective
D) D) all of the above
carcinoma has been increasing
Show comment and answer Show comment and answer
INT - 17.77 INT - 17.82
The treatment of early cervical cancer: Endometrial carcinoma

Can be administered in every is the most common gynecological


A) A)
surgical department tumor
B) Radiotherapy is ineffective it practically does not occur in
B)
C) The standard therapy is surgery premenopausal status
Neoadjuvant chemotherapy is squamous cell cancer is more
D) C)
widely used frequent than the glandular origin
Show comment and answer D) Estrogen excess is not a risk factor
INT - 17.78 Show comment and answer
Most effective treatment of advanced INT - 17.83
cervix cancer: Cuboidal epithelium of the bronchi is
replaced by squamosal cells. What is
A) radical surgery this phenomenon?
B) chemotherapy and irradiation
A) dysplasia
C) only chemotherapy
B) metaplasia
D) only irradiation
C) anaplasia
Show comment and answer
INT - 17.79 D) hyperplasia
In case of a recurrent, disseminated E) neoplasia
cervix cancer: Show comment and answer
INT - 17.84
A) the primary care is the palliation A tüdőrákok szűrésére igaz:
recto- and vesico-vaginal fistulas
B) Pulmonary X-ray screening is one
are frequently onset A)
C) chemotherapy is usually ineffective of the primary prevention tools.
D) all of the listed above Regular, yearly performed chest X-
B) ray tests reduce lung cancer
Show comment and answer mortality.
INT - 17.80
Regular lung chest x-rays combined
with sputum cytology will further Small cell lung cancer is a
C) A)
reduce the mortality rate caused by neuroendocrine carcinoma.
lung cancer. 80-85% of the lung neuroendocrine
Regular chest CT scan is not B) tumors are well and moderately
considered as a screening test due differentiated tumors.
D)
to overdiagnosis and radiation Good and moderately differentiated
exposure. C) carcinomas can produce
PET-CT test is a general cancer biologically active mediators.
E) screening test, also suitable for the Small cell lung cancer is considered
screening of lung cancer. to be a systemic disorder, so
D)
Show comment and answer surgical intervention is not
INT - 17.85 recommended.
It is true for radiographic examination Primary treatment of small cell lung
of lung cancer: E) cancer is chemotherapy and
radiochemotherapy.
Chest X-ray is no longer used to If SCLCs show full or partial
A)
diagnose lung cancer. F) remission, prophylactic skull
Ultrasound examination is not radiation is recommended.
relevant for lung cancer because it Show comment and answer
B) is not suitable for the examination of INT - 17.88
air-filled lung tissue due to technical It is true for medullary thyroid cancer,
reasons. except (indicate the exception):
Magnetic resonance imaging does
not play a role in the diagnosis of Sporadic medullary thyroid cancer
C)
lung cancer due to technical is usually a solitaer lesion, while
difficulties. A) familial cancers are more often
PET CT scan is always suitable for multifocal. 20-30% of medullary
D) separating lung cancer and thyroid cancer is sporadic.
inflammatory lung diseases. In the case of tumors above 1 cm,
CT scan has a crucial role in the B) surgery should be followed by
E)
diagnosis of lung tumors. radio-iodine treatment.
Show comment and answer In case of the diagnosis of
INT - 17.86 medullary thyroid gland cancer,
C)
For the relationship between smoking genetic testing is always
habits and malignant tumors of the recommended to be performed.
lungs are not true: If a RET protooncogene mutation is
detectable during the screening of
D)
85-90% of the tumors are related to the family, prophylactic
A)
smoking habits. thyroidectomy is recommended.
Female smokers are more Serum calcitonin and CEA level is
B)
vulnerable to lung cancer than men. essential to be controlled after
E)
Small cell lung tumors are almost thyreoidectomy to follow-up of
C) medullary thyroid cancer.
always produced in smokers.
Smoking habits do not affect the Show comment and answer
D) INT - 17.89
response to oncological treatment
Which disease does not belong to the
There is a nicotinerg receptor
antagonist that has been clinically cutaneous lymphoma group?
E)
proven to be able to help to quit
smoking. blastosplasmocytoid dendritic cell
A)
neoplasia
Show comment and answer
INT - 17.87 B) Sezary-syndrome
It is not true for the neuroendocrine C) lupus vulgaris
tumors of the lung:
D) mycosis fungoides B) gastric cancer
Show comment and answer C) colorectal cancer
INT - 17.90 D) Hodgkin-disease
What is the most important prognostic
E) CML
factor in malignant melanoma?
Show comment and answer
INT - 17.95
A) type of melanoma cells
Select the correct one from the
B) proliferation rate of melanoma cells following:
C) the degree of depth propagation
the intensity of the peritumoral Only high-energy photon irradiation
D)
reaction of the host A) can be performed with a linear
E) tumor localization accelerator.
Show comment and answer Gamma knife is a special linear
B)
INT - 17.91 accelerator.
Gene mutations that are involved in The cyber-knife is a 6MV robotic
C)
the development of melanoma linear accelerator.
malignant – please indicate the one Traditional X-ray therapies do not
exception: D)
play any role today.
A linear accelerator is used for
A) C-KIT E)
after-loading therapies.
B) B-RAF Show comment and answer
C) Her 2 INT - 17.96
D) CDKN2A Select the correct statement from the
following:
Show comment and answer
INT - 17.92
What is the main direction of During teletherapy irradiation an
A)
haematogenic spread of malignant external radiation source is used.
melanoma of the skin? During brachytherapy the
radioactive source is delivered
B)
A) lungs directly to the tumor or its
immediate vicinity.
B) bone
Brachytherapy can be divided into
C) brain interstitial, intracavital, intraluminal
D) liver C) and moulage based on the
E) adrenal gland technique of insertion into the
Show comment and answer irradiated area.
INT - 17.93 The combination of teletherapy and
What is the most common intraocular brachytherapy is frequently adviced
D)
tumor in adults? to reach the optimal tumor
destruction.
A) meningeoma E) All of the above are true.
B) lymphoma Show comment and answer
C) metastasis INT - 17.98
It is true for angiogenesis treatment
D) melanoma
that:
E) rhabdomyosarcoma
Show comment and answer 1) It leads to the formation of less
INT - 17.94 pervasive blood vessels with preserved
COPP protocol is a highly effective basal membrane and supporting cells.
chemotherapeutic regimen to treat the 2) It increases interstitial pressure.
tumor - first of all in which cancer? 3) It worsens the oxygenation of the
cancerous tissue.
A) squamous cell pulmonary cancer
4) It results in better penetration of 3) Keep at least 1 cm intact surgical
drugs. margin.
5) each of the above listed 4) For synchronous metastases, if
all metastases can be removed.
1st, 2nd and 3rd answers are 5) Residual liver tissue is expected
A)
correct to be sufficient, good functioning.
3rd, 4th and 5th answers are
B) 1st, 2nd and 3rd answers are
correct A)
C) 1st, 3rd and 4th answers are correct correct
D) all of the answers are correct 3rd, 4th and 5th answers are
B)
correct
Show comment and answer
INT - 17.99 C) 4th and 5th answers are correct
Which of the two tumor marker tests is D) all of the answers are correct
recommended for gastric cancer: Show comment and answer
INT - 17.102
1) beta-HCG The following factors may play a role in
2) CEA the development of primary liver
3) CA 19-9 cancer (primary hepatocellular
4) CA 125 carcinoma):

1st, 2nd and 3rd answers are 1) chronic hepatitis C infection


A)
correct 2) chronic hepatitis B infection
B) 3rd and 4th answers are correct 3) toxic liver diseases.
C) 2nd and 3rd answers are correct 4) metabolic liver disease
D) all of the answers are correct
1st, 2nd and 3rd answers are
Show comment and answer A)
correct
INT - 17.100
B) 1st and 2nd answers are correct
Palliative treatment for esophageal
cancer: C) none of the answers are correct
D) all of the answers are correct
1) Esophageal stent implantation Show comment and answer
2) implantation of percutaneous INT - 17.103
endoscopic gastrostoma (PEG) Select which radiological intervention
3) preparation of a nourishing method can be used to treat liver
Jejunostoma metastasis of colorectal carcinoma!
4) making a gastro-entero-
anastomosis (GEA) 1) percutaneous alcohol infiltration
5) radiofrequency ablation 2) J-131-MIBG treatment
3) radiofrequency ablation
1st, 2nd and 3rd answers are 4) embolization
A)
correct
3rd, 4th and 5th answers are 1st, 2nd and 3rd answers are
B) A)
correct correct
C) 4th and 5th answers are correct 1st, 2nd and 3rd answers are
B)
D) all of the answers are correct correct
Show comment and answer C) 2nd and 4th answers are correct
INT - 17.101 D) all of the answers are correct
Criteria for liver resection: Show comment and answer
INT - 17.104
1) Three or less metastases, located Classification systems used for the
in one lobe. staging of colorectal carcinoma (CRC):
2) The metastasis is smaller than 5
cm. 1) TNM assignment
2) Modified Astler-Coller (MAC) It is true for the oncological treatment
classification of metastatic kidney tumors:
3) Dukes Classification
4) FIGO position 1) Most common indication of
5) each of the above listed radiotherapy is palliative treatment of
brain or bone metastases.
1st, 2nd and 3rd answers are 2) If a solid organ metastasis
A) appears in a patient of overall good
correct
3rd, 4th and 5th answers are condition, a surgical resection is
B) advised to be performed.
correct
3) The choice of treatment for
C) 4th and 5th answers are correct
metastatic renal cell carcinoma is
D) all of the answers are correct based on prognostic risk assessment.
Show comment and answer 4) Molecular target therapies can be
INT - 17.105 used as second-line treatments after
What malignant disease can be treated conventional chemotherapy.
with tyrosine kinase inhibitor imatinib 5) The previously standard cytokine
(Glivec)? therapy has no current role in the
treatment of kidney tumors.
1) colon cancer
6) Molecular targeting agents for
2) gastrointestinal stromal tumor
renal cell cancer are VEGF inhibitor
(GIST)
bevacizumab, multikinase inhibitors
3) pancreatic cancer
and mTOR inhibitors.
4) breast cancer
5) chronic myeloid leukemia (CML)
1st, 2nd, 3rd and 6th answers are
A)
correct
1st, 2nd and 3rd answers are
A) 2nd, 3rd, 4th and 5th answers are
correct B)
correct
3rd, 4th and 5th answers are
B) 1st, 2nd, 3rd and 5th answers are
correct C)
correct
C) 2nd and 5th answers are correct
D) all of the answers are correct
D) all of the answers are correct
Show comment and answer
Show comment and answer INT - 17.108
INT - 17.106
It is true for prostate tumors:
The following approaches are used to
treat breast cancer: 1) Patients can be divided into three
risk groups based on T stage, PSA
1) neoadjuvant chemotherapy
value and Gleason score.
2) chemotherapy according to the
2) In case of a prostate tumor
DeGramont protocol
detected by rectal digital examination
3) radiotherapy
(RDV) and diagnostic imaging
4) trastuzumab treatment in HER-2
modalities, with an elevated PSA level
negative tumors
FNAB is enough to be performed to
5) each of the above listed
confirm the diagnosis and initiate the
primary treatment.
1st, 2nd and 3rd answers are 3) If malignant cells are found in
A)
correct
the map-biopsy specimen of the
3rd, 4th and 5th answers are prostate, the oncological treatment
B)
correct must be initiated as soon as possible.
C) 1st and 3rd answers are correct 4) Radical prostatectomy is
D) all of the answers are correct recommended only if the patient's life
Show comment and answer expectancy exceeds 10 years.
INT - 17.107 5) If the histological finding of
prostate cancer in a pure or mixed
form suggests a neuroendocrine INT - 17.111
tumor, it means a poor prognosis. Select the correct statements from the
following.
1st, 2nd and 3rd answers are
A) 1) In case of primary
correct
testis/retroperitoneal non-seminoma if
3rd, 4th and 5th answers are
B)
correct the AFP, HCG, and LDH levels are low
before the surgery the tumor has good
C) 1st, 4th and 5th answers are correct
prognosis; even if pulmonary
D) all of the answers are correct metastases are already onset.
Show comment and answer 2) Since seminomas rarely produce
INT - 17.109 beta hCG and never produce AFP,
It is true for prostate cancer: when elevated AFP level is detected
histological revision is needed to clarify
1) Lymph node metastases are the diagnosis if pure seminoma was
most commonly onset in the triangle of described earlier.
vena iliaca externa and n. 3) Because of its pulmonary
obturatorius. toxicity, bleomycin is no longer
2) Distant metastases most often recommended to be used in
appear in the bones. combinational chemotherapy protocols
3) Radical prostatectomy is still 4) Since non-seminomas with good
recommended even in case of already prognosis are sensitive to
onset bone metastases. chemotherapy, the therapeutic goal
4) Treatment of localized prostate must be the achievement of a long
cancer radiotherapy with curative lasting result with the lowest possible
doses can be a reasonable alternative toxicity, even in advanced disease.
to radical surgery. 5) For patients with non-seminomas
5) The total dose of radiotherapy belonging to the moderate or poor
has been reduced in the recent years prognostic group, radiotherapy is part
to minimize side effects. of standard therapy.
6) Although non-seminomas are
1st, 2nd and 4th answers are less sensitive to irradiation, a good
A)
correct palliative effect can be achieved by
3rd, 4th and 5th answers are radiotherapy in a chemotherapy-
B)
correct resistant, inoperable cases.
C) 4th and 5th answers are correct
D) all of the answers are correct 1st, 2nd, 4th and 6th answers are
A)
Show comment and answer correct
INT - 17.110 2nd, 3rd, 4th and 5th answers are
B)
Which of the following statement (s) correct
are true for non-seminomas? C) 4th and 5th answers are correct
D) all of the answers are correct
1) increased AFP level
2) elevated beta HCG level Show comment and answer
INT - 17.112
3) radiation sensitivity
It's true for the bladder tumors:
4) chemotherapy sensitivity
1) About 90% of the bladder tumors
1st, 2nd and 3rd answers are are transitional cell carcinomas.
A)
correct 2) The symptoms cannot be used to
1st, 2nd and 4th answers are distinguish between superficial and
B)
correct muscle-infiltrating tumors.
C) 3rd and 4th answers are correct 3) After a transurethral resection
D) all of the answers are correct (TUR) it is recommended to wait at
Show comment and answer least 4 weeks to perform CT or MRI
examination due to post procedure The tumors of the head and neck
edema, inflammation and lymph node region are (select the true
enlargement. statements):
4) Definitive radiotherapy is not
recommended because of the high risk 1) Squamous cell carcinoma of the
of bladder- shrinkage. tonsils and tongue are more common
5) Palliative radiotherapy of the in HPV infection.
urinary bladder should be performed 2) In the case of epipharynic
with fewer fractions with a higher daily tumors, radiotherapy is contraindicated
dose and a lower total dosage due to the proximity of the
compared to the treatment of surrounding dose-limiting organs (e.g.
definitive irradiation. brain stem, cerebral cortex,
6) Concomitant radiochemotherapy hypophysis, etc.).
is not used in urinary bladder tumors. 3) Radiochemotherapy and radio-
target therapies can also be used for
1st, 2nd, 4th and 6th answers are the definitive treatment of head and
A) neck cancer.
correct
1st, 2nd, 3rd, 4th and 5th answers 4) The development of second
B) primary tumors in the lungs or
are correct
esophagus is relatively common in
C) 4th and 5th answers are correct
patients with head and neck squamous
D) all of the answers are correct cell carcinoma associated with smoking
Show comment and answer and alcohol abuse.
INT - 17.113 5) Definitive radiotherapy in this
Select the correct statements from the region is only relevant for advanced
following. disease.
1) High energy irradiation (> 1 MV)
A) 1st, 3rd and 4th answers are correct
has higher skin side effects.
2) 3-D (3D) radiation planning CT, 1st, 2nd, 4th and 5th answers are
B)
MRI as well as PET/CT could also be correct
used. C) 2nd and 4th answers are correct
3) Intensity Modulated Irradiation D) all of the answers are correct
(IMRT) is performed by multi- Show comment and answer
directional beams corresponding to the INT - 17.115
region specific spatial and anatomical It's true for thyroid cancer:
conditions within which the beam
intensity is changing. 1) About 90% of thyroid cancer is
4) Hyperfractionation can be used in well-differentiated cancer and
fast growing tumors to reduce expresses TSH receptors.
treatment time and early adverse 2) Therapy of thyroid cancer is
events. primarily surgical.
5) The primary purpose of 3) In case of differentiated thyroid
radiotherapy is to increase the cancer, hematogenous spreading is
effectiveness of radiotherapy. more frequently occurs in younger
patients, especially in the liver.
1st, 2nd and 4th answers are 4) Radionuclide treatment is the
A) second most important element in
correct
2nd, 3rd, 4th and 5th answers are papillary and medullary cancers after
B) surgery.
correct
C) 3rd and 5th answers are correct 5) Anaplastic thyroid cancer is
disease with poor prognosis.
D) all of the answers are correct
6) In case of differentiated thyroid
Show comment and answer cancer, ablative iodide isotope
INT - 17.114
treatment should be performed after 2) Stereotactic brain irradiation is
surgery. possible even in case of multiple brain
lesions.
1st, 3rd, 4th and 5th answers are 3) Cancers of the central nervous
A)
correct system cannot be treated by
1st, 2nd, 3rd and 5th answers are chemotherapy because
B) chemotherapeutic agents do not pass
correct
2nd, 3rd and 6th answers are through the blood-brain barrier.
C) 4) In case of radical surgery,
correct
adjuvant therapy is not recommended,
1st, 2nd, 5th and 6th answers are
D) due to expected late toxicities even in
correct
case of highly malignant tumors.
E) all of the answers are correct
5) Although there is no lymphoid
Show comment and answer tissue in the nervous system,
INT - 17.116
lymphoma may occur in central
It is true for the tumors of the central
nervous system
nervous system:
6) In some cases of central nervous
system irradiation of the tumor is the
1) Metastases are more common
first choice therapy, not surgery.
than primary tumors.
2) Most common first symptoms of
a CNS tumor in the case of low-grade 1st, 2nd, and 4th answers are
A)
correct
tumors are the symptoms of increased
brain pressure. In case of high grade 1st, 2nd, 3rd and 5th answers are
B)
tumors, epileptic seizures are more correct
common signs. 2nd, 5th and 6th answers are
C)
3) The survival probability of correct
oligodendroglioma is worse than 1st, 2nd, 4th and 6th answers are
D)
astrocytoma. correct
4) Glioblastoma multiforme is the 2nd, 5th and 6th answers are
E)
most common adult primary brain correct
tumor. Show comment and answer
5) Anaplastic gliomas have worse INT - 17.118
prognosis than glioblastomas. It is true for the treatment of
6) Cancers of the central nervous malignant melanoma:
system do not give any metastasis.
1) In case of a suspected
1st, 2nd and 4th answers are melanoma, it is strictly forbidden to
A)
correct take a biopsy, immediate excision with
1st, 3rd, 5th and 6th answers are 2-3 mm intact edges of the suspected
B) lesion is required.
correct
2nd, 3rd and 5th answers are 2) Superficially spreading melanoma
C) is lentigo maligna.
correct
3) The necessary edge of the
1st, 2nd, 4th and 6th answers are
D) excision depends on the depth of the
correct
tumor.
E) 1st and 4th answers are correct
4) In the treatment of melanoma,
Show comment and answer clinical studies have shown that new
INT - 17.117
molecularly targeted therapies and
It is true for the treatment of brain vaccination may play a significant role.
tumors: 5) Metastasectomy is not
recommended due to the high
1) In case of cerebral metastases,
metastatic tendency of melanoma
active oncological treatment is not an
malignant.
option.
6) In case of melanoma with a patient rejects the surgery, definitive
thickness greater than 4 mm, or with radio- or radiochemotherapy is
an ulceration, regional block dissection recommended.
is required. 2) Positive resection edge or pN1-2
status requires additional
1st, 2nd, and 3rd answers are chemotherapy or radiotherapy.
A)
correct 3) In case of potentially resectable
1st, 3rd, and 5th answers are cases, curative perioperative chemo-
B) or radiochemotherapy is
correct
3rd, 4th, and 6th answers are recommended.
C) 4) If the tumor is greater than 5 cm
correct
or lymph nodes are involved,
D) 3rd and 4th answers are correct
chemotherapy is required, even if R0
1st, 2nd, 5th and 6th answers are resection was performed.
E)
correct
5) Brachytherapy is an option in
Show comment and answer case of central bronchus cancers
INT - 17.119
6) Active tbc, inflammation,
It is true for the malignant tumors of
empyema or haemoptoe are
the lung:
contraindications for radiation therapy.
1) 80-85% of the tumors are non-
small cell lung cancer (NSCL). 1st, 2nd and 3rd answers are
A)
correct
2) 80% of the adenocarcinomas are
centrally localized processes. 2nd, 3rd, 4th and 5th answers are
B)
3) Squamous cancers are more correct
commonly peripheral. 1st, 4th, and 6th answers are
C)
4) Squamous cancers have worse correct
prognosis than adenocarcinomas and 1st, 4th, 5th and 6th answers are
D)
evolve especially in smokers. correct
5) Pulmonary malignancies are E) all of the answers are correct
generally histologically homogeneous, Show comment and answer
combinations of different tumor types INT - 17.121
are rarely observed. tyrosine kinase inhibitors can be
6) Non-small cell lung cancer is not efficient.
a separate entity, segregation into
subgroups is essential to determine 1) A paradigm-change was made in
personalized therapy. their treatment.
2) In the case of non-squamous cell
1st, 2nd, and 3rd answers are carcinoma, EGFR and KRAS mutation
A)
correct analysis are essential for therapeutic
2nd, 3rd,4th and 5th, answers are decision.
B) 3) The use of VEGF and EGFR
correct
1st, 4th, and 6th answers are inhibitors is part of everyday practice.
C) 4) If EGFR activating mutation
correct
cannot be detected, but if an ALK
1st, 4th, 5th and 6th answers are
D) mutation is onset, ALK
correct
5) EGFR tyrosine kinase inhibitors
E) all of the answers are correct
do not pass through the blood-brain
Show comment and answer barrier, so they cannot be used in
INT - 17.120
brain metastases.
It is true for the treatment of non-
6) Vaccination is proven to be
small cell lung cancer:
effective in clinical trials.
1) In case of a reduced
cardiopulmonary reserve or if the 1st, 2nd and 3rd answers are
A)
correct
1st, 2nd, 3rd and 4th answers are
B)
correct 1st, 2nd, 4th and 5th answers are
A)
1st, 4th, and 6th answers are correct
C)
correct 1st, 2nd, and 4th answers are
B)
1st, 4th, 5th and 6th answers are correct
D)
correct 2nd, 3rd, and 4th answers are
C)
E) all of the answers are correct correct
Show comment and answer D) 1st and 5th answers are correct
INT - 17.122 E) all of the answers are correct
What are the “B” symptoms of Show comment and answer
Hodgkin's disease? INT - 17.125
Burkitt-lymphoma is characterized by:
1) Sweat
2) Weight loss 1) the endemic form is related to
3) Fever malaria infection
4) Diarrhea 2) B-cell hyperplasia
5) Itching 3) activation of the Epstein-Barr
virus
1st, 2nd, and 3rd answers are 4) chromosome translocation (8.14)
A)
correct 5) progression into CLL
3rd, 4th and 5th answers are
B)
correct 1st, 2nd, and 4th answers are
A)
C) 4th and 5th answers are correct correct
D) all of the answers are correct 1st, 3rd, and 5th answers are
B)
Show comment and answer correct
INT - 17.123 1st, 2nd, 3rd and 4th answers are
C)
Which factors help in the development correct
of lymphomas? 1st, 3th, and 4th answers are
D)
correct
1) AIDS E) all of the answers are correct
2) Wiskott-Aldrich syndrome
Show comment and answer
3) Wegener granulomatosis Association question
4) effective chemotherapy Which cytostaticum belongs to which
5) Immunizations group of compounds?

1st, 2nd, 3rd and 4th answers are INT - 17.126 Alkylating agent -
A)
correct INT - 17.127 Antimetabolite -
B) 1st and 4th answers are correct INT - 17.128 Alkaloid -
C) only answer 1 is correct INT - 17.129 Antibiotic -
1st, 2nd, and 4th answers are INT - 17.130 Nitrosourea -
D)
correct
E) all of the answers are correct A) Actinomycin D
B) vinblastine
Show comment and answer
INT - 17.124 C) carmustin
Which factors or diseases caused by D) melphalan
them can be associated with the onset E) 5-fluorouracil
of malignant tumors?

1) Schistosoma haematobium
2) Helicobacter pylori
3) Streptococcus pyogenes
4) Aspergillus flavus
5) Actinomycosis
Association question INT - 17.147 Lung cancer -
Which cytostatic agent causes toxic INT - 17.148 Phaeochromocytoma -
damage to the listed organs - INT - 17.149 Carcinoid -
primarily? INT - 17.150 Hepatoblastoma -

INT - 17.131 5-fluorouracil - A) Serotonin


INT - 17.132 cisplatin - B) AFP
INT - 17.133 cyclophosphamide - C) erythropoetin
INT - 17.134 adriamycin - D) catecholamines
INT - 17.135 bleomycin - E) ADH
INT - 17.151
A) kidney A milk-like ascites indicates what of
B) heart the following? (simple choice)
C) lung
D) bladder A 65 year old woman is complaining
E) Intestine about an abdominal distension which
Association question increased in the last 2 weeks. In the
Which factor plays an important role in last 6 months she had a 5.5 kg weight
the development of certain listed loss, however, in the last 2 weeks her
tumors? weight increased with almost 3 kg.
Physical examination showed a
INT - 17.136 Asbestos - cachectic woman, with faint mucous
INT - 17.137 Benzene exposition - membranes and with a visible ascites.
INT - 17.138 UV radiation - During the discharge of ascites it
INT - 17.139 Chemotherapy - proved to be milk-like. Also a painless,
INT - 17.140 Cigarette - 3x4 cm sized, enlarged left inguinal
lymph node is detected. Htc: 0,30, fvs:
A) larynx cancer 9,8 G / l.
B) mesothelioma
C) Hodgkin's disease A) transsudatum
D) skin cancer B) chylus
E) leukemia C) none
Association question
Find a correlation between viruses and D) all
tumors that are probably involved in Show comment and answer
the development of the tumors. INT - 17.152
The milk-like, above listed type of
INT - 17.141 EBV - ascites may occur in the following
INT - 17.142 HBV - cases: (multiple choice)
INT - 17.143 HTLV-1 -
INT - 17.144 HHV-8 - A 65 year old woman is complaining
INT - 17.145 HPV - about an abdominal distension which
increased in the last 2 weeks. In the
A) cervical cancer last 6 months she had a 5.5 kg weight
B) Burkitt lymphoma loss, however, in the last 2 weeks her
C) Kaposi sarcoma weight increased with almost 3 kg.
D) liver cancer Physical examination showed a
E) T cell lymphoma cachectic woman, with faint mucous
Association question membranes and with a visible ascites.
Find a correlation between tumors and During the discharge of ascites it
the products often secreted by them proved to be milk-like. Also a painless,
which are frequently causing 3x4 cm sized, enlarged left inguinal
paraneoplastic symptoms: lymph node is detected. Htc: 0,30, fvs:
9,8 G / l.
INT - 17.146 Kidney cancer -
1) lymphoma lymph node is detected. Htc: 0,30, fvs:
2) pancreatic cancer 9,8 G / l.
3) tuberculosis
4) nephrosis syndrome A) colon cancer
B) lymphoma
1st, 2nd and 3rd answers are C) syphilis
A)
correct
D) gastric cancer
B) 1st and 3rd answers are correct
E) pneumococcal peritonitis
C) 2nd and 4th answers are correct
D) only 4 is correct

INFECTOLOGY
E) all of the answers are correct

Show comment and answer
INT - 18.1
INT - 17.153
Serologic test results after Hepatitis B
The final diagnosis can be determined
vaccination:
by the following test: (simple choice)

A 65 year old woman is complaining A) HBsAg positive


about an abdominal distension which B) anti-HBs positive
increased in the last 2 weeks. In the C) anti-HBc positive
last 6 months she had a 5.5 kg weight D) anti-HCV positive
loss, however, in the last 2 weeks her
Show comment and answer
weight increased with almost 3 kg. INT - 18.2
Physical examination showed a Definition of sepsis:
cachectic woman, with faint mucous
membranes and with a visible ascites. A) intermittent fever
During the discharge of ascites it
B) bacteraemia
proved to be milk-like. Also a painless,
3x4 cm sized, enlarged left inguinal elevated ESR (erythrocyte
C)
lymph node is detected. Htc: 0,30, fvs: sedimentation rate) and fever
9,8 G / l. a condition involving organ
disfunction due to systemic immune
D)
A) gastrointestinal examinations response in which the role of
infection can be seen or suspected.
B) lymph node biopsy
Show comment and answer
C) rectum biopsy INT - 18.3
D) iv. pyelographia Which of the following statements is
E) arteriogram true about antibiotic therapy in severe
Show comment and answer sepsis?
INT - 17.154
The most likely diagnosis: (simple definitive antimicrobial therapy
choice) A) based on the results of blood
culture and sensitivity test
A 65 year old woman is complaining initial empiric therapy with the intent
about an abdominal distension which to cover multiple possible
B)
increased in the last 2 weeks. In the pathogens commonly associated
last 6 months she had a 5.5 kg weight with the septic focus
loss, however, in the last 2 weeks her combination of at least two
weight increased with almost 3 kg. C)
bactericidal antibiotics
Physical examination showed a the newest and the most expensive
cachectic woman, with faint mucous D)
antibiotic should be administered
membranes and with a visible ascites.
Show comment and answer
During the discharge of ascites it INT - 18.4
proved to be milk-like. Also a painless, The role of Streptococcus pneumoniae
3x4 cm sized, enlarged left inguinal should be considered in community
acquired pneumonia in which of the D) cat scratch disease
following cases? E) tularaemia
Show comment and answer
A) always INT - 18.8
if the patient underwent Tetanus prophylaxis should be
B)
splenectomy administered in which of the following
if the patient has chest pain when cases?
C)
breathing and has purulent sputum
if there is a lobar consolidation on Every type of injury apart from the
D) A) age and the vaccination status of
the chest X-Ray
Show comment and answer the patient.
INT - 18.5 Every type of injury if the patient
A patient has trismus, risus sardonicus has been vaccinated but the last
B)
and opisthotonus. What is the most dose had been given more than 5
likely diagnosis? years ago
Every type of injury if the patient
C)
A) plague has never been vaccinated.
B) gas gangrene D) In every case.
C) tetanus E) B+C
D) rabies Show comment and answer
INT - 18.9
Show comment and answer
INT - 18.6 Adequate treatment of uncomplicated
A 60-year-old patient presents to his infectious mononucleosis is:
primary care provider with headache
and severe muscle pain. He has fever, A) Doxycyclin
itchy skin, and periorbital oedema. B) Ampicillin
Two weeks ago he ate wild boar (which C) Steroids
was cooked rare). His blood tests D) Symptomatic
reveal elevated CK (creatine kinase)
E) Penicillin+ steroids
and eosinophilia. What is the most
likely diagnosis? Show comment and answer
INT - 18.10
A patient has periodic attacks of
A) Salmonellosis
sequential chills and fever, dark urine
B) Trichinellosis and hepatosplenomegaly. He has a
C) Echinococcosis history of returning from the tropics.
D) Dysentery What is the most likely diagnosis?
Show comment and answer
INT - 18.7 A) hepatitis A
A 19-year-old student boy has sore B) malaria
throat for two weeks. A few days after C) plague
the first symptoms he became febrile
D) leprosy
and generalized lymphadenopathy
developed. He has lymphocytosis, E) none of the above
thrombocytopenia, and elevated liver Show comment and answer
enzymes. He lives in the countryside, INT - 18.11
with a dog and a cat. Insects bite him Which of the following treatment
frequently. What is the most likely should be chosen in severe systemic
diagnosis? enterococcal infection?

A) Lyme-disease A) penicillin G + amikacin


B) lymphoma B) vancomycin + ceftriaxon
C) infectious mononucleosis C) ciprofloxacin + oxacillin
D) ampicillin + gentamicin the patient has pyuria and elevated
D)
E) high-dose ampicillin CRP
Show comment and answer Show comment and answer
INT - 18.12 INT - 18.16
Screening and treatment of A 63-year-old patient had been treated
asymptomatic bacteriuria is required in with COPD for 2 weeks at the division
which the following cases? of pulmonology where he got
antibiotics and steroid bolus. Two days
A) before catheter insertion ago profuse diarrhea and fever have
been started a few hours after eating
B) in diabetic females
chicken soup. Other members of his
C) in pregnant women family (who ate with him) have no
in patients using intermittent symptoms. Which of the following is
D) catheter due to transverse cord the most likely cause of his symptoms?
lesion
in patients who have permanent A) salmonellosis
E)
indwelling catheter Clostridium difficile associated
Show comment and answer B)
diarrhea
INT - 18.13
C) campylobacteriosis
What is the acceptable efficacy
probability of an empiric (combined) D) intestinal tumor
antibiotic treatment chosen in severe Show comment and answer
life-threatening infection? INT - 18.17
Which of the following is the most
A) 50% accurate step in diagnosis of
Clostridium difficile associated
B) 60%
diarrhea?
C) 80%
D) more than 90% A) stool sample culture test
Show comment and answer detection of Clostridium difficile A+B
INT - 18.14 B)
toxins in stool specimens
What is the initial treatment for chronic
detection of Clostridium difficile
Hepatitis C infection in Hungary? C)
antigen and pus in stool
D) history is enough
A) acyclovir+interferon-α
Show comment and answer
B) corticosteroids+interferon-α INT - 18.18
C) ribavirin+pegylated interferon-α Which of the following antimicrobial
D) ribavirin agents is recommended for use in
E) interferon- γ methicillin-sensitive Staphylococcus
Show comment and answer aureus caused severe sepsis?
INT - 18.15
How can you diagnose urinary tract A) vancomycin
infection in a febrile patient who has B) flucloxacillin
no other symptoms? C) clindamycin
D) gentamycin
urinalysis shows pyuria and
A) E) ceftriaxon
bacteriuria
Show comment and answer
urinalysis shows pyuria and urine
INT - 18.19
culture is positive with the presence
B) Which of the following drugs has to be
of uropathogen bacteria at high
chosen in case of severe cellulitis with
colony counts.
systemic symptoms?
C) permanent catheter using patient
A) co-trimoxazole
B) cefazolin
C) roxithromycin A) white blood cell differential count
D) ciprofloxacin B) complete blood count
E) all of the above C) erythrocyte sedimentation rate
Show comment and answer D) CRP
INT - 18.20 E) PCT
Which of the following diagnostic steps Show comment and answer
is required for the diagnosis of INT - 18.25
erythema migrans? Which of the following bacteria is the
most common cause of toxic shock
A) serology syndrome?
B) culture
C) clinical features A) Streptococcus agalactiae
D) histology B) Streptococcus pyogenes
Show comment and answer C) Streptococcus bovis
INT - 18.21 D) Group C and group G Streptococci
Which of the following statements is E) Streptococcus pneumoniae
true about spontaneous bacterial
Show comment and answer
peritonitis? INT - 18.26
Tuberculous meningitis is suspected.
A) it is a rare disease What is the most appropriate initial
it can only appear in a patient with step in management?
B)
liver disease
C) lack of abdominal symptoms A) making decision based on culture
D) it is always associated with fever administer antituberculous drugs
B)
Show comment and answer immediately
INT - 18.22 there is nothing to do because TB
C)
Which of the following prophylaxis is meningitis does not exist
recommended for patients underwent symptomatic treatment can be
splenectomy? D)
enough
Show comment and answer
pneumococcal, meningococcal and INT - 18.27
A) Haemophilus influenzae type b Which of the following conditions is not
vaccines a risk factor of community acquired
B) crystalline penicillin for 2 weeks pneumonia?
C) pneumococcal vaccine
D) amoxicillin therapy for years A) patient more than 65 years old
Show comment and answer B) hypertension
INT - 18.23 C) diabetes mellitus
Which of the following anitiotics has no D) uraemia
efficacy against Pseudomonas?
E) long-term steroid treatment
A) piperacillin-tazobactam Show comment and answer
INT - 18.28
B) ciprofloxacin Which of the following bacteria is the
C) ceftazidime most common cause of purulent
D) ceftriaxone meningitis?
E) colistin
Show comment and answer A) E. coli
INT - 18.24 B) Haemophilus influenzae
Which of the following laboratory tests C) Streptococcus pneumoniae
is the most specific for sepsis? D) Listeria monocytogenes
Show comment and answer PCT is stored in the cells as a result
INT - 18.29 C)
of antibiotic therapy
Which of the following signs is one of supportive steroid therapy stops the
the major criteria of infective D)
production of PCT
endocarditis?
Show comment and answer
INT - 18.34
A) fever Which of the following statements is
B) positive blood culture false about the transmission of
C) murmur tularemia?
D) erythrocyte sedimentation rate
E) skin signs can be acquired by tick or horse-fly
A)
bite
Show comment and answer
INT - 18.30 may be spread through
Terminal ileitis and mesenteric adenitis B) microtrauma during meat
mimicking acute appendicitis can be preparation
seen in which of the following fecal-oral transmission from person
C)
diseases? to person
can be transmitted through
D)
A) abdominal typhus inhalation of contaminated dust
B) campylobacteriosis Show comment and answer
INT - 18.35
C) E.coli associated gastroenteritis
Which of the following statements is
D) yersiniosis true about empiric antibiotic treatment
Show comment and answer for severe nosocomial Gram-negative
INT - 18.31 bacterial infections?
Which of the following diseases is
associated with hydrophobia? A) combination of antibiotics is needed
it is important to know the local flora
A) plague B)
and antibiotic resistance rates
B) tetanus
antibiotic treatment should last for
C) rabies C)
10 days
D) all of the above Show comment and answer
E) none of the above INT - 18.36
Show comment and answer Which of the following statements is
INT - 18.32 true about actinomycosis?
Which of the following conditions can
be a manifestation of Lyme-disease? it is a disease characterized by
A)
acute fever
A) erythema chronicum B) often occurs with a tumor
B) carditis it is always associated with regional
C)
C) arthritis lymphadenopathy
D) neurological signs it is a disease involving generalized
D)
lymphadenopathy
E) all of the above
Show comment and answer
Show comment and answer INT - 18.37
INT - 18.33
Is it necessary to collect blood cultures
Which of the following statements is
when FUO is suspected?
true if PCT decreases during
treatment?
it is not, because the most common
A)
cause of FUO is cancer
A) antibiotic therapy is adequate
it is not, because viral infections are
lack of PCT production results from B) more common than bacterial
B)
multiple organ failure infections
yes it is, because endocarditis is B) false
C) one of the most common causes of Show comment and answer
FUO INT - 18.43
yes it is, because rickettsiosis and Pulmonary symptoms of ascariasis are
D) mycoplasma endocarditis can be associated with urticaria, eosinophilia
detected by blood cultures and angioneurotic edema.
Show comment and answer
INT - 18.38 A) true
Which of the following diagnostic tests B) false
should be performed in Staphylococcus
Show comment and answer
aureus bacteremia? INT - 18.44
Typical symptoms of staphylococcal
A) chest X-ray food poisoning are suddenly developed
B)
blood cultures if the patient is febrile vomiting, diarrhea and high fever
despite of treatment (more than 39°C/102,2°F), 1-2 hours
C) abdominal ultrasound after the consumption of infected food.
D) echocardiography
E) all of the above A) true
Show comment and answer B) false
INT - 18.39 Show comment and answer
Early recognition of herpes simplex INT - 18.45
encephalitis is critical, because it can The most common cause of acute
be cured with acyclovir and early exacerbation of COPD is Haemophilus
administration of acyclovir decreases influenzae because of its persistent
mortality. colonization on the respiratory
mucosa.
A) true
B) false A) true
Show comment and answer B) false
INT - 18.40 Show comment and answer
Laboratory diagnosis of Hepatitis C INT - 18.46
infection is based on detection of anti- Ceftriaxone is effective against
hepatitis C antibodies (anti-HCV). penicillin resistant S. pneumoniae in
most cases because resistance
A) true mechanism is based on beta-lactamase
production.
B) false
Show comment and answer
both the statement and the
INT - 18.41
A) explanation are true and a causal
Iron supplementation in severe
relationship exists between them
infections is needed otherwise the
patient will have iron deficiency both the statement and the
anaemia. B) explanation are true but there is no
causal relationship between them
A) true the statement is true, but the
C)
explanation is false
B) false
the statement is false, but the
Show comment and answer D)
explanation itself is true
INT - 18.42
Patients suffering diabetes mellitus are both the statement and the
E)
susceptible for Staphylococcus aureus explanation are false
caused infections. Show comment and answer
INT - 18.47
A) true Antibiotic prophylaxis in Lyme disease
is not needed because the side-effects
of prophylactic antibiotics have greater the statement is true, but the
C)
risk than the chance of developing explanation is false
Lyme disease. the statement is false, but the
D)
explanation itself is true
both the statement and the both the statement and the
A) explanation are true and a causal E)
explanation are false
relationship exists between them
Show comment and answer
both the statement and the INT - 18.50
B) explanation are true but there is no The incubation period of Lyssa virus
causal relationship between them infection depends on how far the virus
the statement is true, but the entry is from the brain because the
C)
explanation is false virus travels along the axons at a rate
the statement is false, but the of 1-2cm/day to enter into the central
D)
explanation itself is true nervous system.
both the statement and the
E)
explanation are false both the statement and the
Show comment and answer A) explanation are true and a causal
INT - 18.48 relationship exists between them
Antibiotics should be avoided in both the statement and the
salmonella gastroenteritis because B) explanation are true but there is no
antibiotic treatment may prolong the causal relationship between them
duration of fecal excretion results in the statement is true, but the
C)
higher resistance and more severe explanation is false
infection. the statement is false, but the
D)
explanation itself is true
both the statement and the both the statement and the
A) explanation are true and a causal E)
explanation are false
relationship exists between them
Show comment and answer
both the statement and the INT - 18.51
B) explanation are true but there is no Macrolides are recommended in
causal relationship between them urinary tract infections in pregnant
the statement is true, but the women, because macrolides can be
C)
explanation is false administered in pregnancy.
the statement is false, but the
D)
explanation itself is true both the statement and the
both the statement and the A) explanation are true and a causal
E) relationship exists between them
explanation are false
Show comment and answer both the statement and the
INT - 18.49 B) explanation are true but there is no
Aminoglycosides must be administered causal relationship between them
once a day because the once-daily the statement is true, but the
C)
dosing of aminoglycosides is explanation is false
associated with higher peak the statement is false, but the
concentration, higher efficacy, and less D)
explanation itself is true
toxicity due to complete excretion both the statement and the
within 24 hours. E)
explanation are false
Show comment and answer
both the statement and the INT - 18.52
A) explanation are true and a causal Empiric combined antibiotic treatment
relationship exists between them is recommended in severe life-
both the statement and the threatening infections, because the
B) explanation are true but there is no doses of antibiotics can be lowered
causal relationship between them which is cheaper.
Association question
both the statement and the A 20-year-old woman presents to her
A) explanation are true and a causal primary care provider with the
relationship exists between them following symptoms. Match each
both the statement and the clinical presentation with one
B) explanation are true but there is no diagnosis!
causal relationship between them
the statement is true, but the INT - fever, tonsillopharyngitis, -
C) 18.55 cervical lymphadenopathy,
explanation is false
hepatosplenomegaly, right
the statement is false, but the
D) shift in white blood cell
explanation itself is true
count
both the statement and the
E) INT - sharing drinks, mucosal -
explanation are false
18.56 hyperemia, sore throat,
Show comment and answer painful chewing, swollen
INT - 18.53
cervical lymph glands
Empiric antibiotic therapy in aspiration
INT - lethargy, headache, fever, -
pneumonia should cover anaerobes,
18.57 asymmetric facial swelling,
because typical pathogens origin from
dysphagia, painful chewing,
the oral flora where there is a
meningeal irritation signs
significant number of anaerobes.
A) stomatitis
both the statement and the B) epidemic parotitis
A) explanation are true and a causal
C) infectious mononucleosis
relationship exists between them Association question
both the statement and the A patient has pneumonia confirmed by
B) explanation are true but there is no chest X-ray. Match each clinical
causal relationship between them presentation with one or more
the statement is true, but the pathogen(s)!
C)
explanation is false
the statement is false, but the INT - severe headache, sore -
D)
explanation itself is true 18.58 throat, and earache
both the statement and the INT - diarrhea, and -
E) 18.59 hyponatraemia
explanation are false
Show comment and answer INT - sore throat, muscle pain, -
INT - 18.54 18.60 joint pain, WBC count: 3,6
Treatment for leptospirosis has to be G/L, CRP: 2mg/l
started within 48 hours, because INT - shivering, chest pain when -
antibiotics are often ineffective during 18.61 breathing, leukocytosis,
leptospiremia. elevated CRP, and PCT
INT - shivering, chest pain when -
both the statement and the 18.61 breathing, leukocytosis,
A) explanation are true and a causal elevated CRP, and PCT
relationship exists between them INT - erythema multiforme -

both the statement and the 18.62


B) explanation are true but there is no INT - splenectomy 1 month ago -
causal relationship between them 18.63
the statement is true, but the
C) A) Mycoplasma pneumoniae
explanation is false
B) Streptococcus pneumoniae
the statement is false, but the
D) C) Influenza virus
explanation itself is true
D) Legionella spp.
both the statement and the
E)
explanation are false
Show comment and answer
Association question Association question
A patient has pneumonia confirmed by Match each of the following bacteria
chest X-ray. Match each clinical with the most likely effective antibiotic
presentation with one or more treatment!
pathogen(s)!
INT - MRSA -
INT - severe headache, sore - 18.67
18.58 throat, and earache INT - Enterococcus faecalis -
INT - diarrhea, and - 18.68
18.59 hyponatraemia INT - Streptococcus pyogenes -
INT - sore throat, muscle pain, - 18.69
18.60 joint pain, WBC count: 3,6 INT - Streptococcus -
G/L, CRP: 2mg/l 18.70 pneumoniae
INT - shivering, chest pain when - INT - ESBL-producing E.coli -
18.61 breathing, leukocytosis, 18.71
elevated CRP, and PCT
INT - shivering, chest pain when - A) imipenem/cilastatin
18.61 breathing, leukocytosis, B) penicillin
elevated CRP, and PCT C) ceftriaxone
INT - erythema multiforme - D) vancomycin
18.62 E) ampicillin
INT - splenectomy 1 month ago -
18.63

A) Mycoplasma pneumoniae
B) Streptococcus pneumoniae
C) Influenza virus
D) Legionella spp.
Association question
Match each clinical presentation with
the most likely pathogen!

INT - A disease mainly occurs in -


18.64 infants and toddlers
characterized by fever,
vomiting and watery
diarrhea. It peaks in
fall/winter, it is highly
contagious and can lead to
dehydration.
INT - A mild disease in adults -
18.65 associated with diarrhea,
headache, abdominal and
muscle pain.
INT - A mild disease involving -
18.66 fever, upper respiratory
symptoms, vomiting and
diarrhea. Most commonly
affects toddlers and it is
often followed by lactose
intolerance.

A) adenovirus
B) Norwalk-virus
C) rotavirus

S-ar putea să vă placă și